You are on page 1of 144

2021/‫‏‬6/‫‏‬22 Immersive Reader

The best treatment for Hyperglycemic hyperosmolar syndrome would be which of


the following:
No answer selected.

Oral rehydration

Correction of hyperkalemia

The blood glucose should be lowered as rapidly as possible

Subcutaneous insulin shots

Volume resuscitation with normal saline infusion

Incorrect

Incorrect!

This condition usually occurs in middle-aged and elderly patients with mild
diabetes.
Volume resuscitation to restore blood pressure, mental status, and urine
output is the initial goal. Oral rehydration is usually not sufficient due to the
extensive dehydration and ongoing losses that occur. Intravenous insulin is
needed to gradually reduce blood glucose levels. Rapidly reducing the glucose or
acutely reducing it below 250 mg/dL is much more likely to result in cerebral
edema.

Marx, JA (ed). Rosen’s Emergency Medicine (6th ed). Mosby Elsevier, 2006

Kronenberg HM, Melmed S (eds). Williams Textbook of Endocrinology (11th


ed.). Saunders Elsevier, 2008.

Goldman L, Ausiella D (eds). Cecil Medicine (23rd ed). Saunders Elsevier, 2008.

2. Incorrect

Question Tools:

The hazards of lowering the blood glucose too rapidly in the patient with
Hyperglycemic Hyperosmolar Syndrome include which of the following:
No answer selected.

…me.com%2Fcustomscrip%2F154857%3Fscored%263739a18c-0c68-43cc-a4cb-b8b99e9bfd72%3Da45491cd-af54-4004-a990-00ee5e3871ae 1/144
2021/‫‏‬6/‫‏‬22 Immersive Reader

Cerebral edema (mainly when the glucose goes rapidly below 250) due to water
being pulled into brain cells by the higher intracellular osmolality

Rebound hyperglycemia

Acute hepatic failure

Hyperkalemia due to potassium accompanying the glucose efflux out of cells

Hypotensive shock

Incorrect

Incorrect!

Correction of glucose too rapidly can lead to osmolar shifts of water into brain
cells. Restoration of volume losses can in itself lead to decreased glucose levels
increasing insulin’s effect on blood glucose. Potassium influxes into cells leading
to serum hypokalemia.

Marx, JA (ed). Rosen’s Emergency Medicine (6th ed). Mosby Elsevier, 2006

Kronenberg HM, Melmed S (eds). Williams Textbook of Endocrinology (11th


ed.). Saunders Elsevier, 2008.

Goldman L, Ausiella D (eds). Cecil Medicine (23rd ed). Saunders Elsevier, 2008.

3. Incorrect

Question Tools:

A common lab finding in Hyperglycemic Hyperosmolar Syndrome would be


which of the following:

No answer selected.

Potassium depletion is rare

Metabolic alkalosis is common

BUN is usually low due to osmotic diuresis

Blood glucose is usually >600 mg/dL and osmolality is >340 mOsm/L

Serum sodium levels are usually elevated when corrected for hyperglycemia

…me.com%2Fcustomscrip%2F154857%3Fscored%263739a18c-0c68-43cc-a4cb-b8b99e9bfd72%3Da45491cd-af54-4004-a990-00ee5e3871ae 2/144
2021/‫‏‬6/‫‏‬22 Immersive Reader

Incorrect

Incorrect!

Prerenal azotemia prompts elevated BUN and creatinine levels. Fluid shifts from
intracellular to extracellular cause potassium derangements. Total body potassium
is usually depleted and the actual serum numbers can be either low or normal.
Metabolic acidosis develops from dehydration.

Marx, JA (ed). Rosen’s Emergency Medicine (6th ed). Mosby Elsevier, 2006

Kronenberg HM, Melmed S (eds). Williams Textbook of Endocrinology (11th


ed.). Saunders Elsevier, 2008.

Goldman L, Ausiella D (eds). Cecil Medicine (23rd ed). Saunders Elsevier, 2008.

4. Incorrect

Question Tools:

In Hyperglycemic Hyperosmolar Syndrome, serum ketones are absent and the


glucose is usually greater than __ mg/dL. The osmolality is greater than in DKA,
generally >350 mOsm/kg. In many cases there is no past history of diabetes, and
many of these patients do not require insulin once their acute problem is
corrected.

No answer selected.

200

300

600

500

400

Incorrect

Incorrect!

Unless the osmolality (calculated without the BUN since neurons are permeable
to urea) is >340, look for another cause of coma.

…me.com%2Fcustomscrip%2F154857%3Fscored%263739a18c-0c68-43cc-a4cb-b8b99e9bfd72%3Da45491cd-af54-4004-a990-00ee5e3871ae 3/144
2021/‫‏‬6/‫‏‬22 Immersive Reader

Apparently these patients produce sufficient insulin to prevent ketosis, but not to
inhibit gluconeogenesis or permit peripheral glucose uptake.

Marx, JA (ed). Rosen’s Emergency Medicine (6th ed). Mosby Elsevier, 2006

Kronenberg HM, Melmed S (eds). Williams Textbook of Endocrinology (11th


ed.). Saunders Elsevier, 2008.

Goldman L, Ausiella D (eds). Cecil Medicine (23rd ed). Saunders Elsevier, 2008.

5. Incorrect

Question Tools:

A common physical examination finding in Hyperglycemic Hyperosmolar


Syndrome would be which of the following:
No answer selected.

Kussmaul's respirations

Hypertension

Gastrointestinal upset

Focal neurologic deficits and seizures

Bradycardia due to AV node disruption

Incorrect

Incorrect!

Kussmaul's respirations usually imply acidosis, as in diabetic ketoacidosis, but


may sometimes occur in nonketotic hyperglycemic coma if lactic acidosis or renal
failure are profound.
Since many of these patients are elderly and have a history
of prior stroke, focal deficits are common. Subclinical focal defects may be
unmasked by hyperosmolarity. Tachycardia and hypotension are common
findings. Gastrointestinal upset is less common than in DKA.

Marx, JA (ed). Rosen’s Emergency Medicine (6th ed). Mosby Elsevier, 2006

Kronenberg HM, Melmed S (eds). Williams Textbook of Endocrinology (11th


ed.). Saunders Elsevier, 2008.

…me.com%2Fcustomscrip%2F154857%3Fscored%263739a18c-0c68-43cc-a4cb-b8b99e9bfd72%3Da45491cd-af54-4004-a990-00ee5e3871ae 4/144
2021/‫‏‬6/‫‏‬22 Immersive Reader

Goldman L, Ausiella D (eds). Cecil Medicine (23rd ed). Saunders Elsevier, 2008.

6. Incorrect

Question Tools:

Even without actual sodium depletion, hyperglycemia causes a decrease in the


serum sodium concentration of __ mEq/L per 100 mg/dL increase in blood
glucose. This effect is due to the physical displacement of sodium by glucose and
should be kept in mind when evaluating electrolyte levels in the hyperglycemic
patient.
No answer selected.

2.5

1.6

1.8

3.2

0.9

Incorrect

Incorrect!

For example, in a patient with a blood glucose of 1000 mg/dL and a serum
sodium of 124 mEq/L, the corrected sodium would be 124 + 1.6*[(1000-
100)/100] = 138.4.

Marx, JA (ed). Rosen’s Emergency Medicine (7th ed). Mosby Elsevier, 2009;
Chapter 123 - Electrolyte Disturbances > ... > Hyponatremia6

Kronenberg HM, Melmed S (eds). Williams Textbook of Endocrinology (11th


ed.). Saunders Elsevier, 2008.

Goldman L, Ausiella D (eds). Cecil Medicine (23rd ed). Saunders Elsevier, 2008.

7. Incorrect

Question Tools:

…me.com%2Fcustomscrip%2F154857%3Fscored%263739a18c-0c68-43cc-a4cb-b8b99e9bfd72%3Da45491cd-af54-4004-a990-00ee5e3871ae 5/144
2021/‫‏‬6/‫‏‬22 Immersive Reader

Many drugs may precipitate an episode of nonketotic hyperglycemic coma


including which of the following:
No answer selected.

Mineralocorticoids

Angiotensin converting enzyme inhibitors

Hydroxyzine

Statins

Beta blockers

Incorrect

Incorrect!

Other medications include calcium channel blockers, diuretics, glucocorticoids,


phenytoin, mannitol, and neuroleptics.

Marx, JA (ed). Rosen’s Emergency Medicine (6th ed). Mosby Elsevier, 2006

Kronenberg HM, Melmed S (eds). Williams Textbook of Endocrinology (11th


ed.). Saunders Elsevier, 2008.

Goldman L, Ausiella D (eds). Cecil Medicine (23rd ed). Saunders Elsevier, 2008.

8. Incorrect

Question Tools:

Give the formula for calculating serum osmolality (Na and K are expressed in
mEq/l, BUN and glucose in mg/dl).
No answer selected.

mOsm/kg = Na/18 + K/2.8 + (glucose + BUN) * 2

mOsm/kg = 2 Na + glucose/18 + BUN/2.8

mOsm/kg = Na /18 + glucose/2.8 + BUN/3

mOsm/kg = 2Na + K + glucose/18 + BUN/2

mOsm/kg = Na + K + glucose/18 + BUN/2.8


…me.com%2Fcustomscrip%2F154857%3Fscored%263739a18c-0c68-43cc-a4cb-b8b99e9bfd72%3Da45491cd-af54-4004-a990-00ee5e3871ae 6/144
2021/‫‏‬6/‫‏‬22 Immersive Reader

Incorrect

Normal serum osmolality is 280 -300 mOsm/kg.

Serum osmolality is elevated in: dehydration, hypernatremia, diabetes insipidus,


uremia, hyperglycemia, mannitol Rx, ingestion of toxins (ethylene glycol,
methanol, ethanol), hypercalcemia, diuretics.

Serum osmolality is decreased in: SIADH, hyponatremia, overhydration,


Addison's disease, hypothyroidism.

Calculating the osmolal gap is mainly useful in the context of suspected toxic


alcohol ingestion. The measured serum osmolality will be higher than the
calculated one in certain alcohol ingestions because the alcohol contributes
(hidden) osmoles that are not taken into consideration in the calculated serum
osmolality. In most of the other conditions, the measured and calculated
osmolality will remain similar, even though the osmolality is elevated or
decreased.

A common formula [used, for example, by Rosen and the American Diabetic
Association (ADA)] for calculating serum osmolality is:

2Na + Glucose/18 + BUN/2.8

Several authors (for example, Ferri's 2012 Clinical Advisor) retain potassium in
the equation:

2([Na] + [K]) + Glucose/18 + BUN/2.8.

Several authors will also round the number 2.8 to the number 3 for ease of
calculation.

Osmolal gap = calculated osmolality minus measured osmolality.

Pearl: Notice that the above formula is calculating serum osmolality, not serum
osmolarity. In reality, the situation is more complex. Often, the measured value
will indeed reflect osmolality whereas the calculated value reflects osmolarity.
This is of little clinical consequence, but understanding this and using the correct
terms can prevent confusion when trying to communicate with others about these
concepts. See the following excerpt from Brian L. Erstad, Pharm.D:

"The terminology associated with calculated and measured osmotic activity is


often confusing and is not consistent in the medical literature. Osmotic
concentration determinations are typically expressed as either milliosmoles/
kilogram (mOsm/kg) of solvent, which is referred to as osmolality, or as

…me.com%2Fcustomscrip%2F154857%3Fscored%263739a18c-0c68-43cc-a4cb-b8b99e9bfd72%3Da45491cd-af54-4004-a990-00ee5e3871ae 7/144
2021/‫‏‬6/‫‏‬22 Immersive Reader

milliosmoles/liter (mOsm/L) of solution, which is referred to as osmolarity. The


selection of which term to use (osmolality or osmolarity) depends on how the
concentration was derived. When derived by an osmometer in clinical laboratories
that use a method such as freezing point depression of water (or less commonly,
the vapor pressure technique), the concentration is expressed in terms of solvent
and is appropriately referred to as osmolality. Bedside calculations of osmotic
activity by clinicians (using the patient's laboratory data), however, are usually
expressed in terms of solution, and hence the term osmolarity is
appropriate.Therefore, when evaluating published literature, the reader must refer
to the study methodology to determine which term is appropriate, since the
investigators may have converted osmolar units to osmolal units in an attempt to
increase the accuracy of calculated values, or they may have used a term such as
osmolality to describe both measured and calculated values.

Marx, JA (ed). Rosen's Emergency Medicine (76th ed). Chapter 124: "Diabetes
Mellitus and Disorders of Glucose Homeostasis" and chapter 153: "Toxic
Alcohols". Mosby Elsevier, 20096

Ferri: Ferri's Clinical Advisor 2012, ist ed.

Brian L. Erstad, Pharm.D. "Osmolality and Osmolarity: Narrowing the


Terminology Gap." Posted: 09/18/2003.

9. Incorrect

Question Tools:

A 65-year-old woman presents to you with complaints of mild diaphoresis,


palpitations, and hand tremor. She is alert and oriented to questions of person,
place, and time. She denies any perceptual disturbances or hallucinations. She
states that she frequently drinks ethanol, sometimes 1 L of vodka per day. Her last
drink was yesterday. Which of the following best describes this patient’s clinical
presentation?

No answer selected.

no abnormalities on her clinical examination or in her history

uncomplicated alcohol withdrawal

at risk for alcoholic withdrawal seizure

delirium tremens

…me.com%2Fcustomscrip%2F154857%3Fscored%263739a18c-0c68-43cc-a4cb-b8b99e9bfd72%3Da45491cd-af54-4004-a990-00ee5e3871ae 8/144
2021/‫‏‬6/‫‏‬22 Immersive Reader

alcoholic hallucinosis

Incorrect

Incorrect!

This patient is experiencing uncomplicated alcohol withdrawal. She is


manifesting autonomic hyperactivity (diaphoresis, palpitations) and hand tremor.
This is not delirium tremens, although many health care professionals may falsely
label all alcohol withdrawal as delirium tremens. According to the Diagnostic and
Statistical Manual of Mental Disorders, Fourth Edition, the diagnosis of alcohol
withdrawal requires recent cessation of alcohol use, which was previously heavy
or prolonged, and at least two or more of the following (which may develop a few
hours or a few days after the last drink):

Autonomic hyperactivity (i.e., sweating, pulse > 100)


Increased hand tremor
Insomnia
Nausea or vomiting
Transient visual, auditory, or tactile hallucinations/illusions
Psychomotor agitation
Anxiety
Grand mal seizures

These symptoms may result in significant distress or impairment, but they are not
better explained by a general medical condition or another mental disorder.

The patient in this case is alert and oriented, a fact that argues against the
diagnosis of delirium tremens. She denies any hallucinations, so a diagnosis of
alcoholic hallucinosis is not appropriate.

Approximately 10% of patients who experience alcohol withdrawal will develop a


seizure. For many patients, this seizure is the first manifestation of alcohol
withdrawal. Withdrawal seizures are generally short, generalized tonic-clonic,
with a very brief postictal period. Rarely (3% of patients), a patient experiencing
alcohol withdrawal may develop status epilepticus. It is worth noting that the
presence of alcohol withdrawal symptoms does not predict the development of
alcohol withdrawal seizures.

References:

American Psychiatric Association. Diagnostic and Statistical Manual of Mental


Disorders. 4th ed, 1994.

Gold JA, Nelson LS. Ethanol withdrawal. In: Goldfrank L, et al, eds. Goldfrank's
…me.com%2Fcustomscrip%2F154857%3Fscored%263739a18c-0c68-43cc-a4cb-b8b99e9bfd72%3Da45491cd-af54-4004-a990-00ee5e3871ae 9/144
2021/‫‏‬6/‫‏‬22 Immersive Reader

Toxicologic Emergencies. 9th ed, 2011.

Sullivan JT, Sykora K, Schneiderman J, et al. Assessment of alcohol withdrawal:


the revised clinical institute withdrawal assessment for alcohol scale (CIWA-Ar).
Br J Addict. 1989;84:1353-1357.

10. Incorrect

Question Tools:

A 62-year-old man presents to you with a profound alteration in mental status,


confusion, and somnolence. His blood alcohol level is measured at 250 mg/dL. A
head CT scan and laboratory studies are unremarkable. Of the following, how
quickly do you expect his blood alcohol level fall?

No answer selected.

half-life of ethanol is 2 hours

15-20 mg/dL per hour

too much individual variability to accurately predict

50 mg/dL per hour

administering intravenous (IV) fluid accelerates metabolism

Incorrect
Incorrect!

Ethanol elimination rapidly shifts from first order (a constant percentage of the
total quantity per hour) to zero order (a constant amount per hour). With zero
order metabolism, there is no half-life that can be determined. Therefore, there is
no absolute half-life for ethanol metabolism. A number of studies have
demonstrated that, on average, an adult metabolizes 7-10 g of ethanol per hour
and blood ethanol concentration will fall by 15-20 mg/dL per hour. Administering
IV fluid does not accelerate metabolism of ethanol.

Some individuals who are extremely tolerant to ethanol may see their metabolism
increase to 30 mg/dL; however, the tolerance of the patient in this case is not
known. Although there is individual variability in alcohol metabolism, an average
clearance rate of 20 mg/dL per hour has been repeatedly demonstrated, so this is a
good reference number.

…e.com%2Fcustomscrip%2F154857%3Fscored%263739a18c-0c68-43cc-a4cb-b8b99e9bfd72%3Da45491cd-af54-4004-a990-00ee5e3871ae 10/144
2021/‫‏‬6/‫‏‬22 Immersive Reader

References:

Brennan DF, Betzelos S, Reed R, et al. Ethanol elimination rates in an ED


population. Am J Emerg Med. 1995;13:276-280.

Gershman H, Steper J. Rate of clearance of ethanol from the blood of intoxicated


patients in the emergency department. J Emerg Med. 1991;9:307-311.

Li J, Mills T, Erato R. Intravenous saline has no effect on blood ethanol clearance.


J Emerg Med. 1999;17:1-5.

Yip L. Ethanol. In: Goldfrank L, et al, eds. Goldfrank’s Toxicologic Emergencies.


9th ed, 2011.

11. Incorrect

Question Tools:

A physiologic effect of the very high glucose levels seen in nonketotic


hyperglycemic coma is which of the following:

No answer selected.

Since the cell membranes are freely permeable to it, glucose moves in and out of
the cells to maintain osmotic equilibrium between the extra- and intracellular
compartments

The high glucose concentration exceeds the renal reabsorption capacity,


resulting in an osmotic diuresis, with depletion of Na, K, PO4, and magnesium

Electrolyte and H2O depletion is less than with diabetic ketoacidosis (DKA)

The shift of H2O out of the cells may accelerate hypotension prior to the
dehydration becoming severe

Acidosis quickly becomes severe shifting potassium into cells

Incorrect

Incorrect!

Dehydration and electrolyte depletion are generally greater than with DKA. The
cell membrane is not freely permeable to glucose but rather water moves out of
the cell to maintain osmotic equilibrium. Water shifts out of the cells can slow the
development of hypotension. Acidosis is usually less severe than DKA due to
some continued low levels of insulin production and less lipolysis.

Marx, JA (ed). Rosen’s Emergency Medicine (6th ed). Mosby Elsevier, 2006
…e.com%2Fcustomscrip%2F154857%3Fscored%263739a18c-0c68-43cc-a4cb-b8b99e9bfd72%3Da45491cd-af54-4004-a990-00ee5e3871ae 11/144
2021/‫‏‬6/‫‏‬22 Immersive Reader

Kronenberg HM, Melmed S (eds). Williams Textbook of Endocrinology (11th


ed.). Saunders Elsevier, 2008.

Goldman L, Ausiella D (eds). Cecil Medicine (23rd ed). Saunders Elsevier, 2008.

12. Incorrect

Question Tools:

A 65-year-old woman presents to you with complaints of mild diaphoresis,


palpitations, and hand tremor. She is alert and oriented to questions of person,
place, and time. She denies any perceptual disturbances or hallucinations. She
states that she frequently drinks ethanol, sometimes 1 L of vodka per day. Her last
drink was yesterday. Which of the following is the best management course for
this patient?

No answer selected.

administer intravenous (IV) fluids, observe the patient, then discharge without
any medications

administer intravenous (IV) ethanol

complete a medical evaluation, rule out other comorbid conditions, then provide
benzodiazepine therapy and outpatient follow-up

admit the patient with intravenous (IV) benzodiazepine therapy

prescribe clonidine to her as an outpatient

Incorrect

Because the patient has minimal symptoms, it is reasonable to consider outpatient


management. All patients with alcohol withdrawal should undergo a complete
medical evaluation to ensure that comorbid conditions are not present. In one
study, patients who exhibited a current lack of intoxication, no history of delirium
tremens or withdrawal seizures, no comorbid medical or psychiatric conditions,
and a Clinical Institute Withdrawal Assessment of Alcohol, Revised (CIWA-Ar)
scale score of less than 8 were safely treated as outpatients. The CIWA-Ar scale
contains 10 clinical categories that include both signs and symptoms of
withdrawal. It takes less than 5 minutes to complete.

Outpatient management also has significant cost savings. Oral benzodiazepines


such as chlordiazepoxide (50-100 mg) can be used to ameliorate the effects of
…e.com%2Fcustomscrip%2F154857%3Fscored%263739a18c-0c68-43cc-a4cb-b8b99e9bfd72%3Da45491cd-af54-4004-a990-00ee5e3871ae 12/144
2021/‫‏‬6/‫‏‬22 Immersive Reader

alcohol withdrawal in patients who are mildly symptomatic.

IV fluids are unlikely to ameliorate the patient’s current symptoms of alcohol


withdrawal. Withdrawal is complex and involves multiple neurotransmitter
systems, including gamma-aminobutyric acid (GABA) and glutamate. Chronic
alcohol consumption will down regulate GABA type A receptors and GABA
transmission while up regulating glutamate receptors. With the removal of alcohol
(an exogenous sedative), the body is in a mismatched state with excess excitation
relative to inhibition. Normal saline will not reverse this process.

IV ethanol has been used in some hospital settings to treat patients with ethanol
withdrawal. In one survey, 72% of 122 hospitals had staff members who
administered either IV or oral ethanol for the treatment of withdrawal. However,
there are few randomized controlled data to support its use. There are also
conflicting data as to whether IV ethanol is actually effective. Results from one
study suggested that ethanol therapy was no more effective than oral
flunitrazepam, while results from another study suggested that ethanol therapy
was inferior to diazepam. Because IV ethanol involves the need for frequent
blood monitoring, it results in vein irritation via the IV route and theoretically
inhibits wound healing, a method that is generally not recommended.

Clonidine (an alpha-1 antagonist) and beta blockers have been demonstrated to
reduce heart rate and blood pressure in clinical trials. However, these agents do
not address the underlying cause of these vital sign abnormalities; rather, they
potentially mask the symptoms of progressing withdrawal. By “masking”
abnormalities in vital signs, there is also concern of not administering sufficient
benzodiazepines to treat withdrawal. For this and several other reasons, these
agents are generally not recommended as sole therapy.

References:

Amato L, Minozzi S, Davoli M. Efficacy and safety of pharmacological


interventions for the treatment of the Alcohol Withdrawal Syndrome. Cochrane
Database Syst Rev. 2011;(6):CD008537.

Edelman EJ, Fiellin DA. In the Clinic. Alcohol Use. Ann Intern


Med. 2016;164(1):ITC1-16.

13. Incorrect

Question Tools:

Which of the following organs or organ systems is affected by alcohol use?

…e.com%2Fcustomscrip%2F154857%3Fscored%263739a18c-0c68-43cc-a4cb-b8b99e9bfd72%3Da45491cd-af54-4004-a990-00ee5e3871ae 13/144
2021/‫‏‬6/‫‏‬22 Immersive Reader

No answer selected.

liver, pancreas, esophagus, and cardiovascular and pulmonary systems

liver and pancreas

pulmonary system

esophagus

cardiovascular system

Incorrect
Incorrect!

Both acute and chronic alcohol abuse can have a detrimental effect on the
cardiovascular system. Dysrhythmia has been reported following acute and
chronic alcohol use. In addition, acute intoxication can result in decreased cardiac
output in patients with underlying cardiac disease who abuse alcohol as well as in
those who do not.

Alcohol decreases the ability of pulmonary macrophages to mobilize and clear


bacteria, a process that may increase the risk of aspiration and lobar pneumonia in
people who chronically abuse alcohol. Alcohol also induces bronchospasm in
patients with underlying pulmonary diseases such as asthma, ventricular ectopy,
and sleep apnea in those with chronic obstructive pulmonary disease.

The liver is the primary site of alcohol metabolism; therefore, it is a primary site
of chronic alcohol toxicity. The earliest changes that occur here include the
accumulation of fat in the hepatocytes (fatty liver) with transaminase elevations.
Alcoholic liver disease is the most common liver disorder in the Western
Hemisphere and is a leading cause of liver transplant. Hepatitis is also a common
condition that results from alcohol consumption and may occur in up to 35% of
people who chronically abuse alcohol.

Pancreatitis is a well-established phenomenon of alcohol abuse; however, its exact


mechanism remains unclear. Both acute and chronic pancreatitis may result from
the overconsumption of alcohol. Theories for the cause of the disease include
reflux of duodenal contents into the pancreatic and biliary tree, obstruction by the
pancreatic “juice” plug, and/or a direct toxic effect of alcohol on the pancreas.

People who abuse alcohol have a higher rate of esophagitis, gastric cancer, and
esophageal carcinoma than the general population. Vomiting is also common

…e.com%2Fcustomscrip%2F154857%3Fscored%263739a18c-0c68-43cc-a4cb-b8b99e9bfd72%3Da45491cd-af54-4004-a990-00ee5e3871ae 14/144
2021/‫‏‬6/‫‏‬22 Immersive Reader

among those who drink alcohol and may lead to Mallory-Weiss tears of the
esophagus or frank esophageal rupture (Boerhaave syndrome).

References:

Finnell J, et al. Alcohol-related disease. In: Marx JA, et al, eds. Rosen's
Emergency Medicine: Concepts and Clinical Practice. 7th ed, 2009.

Purohit V, Russo D. Cellular and molecular mechanisms of alcoholic hepatitis:


introduction and summary of the symposium. Alcohol. 2002;27:3.

Segal L. Alcohol and the heart. Med Clin North Am. 1984;68:147.

14. Incorrect

Question Tools:

Pathogenesis of hyperglycemic hyperosmolar syndrome includes which of the


following:
No answer selected.

Increased liver gluconeogenesis and glycogenolysis

Increased glucose excretion by kidneys

Increased cortisol production

Increased lipolysis

Increased insulin utilization

Incorrect

Incorrect!

Apparently there is sufficient insulin to inhibit ketogenesis, but not to stimulate


peripheral uptake of glucose.

Marx, JA (ed). Rosen’s Emergency Medicine (6th ed). Mosby Elsevier, 2006

Kronenberg HM, Melmed S (eds). Williams Textbook of Endocrinology (11th


ed.). Saunders Elsevier, 2008.

Goldman L, Ausiella D (eds). Cecil Medicine (23rd ed). Saunders Elsevier, 2008.

15. Incorrect

…e.com%2Fcustomscrip%2F154857%3Fscored%263739a18c-0c68-43cc-a4cb-b8b99e9bfd72%3Da45491cd-af54-4004-a990-00ee5e3871ae 15/144
2021/‫‏‬6/‫‏‬22 Immersive Reader

Question Tools:

A 14-year-old boy who plays football presents with an insidious onset of pain
located in the infrapatellar region.

Which of the following is the most likely diagnosis?

No answer selected.

plica syndrome

tibial shaft stress fracture

Osgood-Schlatter disease

anserine bursitis

Incorrect

Educational Objective:

Recognize the clinical presentation of Osgood-Schlatter disease.

Key Point:

Osgood-Schlatter disease is a childhood disorder associated with gradual knee


pain and is more common in children who play sports.

Explanation:

Osgood-Schlatter disease, also called tibial tubercle apophysitis, is caused by


repetitive microtrauma to the developing tibial tubercle in pubescent children, and
it is up to 5 times more common in children who play sports. The condition is a
traction phenomenon that results from the repetitive contraction of the quadriceps
through the patellar tendon where it is inserted at the skeletally immature tibial
tubercle. Osgood-Schlatter disease is sometimes misdiagnosed as an avulsion
fracture of the tibial tubercle because the distinction can be subtle (even on
radiography) because the immature tubercle has not yet fused to the tibia.

Typically, patients with Osgood-Schlatter disease can ambulate with pain and the
injury only gradually develops. Jumping and running, which can significantly
stress the insertion point of the patellar tendon at the tibial tubercle, may
aggravate Osgood-Schlatter disease. However, if the child cannot ambulate and
the injury occurred without a preceding event, a gradual worsening of the pain
and an eventual avulsion fracture of the tibial tubercle are likely.
…e.com%2Fcustomscrip%2F154857%3Fscored%263739a18c-0c68-43cc-a4cb-b8b99e9bfd72%3Da45491cd-af54-4004-a990-00ee5e3871ae 16/144
2021/‫‏‬6/‫‏‬22 Immersive Reader

References:

Palin DJ. Knee and lower leg. In: Walls R, et al, eds. Rosen's Emergency
Medicine: Concepts and Clinical Practice. 19th ed., 2018:698-722.

Sullivan JA. Osgood-Schlatter disease. Revised February 3, 2017. Accessed


January 18, 2018.

16. Incorrect

Question Tools:

A 65-year-old woman presents to you with complaints of mild diaphoresis,


palpitations, and hand tremor. She is alert and oriented to questions of person,
place, and time. She denies any perceptual disturbances or hallucinations. She
states that she frequently drinks ethanol, sometimes 1 L of vodka per day. Her last
drink was yesterday.

While in your presence, the patient has a seizure but experiences a rapid return to
consciousness. Which of the following is true regarding alcohol withdrawal
seizures as they relate to this case?

No answer selected.

The patient should be administered a benzodiazepine (e.g., lorazepam,


diazepam).

The patient is at risk for status epilepticus.

None of the options listed are correct statements.

The patient should be administered an anticonvulsant such as phenytoin and


admitted to the hospital.

The patient should be discharged from the hospital with an anticonvulsant.

Incorrect
Incorrect!

Administering a benzodiazepine, such lorazepam or diazepam, will address the


underlying neurochemical discrepancy that results from chronic alcohol use. Both
lorazepam and diazepam enhance the gamma-aminobutyric acid effect. In patients
with alcohol withdrawal who also lack a history of seizures, benzodiazepines

…e.com%2Fcustomscrip%2F154857%3Fscored%263739a18c-0c68-43cc-a4cb-b8b99e9bfd72%3Da45491cd-af54-4004-a990-00ee5e3871ae 17/144
2021/‫‏‬6/‫‏‬22 Immersive Reader

generally have sufficient anticonvulsant activity to prevent continued withdrawal


seizures.

Phenytoin should not be administered in the setting of an alcohol withdrawal


seizure or generally in the treatment of any toxicologic seizure. Its mechanism of
action does not address the underlying physiologic derangements that may occur
in toxin-mediated seizures.

The patient in this scenario does not necessarily need to be discharged with an
anticonvulsant because this alcohol withdrawal seizure does not indicate a seizure
disorder or the need for long-term anticonvulsant therapy.

Less than 3% of alcohol withdrawal seizures are seen in cases of status


epilepticus. The vast majority of alcohol withdrawal-related seizures are short,
tonic-clonic, and generalized in nature with a very brief postictal period.

References:

Gold JA, Nelson LS. Ethanol withdrawal. In: Goldfrank L, et al, eds. Goldfrank’s
Toxicologic Emergencies. 9th ed, 2011.

Finnell J, et al. Alcohol-related disease. In: Marx JA, et al, eds. Rosen's
Emergency Medicine: Concepts and Clinical Practice. 7th ed, 2009.

17. Incorrect

Question Tools:

A 42-year-old woman presents to you and is markedly confused, ataxic, and


alternating between agitation and lethargy. She is well known to your staff as a
frequent patient for alcohol intoxication. Her blood alcohol concentration is 0
mg/dL during this visit. Her vital signs demonstrate a heart rate of 125 and blood
pressure of 170/90. She is diaphoretic, tremulous, and is oriented to self only. She
has received lorazepam 4 mg with minimal response. What is the most
appropriate next management step?
No answer selected.

intubation and paralysis

administer alcohol orally

no management necessary

additional escalating doses of lorazepam

phenobarbital 240 mg intravenously, then discharge

…e.com%2Fcustomscrip%2F154857%3Fscored%263739a18c-0c68-43cc-a4cb-b8b99e9bfd72%3Da45491cd-af54-4004-a990-00ee5e3871ae 18/144
2021/‫‏‬6/‫‏‬22 Immersive Reader

Incorrect
Incorrect!

This patient is demonstrating many signs and symptoms of severe alcohol


withdrawal, including a hyperadrenergic state, an alteration in her level of
consciousness, and a decreased awareness of her surroundings. She is also
manifesting delirium tremens.

Lorazepam 4 mg is a fairly miniscule dose. Health care professionals should


prescribe appropriate doses of one class of medications prior to moving to
alternative classes. It is not uncommon for delirium tremens to require hundreds
of milligrams of a benzodiazepine to resolve withdrawal. One study of patients
admitted to a hospital demonstrated an average of 234 mg of diazepam required
during the first 24 hours of admission for alcohol withdrawal.

Phenobarbital is an acceptable medication in the treatment of alcohol withdrawal;


however, it may cause marked bradycardia and hypotension, and the effects are
often delayed for up to 40 minutes. Phenobarbital should be given when the
patient is resistant to benzodiazepines and typically only in an inpatient setting.

Intubation and paralysis will eliminate the peripheral manifestations of


withdrawal, but these effects do nothing to correct the underlying physiologic
derangements.

This patient is in marked withdrawal, so management is necessary to prevent


serious morbidity or mortality. Administrating alcohol by mouth is ill advised
because the patient is confused, agitated, and disoriented. It is also extremely
difficult to safely administer alcohol orally or intravenously.

Reference:

Gold JA, Nelson LS. Ethanol withdrawal. In: Goldfrank L, et al, eds. Goldfrank’s
Toxicologic Emergencies. 9th ed, 2011.

18. Incorrect

Question Tools:

A 37-year-old pregnant woman presents to you for evaluation. Her blood alcohol
concentration (BAC) is 172 mg/dL. She is alert and oriented to questions of
person, place, and time and is cogent and appropriate. She is able to ambulate
without difficulty or distress. When is it appropriate to discharge this patient?
No answer selected.

…e.com%2Fcustomscrip%2F154857%3Fscored%263739a18c-0c68-43cc-a4cb-b8b99e9bfd72%3Da45491cd-af54-4004-a990-00ee5e3871ae 19/144
2021/‫‏‬6/‫‏‬22 Immersive Reader

She must be admitted to the hospital for acute alcohol intoxication because her
BAC is 172 mg/dL.

She can be discharged once a friend or family member arrives.

She can be discharged right now.

Her BAC must be recorded as 0 mg/dL before she can be discharged.

She must first undergo a thorough evaluation for concomitant medical


conditions and her chief concern or reason for presentation must be addressed.

Incorrect
Incorrect!

Alcohol intoxication is a clinical diagnosis, so the decision to admit a patient


should be based on clinical findings rather than on BAC. Similarly, it is not
required for a patient’s BAC to be 0 mg/dL prior to discharge. Waiting for this
patient to reach a BAC of 0 mg/dL would take several hours, which would mean
needlessly using a patient bed and potentially risking the development of alcohol
withdrawal symptoms in patients who chronically drink.

The decision to discharge a patient should be considered when the patient can
independently function, is able to dress, walk, and is clinically sober. However,
the patient’s reason for presentation should be addressed and an evaluation for
concomitant medical conditions should be pursued. Ideally, someone will take
responsibility for the patient after discharge; however, this is not absolutely
necessary.

References:

Finnell J, et al. Alcohol-related disease. In: Marx JA, et al, eds. Rosen's
Emergency Medicine: Concepts and Clinical Practice. 7th ed, 2009.

Yip L. Ethanol. In: Goldfrank L, et al, eds. Goldfrank’s Toxicologic Emergencies.


9th ed, 2011.

19. Incorrect

Question Tools:

A 28-year-old man with chronic knee effusions presents to you with rapid-onset
severe lower leg pain that was preceded by a few days of swelling behind the
knee. He has no risk factors for deep venous thrombosis and has no recent history
of trauma.

…e.com%2Fcustomscrip%2F154857%3Fscored%263739a18c-0c68-43cc-a4cb-b8b99e9bfd72%3Da45491cd-af54-4004-a990-00ee5e3871ae 20/144
2021/‫‏‬6/‫‏‬22 Immersive Reader

Which of the following is the most likely diagnosis?

No answer selected.

ruptured Baker cyst

popliteal artery pseudoaneurysm

compartment syndrome

plantaris tendon rupture

Incorrect

Educational Objective:

Recognize the clinical presentation of a ruptured Baker cyst.

Key Point:

A ruptured Baker cyst usually results from chronic knee effusions due to various
etiologies.

Explanation:

A Baker or popliteal cyst usually forms as the result of recurrent synovitis of the
knee from various causes. Rupture of the cyst with an escape of fluid into the calf
muscle may produce a swollen leg, a painful symptom that mimics infection and
thrombophlebitis. Treatment consists of nonsteroidal anti-inflammatory drugs,
elevation, and repair of the underlying cause of the knee effusion. Plantaris tendon
rupture may present similarly, but bruising of the calf is often present and a
history of popliteal swelling would be absent.

Reference:

Palin DJ. Knee and lower leg. In: Walls R, et al, eds. Rosen's Emergency
Medicine: Concepts and Clinical Practice. 19th ed., 2018:698-722.

20. Incorrect

Question Tools:

A football player presents to you with knee pain and an inability to bear weight
after experiencing a twisting injury to his knee. A Lachman test is positive and

…e.com%2Fcustomscrip%2F154857%3Fscored%263739a18c-0c68-43cc-a4cb-b8b99e9bfd72%3Da45491cd-af54-4004-a990-00ee5e3871ae 21/144
2021/‫‏‬6/‫‏‬22 Immersive Reader

there is valgus (medial) laxity.

Which of the following is the most likely group of damaged structures in this
scenario?

No answer selected.

posterior cruciate ligament, medial collateral ligament, and lateral meniscus

posterior cruciate ligament, medial collateral ligament, and medial meniscus

anterior cruciate ligament, medial collateral ligament, and medial or lateral


meniscus

anterior cruciate ligament, lateral collateral ligament, and lateral meniscus

Incorrect

Educational Objective:

Describe the anatomic structures damaged in a positive Lachman test in the


presence of valgus laxity.

Key Point:

A positive Lachman test indicates a tear of the anterior cruciate ligament, which is
often involved in the "unhappy triad" valgus laxity is associated with injury to the
medical collateral ligament.

Explanation:

A positive Lachman test indicates a tear of the anterior cruciate ligament. Valgus
(medial) laxity signifies an injury to the medial collateral ligament. In addition,
the lateral meniscus commonly is injured, as is the medical meniscus (also called
the "unhappy triad" of the anterior cruciate ligament or medial collateral
ligament).

Reference:

Palin DJ. Knee and lower leg. In: Walls R, et al, eds. Rosen's Emergency
Medicine: Concepts and Clinical Practice. 19th ed., 2018:698-722.

21. Incorrect

Question Tools:
…e.com%2Fcustomscrip%2F154857%3Fscored%263739a18c-0c68-43cc-a4cb-b8b99e9bfd72%3Da45491cd-af54-4004-a990-00ee5e3871ae 22/144
2021/‫‏‬6/‫‏‬22 Immersive Reader

A 19-year-old man injured during a motor vehicle collision presents to you with
knee pain. Paramedics on the scene indicate there was initial gross deformity of
the knee, but this spontaneously resolved during extrication. He has a grossly
unstable knee on examination with a large effusion; dorsalis pedis and posterior
tibial pulses are weakly present.

Which of the following studies is recommended for evaluating the knee injury in
this patient?

No answer selected.

computed tomography (CT) angiogram

arthroscopy

Doppler pulse examination

arteriography of the popliteal artery

Incorrect

Educational Objective:

Identify the recommended workup for suspected knee dislocation in a high-energy


trauma incident.

Key Point:

CT angiogram is the diagnostic study of choice for a suspected knee dislocation in


a high-energy trauma accident.

Explanation:

CT angiography has largely replaced arteriography in patients with high-energy


trauma and suspected knee dislocation and popliteal artery injury, even in those
with normal pedal pulses. Normal pedal pulses may be present in nearly 10% of
patients with an injury to the popliteal artery from knee dislocation.

CT angiography does not require mobilization of the angiography team.

Note the left-sided disruption and extravasation of the popliteal artery in Figure 1.

…e.com%2Fcustomscrip%2F154857%3Fscored%263739a18c-0c68-43cc-a4cb-b8b99e9bfd72%3Da45491cd-af54-4004-a990-00ee5e3871ae 23/144
2021/‫‏‬6/‫‏‬22 Immersive Reader

Figure 1.

If CT angiography is not available, then an arteriogram is an option. Figure 2


demonstrates an intimal injury to the distal popliteal artery just above the
bifurcation. Also note the tibial fracture.

Figure 2.

…e.com%2Fcustomscrip%2F154857%3Fscored%263739a18c-0c68-43cc-a4cb-b8b99e9bfd72%3Da45491cd-af54-4004-a990-00ee5e3871ae 24/144
2021/‫‏‬6/‫‏‬22 Immersive Reader

Reference:

Palin DJ. Knee and lower leg. In: Walls R, et al, eds. Rosen's Emergency
Medicine: Concepts and Clinical Practice. 19th ed., 2018:698-722.

22. Incorrect

Question Tools:

A 56-year-old man presents to you with complaints of flushing, nausea, vomiting,


headache, chest, and abdominal pain after drinking a glass of wine. He was
recently treated with an oral antibiotic for the treatment of diverticulitis. Which of
the following is the most likely antibiotic he was prescribed?
No answer selected.

ciprofloxacin

ceftriaxone

none; antibiotics do not cause this reaction

metronidazole

clindamycin

Incorrect
Incorrect!

This patient is experiencing an ethanol-disulfiram reaction. This results from the


inhibition in the metabolism of alcohol and the accumulation of toxic metabolites
that result in a hypersensitivity-like reaction. Several antibiotics are implicated in
this reaction, including metronidazole, which is a common antibiotic used to treat
diverticulitis. Patients should be warned of this reaction when prescribed
metronidazole. Ciprofloxacin, ceftriaxone, and clindamycin have not been
reported to cause this type of reaction.

References:

Finnell J, et al. In: Alcohol-related disease. In: Marx JA, et al, eds. Rosen's
Emergency Medicine: Concepts and Clinical Practice. 7th ed, 2009.

Yip L. Ethanol. In: Goldfrank L, et al, eds. Goldfrank’s Toxicologic Emergencies.


9th ed, 2011.

23. Incorrect

…e.com%2Fcustomscrip%2F154857%3Fscored%263739a18c-0c68-43cc-a4cb-b8b99e9bfd72%3Da45491cd-af54-4004-a990-00ee5e3871ae 25/144
2021/‫‏‬6/‫‏‬22 Immersive Reader

Question Tools:

A 32-year-old woman presents to you with knee pain after being struck in the
knee. She cannot extend her leg. During triage, x-rays were obtained (see Figure)
and a knee deformity was noted; however, the knee now appears to be normal. A
patellar apprehension test is positive and the consulting orthopedist measures the
Q angle to be 20 degrees.

Figure.

Which of the following is the most appropriate treatment option for this patient?

No answer selected.

immobilization in full extension with follow-up in 2 weeks

compressive wrap to the knee with weight bearing as tolerated

long leg cast for 10 weeks

immediate orthopedic consultation

Incorrect

Educational Objective:

Identify the treatment for a dislocated patella.


…e.com%2Fcustomscrip%2F154857%3Fscored%263739a18c-0c68-43cc-a4cb-b8b99e9bfd72%3Da45491cd-af54-4004-a990-00ee5e3871ae 26/144
2021/‫‏‬6/‫‏‬22 Immersive Reader

Key Point:

Dislocation of the patella requires reduction, followed by rest and immobilization


for at least 2 weeks followed by an orthopedic consultation or primary care
follow-up.

Explanation:

In general, dislocation of the patella is the result of a direct blow to the medial or
anterior surface of the knee, and spontaneous reduction is common. There is a
high rate of recurrent patellar dislocation; therefore, long-term immobilization
helps allow the medial retinaculum to heal. In light of current evidence, some
period of immobilization in extension is advisable after the first dislocation event
by placing the patient in an extension brace with either an orthopedic consultation
or primary care follow-up visit within 2 weeks (2-week follow-up visit for
reassessment is now preferred over the historically recommended prolonged 6
weeks of immobilization), followed by physical therapy or patient-directed home
therapy focused on range of motion and quadriceps strengthening.

The patellar apprehension test can be helpful in patients with a nondisplaced


patella on exam and a history of a possible patellar dislocation followed by
spontaneous reduction. The test consists of gently pushing the patella laterally,
which elicits anxiety and anticipatory reactions in the patient with a history of
patella dislocation.

A normal Q angle is 13.5 ± 4.5°, Values above 18° or below 13.5° indicate an
increased risk of r maltracking of the patella.

…e.com%2Fcustomscrip%2F154857%3Fscored%263739a18c-0c68-43cc-a4cb-b8b99e9bfd72%3Da45491cd-af54-4004-a990-00ee5e3871ae 27/144
2021/‫‏‬6/‫‏‬22 Immersive Reader

Figure: Q- Angle

References:

Jain NP, Khan N, Fithian DC. A treatment algorithm for primary patellar
dislocations. Sports Health. 2011;3(2):170-174.

Nwachukwu BU, So C, Schairer WW, Green DW, Dodwell ER. Surgical versus
conservative management of acute patellar dislocation in children and
adolescents: a systematic review. Knee Surg Sports Traumatol Arthrosc.
2016;24(3):760-767.

Palin DJ. Knee and lower leg. In: Walls R, et al, eds. Rosen's Emergency
Medicine: Concepts and Clinical Practice. 19th ed., 2018:698-722.

24. Incorrect

Question Tools:

A 49-year-old woman presents to you with left knee pain after a ground-level fall
that was triggered when she stumbled on an extension cord. She is complaining of
pain along the medial joint.

On examination, she is able to bear weight and is tender across the medial joint
line. The knee flexes to 90 degrees, and she is neurovascularly intact.

Using the Ottawa knee rules, which of the following is true?

No answer selected.

Radiography is not indicated.

Radiography is indicated because the range of motion is abnormal.

Radiography is indicated because she has tenderness on examination.

Radiography is indicated because of her age.

Incorrect

Educational Objective:

Describe when to use radiographs according to the Ottawa knee rules.

Key Point:
…e.com%2Fcustomscrip%2F154857%3Fscored%263739a18c-0c68-43cc-a4cb-b8b99e9bfd72%3Da45491cd-af54-4004-a990-00ee5e3871ae 28/144
2021/‫‏‬6/‫‏‬22 Immersive Reader

If the patient does not meet the specific criteria according to the Ottawa knee
rules, then there is no need for radiography of the knee.

Explanation:

The Ottawa knee rules state that radiography is necessary only if any 1 of these 5
conditions is present:

Patient age > 55 years


Patient cannot transfer weight from one foot to the next 4 times at the
time of injury and in the emergency department
Patient cannot flex knee 90 degrees
Patellar tenderness is present with no other bone tenderness
Tenderness is present in the fibular head

Initial tests found that the Ottawa knee rules detected 100% of fractures, meaning
that significantly fewer radiographs are necessary.

The Pittsburgh knee rules similarly were found to be 100% sensitive. This set of
rules states that radiography is only necessary if the patient has fallen or sustained
blunt trauma to the knee and if 1 of these 2 conditions is present:

Patient age < 12 years or > 50 years


Patient cannot walk four full weight-bearing steps in the emergency
department

The Pittsburgh decision rule (99% sensitivity, 60% specificity; reduces


unnecessary x-rays by 52%) is more specific than the Ottawa rule (97%
sensitivity; 27% specificity, reduces unnecessary x-rays by 28%), thus resulting in
less false-positive (unnecessary) results on radiography. However, the Ottawa
knee rules are more useful in children because the Pittsburgh knee rules would
require radiography for all children.

References:

Palin DJ. Knee and lower leg. In: Walls R, et al, eds. Rosen's Emergency
Medicine: Concepts and Clinical Practice. 19th ed., 2018:698-722.

Seaberg DC, Jackson R. Clinical decision rule for knee radiographs. Am J Emerg
Med. 1994;12(5):541-543.

…e.com%2Fcustomscrip%2F154857%3Fscored%263739a18c-0c68-43cc-a4cb-b8b99e9bfd72%3Da45491cd-af54-4004-a990-00ee5e3871ae 29/144
2021/‫‏‬6/‫‏‬22 Immersive Reader

Tandeter HB, Shvartzman P. Acute knee injuries: use of decision rules for
selective radiograph ordering. Am Fam Physician. 1999;60(9):2599-2608.

25. Incorrect

Question Tools:

A 15-year-old girl presents to you with bilateral, poorly localized knee pain. She
had just begun basketball training after a period of inactivity and denies any
history of trauma. She indicates that the pain is worse when she climbs stairs. A
patellar compression test produces pain.

This pain is consistent with which of the following conditions?

No answer selected.

Osgood-Schlatter disease

infrapatellar bursitis

plica syndrome

patellofemoral pain syndrome

Incorrect

Educational Objective:

Recognize the clinical presentation of patellofemoral pain syndrome.

Key Point:

Patellofemoral pain syndrome is a very common form of knee pain in young girls
and presents with pain in the setting of prolonged flexion. On physical
examination, compression of the patella against with femur with the knee
extended will elicit pain.

Explanation:

Patellofemoral pain syndrome is most commonly seen in young girls. Treatment


consists of strengthening the vastus medialis oblique muscle through
rehabilitation, nonsteroidal anti-inflammatory drugs, and activity modification.

…e.com%2Fcustomscrip%2F154857%3Fscored%263739a18c-0c68-43cc-a4cb-b8b99e9bfd72%3Da45491cd-af54-4004-a990-00ee5e3871ae 30/144
2021/‫‏‬6/‫‏‬22 Immersive Reader

A patellar compression test is best performed when the patient is in a sitting


position and the legs are dangling downward. While compressing the patella
against the femur, the knee will either be passively or actively extended and
flexed. The test is positive for patellofemoral pain syndrome if the patient
experiences pain in either active or passive knee flexion or extension when the
patella is gently pressed against the femur.

References:

Cutler N; Institute for Integrative Healthcare Studies. Eight tests for anterior knee
pain. Published November 4, 2005. Accessed January 18, 2018.

Palin DJ. Knee and lower leg. In: Walls R, et al, eds. Rosen's Emergency
Medicine: Concepts and Clinical Practice. 19th ed., 2018:698-722.

26. Incorrect

Question Tools:

A 21-year-old woman presents to you after being involved in a motor vehicle


collision. She has an obvious right knee dislocation. After reduction, which of the
following is the most appropriate way to assess her peroneal nerve function?

No answer selected.

dorsiflexion and sensation to the first web space of the foot

plantar flexion and plantar sensation of the foot

plantar flexion and sensation to the lateral foot

dorsiflexion and sensation to the dorsum of the foot

Incorrect

Educational Objective:

Identify peroneal nerve function.

Key Point:

Peroneal nerve function is evaluated by determining sensation of the dorsum of


the foot and by asking the patient dorsiflex the ankle.

…e.com%2Fcustomscrip%2F154857%3Fscored%263739a18c-0c68-43cc-a4cb-b8b99e9bfd72%3Da45491cd-af54-4004-a990-00ee5e3871ae 31/144
2021/‫‏‬6/‫‏‬22 Immersive Reader

Explanation:

Injury to the peroneal nerve is the most common major neurologic problem
associated with knee dislocation; some degree of dysfunction occurs in 20% to
40% of patients, and it is permanent in approximately 80% of these cases. The
peroneal nerve should be evaluated by determining the sensation of the dorsum of
the foot and by asking the patient to dorsiflex the ankle. Complete nerve palsy in
the acute setting has been associated with a poor prognosis for recovery.

Reference:

Palin DJ. Knee and lower leg. In: Walls R, et al, eds. Rosen's Emergency
Medicine: Concepts and Clinical Practice. 19th ed., 2018:698-722.

27. Incorrect

Question Tools:

A 10-year-old boy presents to you after his parents report that he did a “face
plant” into his cereal at breakfast. On arrival, he is confused, lethargic, and
markedly ataxic. His blood alcohol concentration is 80 mg/dL. Given the patient’s
symptoms, what additional laboratory or radiographic study is likely to be
abnormal in this patient?

No answer selected.

finger-stick glucose concentration

no other abnormality is expected

head CT

serum sodium concentration

Incorrect

Educational Objective:

Discuss the hypoglycemic effects of alcohol in patients with low glycogen stores.

Key Point:

Hypoglycemia is a common clinical abnormality related to alcohol ingestion in


children. It occurs frequently following a prolonged fast (e.g., just prior to

…e.com%2Fcustomscrip%2F154857%3Fscored%263739a18c-0c68-43cc-a4cb-b8b99e9bfd72%3Da45491cd-af54-4004-a990-00ee5e3871ae 32/144
2021/‫‏‬6/‫‏‬22 Immersive Reader

breakfast).

Explanation:

In this patient, after correcting his blood sugar, the patient admitted to drinking
from his parent’s liquor cabinet prior to breakfast. Studies have demonstrated a
3.4% incidence of hypoglycemia in children and adolescents in the setting of
recent ethanol consumption. Hypoglycemia is thought to occur secondary to a
shift in the redox potential with a resultant shift of pyruvate away from
gluconeogenesis. Children and adolescents have limited glycogen stores in the
liver secondary to a reduced size.

Head CT, serum creatinine, and serum sodium concentrations are not anticipated
to be abnormal from alcohol consumption alone. Other laboratory findings
anticipated from ethanol-induced hypoglycemia include a positive ethanol
concentration, ketonemia without glycosuria, and mild acidosis.

References:

Yip L. Ethanol. In: Goldfrank L, et al, eds. Goldfrank’s Toxicologic Emergencies.


9th ed, 2011.

Maloney GE. Diabetes Mellitus and Disorders of Glucose Homeostasis. (Chapter


118) In: Walls R, et al. Rosen’s Emergency Medicine: Concepts and Clinical
Practice. 9th ed., 2018: 1533-1547.e3

28. Incorrect

Question Tools:

A 72-year-old patient who chronically abuses alcohol presents with a productive


cough, chest discomfort, and subjective complaints of shortness of breath. During
your evaluation, you note that the patient’s pulse oximetry is 92%. Which of the
following is true regarding this patient’s risk for infection?

No answer selected.

Alcohol is an immunosuppressive drug.

Alcohol confers an immunoprotective effect through a poorly understood


mechanism (possibly due to its germicidal effects).

Streptococcus pneumoniae is a less common organism seen in patients who


abuse alcohol than in patients with pneumonia.

…e.com%2Fcustomscrip%2F154857%3Fscored%263739a18c-0c68-43cc-a4cb-b8b99e9bfd72%3Da45491cd-af54-4004-a990-00ee5e3871ae 33/144
2021/‫‏‬6/‫‏‬22 Immersive Reader

Klebsiella pneumoniae occurs most commonly in patients who abuse alcohol.

The patient is at no greater risk for infection than the general population.

Incorrect
Incorrect!

The patient is at much greater risk for infections than the general population.
Alcohol is a direct immunosuppressive drug that is associated with marked
impairments (both acutely and chronically) in the immune system. The
mobilization of macrophages to the site of infection, the bactericidal activity of
macrophages, generations of antibodies, and other cell-mediated immune
functions are disrupted in patients who abuse alcohol. Neutropenia may be seen in
up to 8% of those who abuse alcohol. Streptococcus pneumoniae is still the most
common organism seen in individuals with pneumonia who also abuse alcohol.
By contrast, Klebsiella pneumoniae, which is classically associated with patients
who abuse alcohol, occurs more frequently in patients undergoing chemotherapy,
organ transplantation, or who have hematologic malignancies than in those who
abuse alcohol. Klebsiella pneumoniae is generally thought to be a nosocomial
rather than a community-acquired infection.

References:

Cook RT. Alcohol abuse, alcoholism, and damage to the immune system—a
review. Alcohol Clin Exp Res. 1998;22:1927.

Finnell J, et al. Alcohol-related disease. In: Marx JA, et al, eds. Rosen's
Emergency Medicine: Concepts and Clinical Practice. 7th ed, 2009.

29. Incorrect

Question Tools:

A long-distance runner presents to you with gradual-onset knee pain, which


usually begins after he has run about 7 miles. He is maximally tender across the
lateral condyle of the femur; otherwise, the joint examination is normal.

Which of the following is the most likely diagnosis?

No answer selected.

lateral meniscal tear

Baker cyst

…e.com%2Fcustomscrip%2F154857%3Fscored%263739a18c-0c68-43cc-a4cb-b8b99e9bfd72%3Da45491cd-af54-4004-a990-00ee5e3871ae 34/144
2021/‫‏‬6/‫‏‬22 Immersive Reader

iliotibial band syndrome

plica syndrome

Incorrect

Educational Objective:

Recognize the clinical presentation of iliotibial band syndrome.

Key Point:

Iliotibial band syndrome is an overuse injury that commonly occurs in runners


and is associated with lateral knee pain.

Explanation:

An overuse syndrome frequently seen in runners, iliotibial band syndrome is


caused by inflammation of the bursa underlying the iliotibial band at the lateral
femoral condyle. Treatment involves modifying activity, doing stretches, taking
anti-inflammatory medications, and performing functional exercises to potentially
alter the neuromuscular dynamics exercised when the patient uses the joint.

Reference:

Palin DJ. Knee and lower leg. In: Walls R, et al, eds. Rosen's Emergency
Medicine: Concepts and Clinical Practice. 19th ed., 2018:698-722.

30. Incorrect

Question Tools:

A 37-year-old pregnant woman presents to you for evaluation. Her blood alcohol
concentration (BAC) is 172 mg/dL. Which of the following clinical
manifestations will she have?

No answer selected.

impaired judgment and impaired coordination

difficulty with gait and balance

unable to determine because the patient’s tolerance is unknown

diminished fine motor control


…e.com%2Fcustomscrip%2F154857%3Fscored%263739a18c-0c68-43cc-a4cb-b8b99e9bfd72%3Da45491cd-af54-4004-a990-00ee5e3871ae 35/144
2021/‫‏‬6/‫‏‬22 Immersive Reader

lethargy and difficulty sitting upright without assistance

Incorrect
Incorrect!

Although many textbooks will list BAC and a corresponding physical sign or
symptom as a manifestation of alcohol intoxication, these should be interpreted
with considerable caution. There is considerable variability in the metabolism of
alcohol from person to person and individual experience, so a patient’s tolerance
level to alcohol may lead to a diverse spectrum of manifestations. This patient is
likely to be manifesting all of these symptoms based on her BAC. However, if
this patient is a habitual drinker and her BAC is typically 300 mg/dL, then she
may be manifesting symptoms of alcohol withdrawal at a level of 172 mg/dL.
Health care professionals should carefully consider BAC and a particular physical
manifestation because intoxication and withdrawal are clinical diagnoses and are
not simply based on BAC.

Reference:

Finnell J, et al. Alcohol-related disease. In: Marx JA, et al, eds. Rosen's
Emergency Medicine: Concepts and Clinical Practice. 7th ed, 2009.

31. Incorrect

Question Tools:

A 2-year-old boy is brought in for elbow pain and lack of elbow use after a fall off
playground equipment onto his outstretched arm. The following x-rays are
obtained (see Figures 1 and 2).

…e.com%2Fcustomscrip%2F154857%3Fscored%263739a18c-0c68-43cc-a4cb-b8b99e9bfd72%3Da45491cd-af54-4004-a990-00ee5e3871ae 36/144
2021/‫‏‬6/‫‏‬22 Immersive Reader

Figure 1.

…e.com%2Fcustomscrip%2F154857%3Fscored%263739a18c-0c68-43cc-a4cb-b8b99e9bfd72%3Da45491cd-af54-4004-a990-00ee5e3871ae 37/144
2021/‫‏‬6/‫‏‬22 Immersive Reader

Figure 2.

Which of the following statements is CORRECT?

No answer selected.

Radial nerve injury is the most common soft-tissue trauma associated with this
injury.

The injury visible on the x-ray should be treated with a posterior splint, with
the elbow flexed as tolerated and the forearm pronated.

The x-rays show 2 abnormalities that both indicate the presence of fracture.
…e.com%2Fcustomscrip%2F154857%3Fscored%263739a18c-0c68-43cc-a4cb-b8b99e9bfd72%3Da45491cd-af54-4004-a990-00ee5e3871ae 38/144
2021/‫‏‬6/‫‏‬22 Immersive Reader

The patient has radial head subluxation.

Incorrect

Educational Objective:

Identify supracondylar fracture on x-ray in a pediatric patient.

Key Point:

Especially in children with greenstick fracture through the supracondylar area, the
inconspicuous posterior fat-pad sign may be the only--and easily missed--
radiographic clue to occult supracondylar fracture.

Explanation:

The lateral x-ray shows both nondisplaced supracondylar fracture and an (easily
overlooked) posterior fat-pad sign. In this x-ray, the fracture itself is plainly
visible, but the educational point is to notice the associated, faint posterior fat-pad
sign. Incidentally, this patient’s lateral x-ray also shows a very faint anterior fat-
pad sign and abnormal anterior humeral line. The anteroposterior x-ray just shows
the supracondylar fracture.

Nondisplaced supracondylar fractures can be splinted in a posterior splint at 90-


degree elbow flexion and with the forearm in a neutral position (thumb pointing
up). Typically, protected range-of-motion exercises are started within 3 weeks.
Nondisplaced supracondylar fractures tend to heal uneventfully, although
associated soft-tissue swelling can lead to neurovascular compromise and
ischemic Volkmann contracture. To prevent this devastating complication, this
fracture requires careful patient and caretaker education (and documentation of
that education) about signs of neurovascular compromise.

In nondisplaced supracondylar fractures, the radial nerve is not at any higher risk
for injury than the other neurovascular structures.

Beware that the posterior fat-pad sign may also indicate the presence of an occult
radial head fracture.

The mnemonic CRITOE helps with memorizing the ossification "schedule" of the
pediatric elbow:

C: Capitellum

R: Radial head

…e.com%2Fcustomscrip%2F154857%3Fscored%263739a18c-0c68-43cc-a4cb-b8b99e9bfd72%3Da45491cd-af54-4004-a990-00ee5e3871ae 39/144
2021/‫‏‬6/‫‏‬22 Immersive Reader

I: Internal (medial) epicondyle

T: Trochlea

O: Olecranon

E: External (lateral) epicondyle

The general rule of thumb for the time of appearance of the ossification centers is
"1-3-5-7-9-11," which are roughly the ages in years (≤ 1 year later in boys) that
the ossification centers appear, corresponding to CRITOE. In the patient in this
case, the capitellum has begun ossification and appears as a round ball in line with
the radial head.

References:

Bookman K. Humerus and elbow. In: Walls R, et al. Rosen’s Emergency


Medicine: Concepts and Clinical Practice. 9th ed., 2018: 530-548.e1.

Smithuis R. Elbow - fractures in children. Accessed March 21, 2018.

32. Incorrect

Question Tools:

A 15-year-old boy presents to you with a lateral patella dislocation (see Figure).

Figure.

You initially attempt to perform a reduction, but this technique is unsuccessful


secondary to pain. You give the patient an analgesic.

After achieving analgesia, the proper technique for reducing the dislocation
includes which of the following procedures?

…e.com%2Fcustomscrip%2F154857%3Fscored%263739a18c-0c68-43cc-a4cb-b8b99e9bfd72%3Da45491cd-af54-4004-a990-00ee5e3871ae 40/144
2021/‫‏‬6/‫‏‬22 Immersive Reader

No answer selected.

downward pressure on the lateral patella and gently flex the knee

downward pressure on the lateral patella and gently extend the knee

internal rotation and extension of the knee

external rotation and flexion of the knee

Incorrect

Educational Objective:

Describe the technique for lateral patellar dislocation.

Key Point:

Closed reduction for lateral patellar dislocation includes gentle, passive extension
of the knee with inferomedial pressure on the patella.

Explanation:

Pressure or force should be anteromedially directed on the lateral patellar margin


to reduce a lateral patella dislocation; simultaneously, gentle extension of the leg
should be attempted. Although it is rare, patellar reduction may be difficult and
this method could fail. If the method fails, then closed reduction often may be
achieved by applying downward pressure to the lateral aspect of the patella. This
creates an external rotational force that unlocks the medial patellar facet.

Reference:

Palin DJ. Knee and lower leg. In: Walls R, et al, eds. Rosen's Emergency
Medicine: Concepts and Clinical Practice. 19th ed., 2018:698-722.

33. Incorrect

Question Tools:

A 42-year-old man presents to you with a tense knee effusion. He recently


suffered a puncture wound to the knee that has since healed. Now the knee is red
and warm and he cannot bear weight.

…e.com%2Fcustomscrip%2F154857%3Fscored%263739a18c-0c68-43cc-a4cb-b8b99e9bfd72%3Da45491cd-af54-4004-a990-00ee5e3871ae 41/144
2021/‫‏‬6/‫‏‬22 Immersive Reader

His vital signs are as follows: pulse, 110 beats/minute; blood pressure, 130/90 mm
Hg; respiratory rate, 16 breaths/minute; temperature, 102.1°F (38.9°C); oxygen
saturation, 96%. You decide to perform arthrocentesis.

Which of the following options describes a proper technique for arthrocentesis of


the knee?

No answer selected.

Knee extended, enter 1 cm medial to patella, direct needle superiorly

Knee flexed 45 degrees, enter 1 cm lateral to patella, direct needle toward the
intercondylar femoral notch

Knee flexed, enter 1 cm superolateral to patella, direct needle posteriorly

Knee either extended or flexed 15-20 degrees, enter 1 cm medial to patella,


direct needle towards the intercondylar femoral notch

Incorrect

Educational Objective:

Discuss the clinical technique for arthrocentesis.

Key Point:

The knee can be entered from various locations around the knee; however,
positioning the knee and directing the needle correctly is important for each
approach. Above, the only approach described correctly is the commonly
recommended superior-medial approach, in which the needle is entered 1 cm
medial to the patella at the superior/medial edge and directed towards the femoral
intercondylar notch. In this approach, the knee can be extended (recommended) or
bent to 15-20 degrees, using a pad placed underneath the knee.

Explanation:

The patient should relax the quadriceps muscle because doing so relaxes the
patella and makes it easier to enter the space between the joint. Thoroughly clean
the skin with an antiseptic and drape it with sterile towels to reduce the risk of

…e.com%2Fcustomscrip%2F154857%3Fscored%263739a18c-0c68-43cc-a4cb-b8b99e9bfd72%3Da45491cd-af54-4004-a990-00ee5e3871ae 42/144
2021/‫‏‬6/‫‏‬22 Immersive Reader

infection. Using lidocaine to anesthetize the skin and the joint capsule before
aspiration can greatly ease procedural pain as well as to help the patient relax.

The most common entry point – the superior-medial site - can be found by
palpating the middle or superior edge of the patella on the medial side of the knee.
The needle should be inserted approximately 1-cm medial to the edge of the
patella and directed at the intercondylar femoral notch.

Multiple other sites around the patella can also be used (see image; most common
alternate favorites are the superior-lateral and inferior-medial approaches).

For a superolateral approach, the knee must be extended (not flexed, as falsely
stated in the above answer choices), after which the suprapatellar pouch is entered
from the lateral aspect of the superior patellar margin and the needle is directed
towards the intercondylar notch (not posteriorly, as falsely stated in the above
answer choices). If the pouch is greatly expanded by large effusions, it may be
easier to obtain fluid using this approach.

Though the knee is usually easy to aspirate, ultrasound may further facilitate the
procedure.

…e.com%2Fcustomscrip%2F154857%3Fscored%263739a18c-0c68-43cc-a4cb-b8b99e9bfd72%3Da45491cd-af54-4004-a990-00ee5e3871ae 43/144
2021/‫‏‬6/‫‏‬22 Immersive Reader

Knee arthrocentesis – superior-medial entry point

References:

Sanford SO. Arthrocentesis (Chapter 53). In: Roberts JR. Roberts and Hedgers’
Clinical Procedures in Emergency Medicine 7th ed., 2019:1295-1339. 1105-
1124.e1

Reichman, Eric F., et al. Arthrocentesis (Chapter 97). Reichman's Emergency


Medicine Procedures, 3e Ed.
2018, https://accessemergencymedicine.mhmedical.com/content.aspx?
bookid=2498&sectionid=201311967.

34. Incorrect

Question Tools:

A patient presents in significant pain with obvious medial elbow dislocation.


Which of the following statements is CORRECT about medial elbow
dislocations?

No answer selected.

The lateral x-ray view is the key to differentiating these dislocations from
posterior elbow dislocation.

Care must be taken not to convert medial dislocation into posterior dislocation
during reduction.

Due to the frequently associated, proximal radioulnar fractures, reduction of


either medial or lateral elbow dislocations usually requires general anesthesia.

Medial and lateral elbow dislocations are usually due to a direct blow to the
elbow rather than a fall on the outstretched hand.

…e.com%2Fcustomscrip%2F154857%3Fscored%263739a18c-0c68-43cc-a4cb-b8b99e9bfd72%3Da45491cd-af54-4004-a990-00ee5e3871ae 44/144
2021/‫‏‬6/‫‏‬22 Immersive Reader

Incorrect

Educational Objective:

Examine the steps for managing posterior elbow dislocation.

Key point:

Do not elicit posterior dislocation by reducing medial elbow dislocation.

Explanation:

Medial and lateral elbow dislocations are produced by a similar mechanism as


posterior dislocations, with a vector of force displacing both the ulna and radius as
a unit either medially or laterally instead of posteriorly.

The anteroposterior view, not the lateral view, is the key to appreciating either
medial or lateral elbow dislocations. They are easily missed if the clinician
anticipates posterior dislocation and therefore focuses on the lateral x-ray to either
verify or rule out the clinical impression.

The method of reduction is similar to that for posterior dislocation, except it is


carried out with the arm remaining in more extension. Indeed, care must be taken
not to convert medial or lateral elbow dislocations into posterior dislocation
during reduction.

Associated injuries, complications, and aftercare are the same as those for
posterior dislocations. The literature does not report higher or lower incidence
rates of injury to the ulnar nerve (which courses below the medial epicondyle)
with any one direction of elbow dislocation.

Reference:

Bookman K. Humerus and elbow. In: Walls R, et al. Rosen’s Emergency


Medicine: Concepts and Clinical Practice. 9th ed., 2018: 530-548.e1.

35. Incorrect

Question Tools:

A Segond fracture (lateral capsular sign) is a small, vertical avulsion injury of the
lateral aspect of the proximal tibia just distal to the tibial plateau (see Figure).

…e.com%2Fcustomscrip%2F154857%3Fscored%263739a18c-0c68-43cc-a4cb-b8b99e9bfd72%3Da45491cd-af54-4004-a990-00ee5e3871ae 45/144
2021/‫‏‬6/‫‏‬22 Immersive Reader

Figure.

Which of the following injuries are assumed to be present when a Segond fracture
is seen on radiography?

No answer selected.

tibial plateau fractures

lateral collateral and posterior cruciate ligament injuries

medial collateral and anterior cruciate ligament (ACL) injuries

lateral collateral and anterior cruciate ligament (ACL) injuries

Incorrect

Educational Objective:

Identify injuries associated with Segond fracture.

…e.com%2Fcustomscrip%2F154857%3Fscored%263739a18c-0c68-43cc-a4cb-b8b99e9bfd72%3Da45491cd-af54-4004-a990-00ee5e3871ae 46/144
2021/‫‏‬6/‫‏‬22 Immersive Reader

Key Point:

Segond fractures are usually accompanied by ACL disruption and anterolateral


rotatory instability.

Explanation:

A Segond fracture usually is a sport-related injury that occurs when a flexed knee
experiences a varus stress (lateral buckling of the knee that involves
overstretching of the lateral knee ligaments and compression of the medial
elements) in combination with excessive internal rotation. The easily missed and
deceptively minor-appearing fracture is associated with extensive internal knee
damage. Avulsion of a portion of the lateral collateral ligament and is considered
a marker of a torn ACL. Surgical repair is required for this type of fracture.

Reference:

Palin DJ. Knee and lower leg. In: Walls R, et al, eds. Rosen's Emergency
Medicine: Concepts and Clinical Practice. 19th ed., 2018:698-722.

36. Incorrect

Question Tools:

A 19-year-old rodeo rider arrives with left elbow pain and decreased motion after
a fall off the rodeo bull (see Figure 1.)

…e.com%2Fcustomscrip%2F154857%3Fscored%263739a18c-0c68-43cc-a4cb-b8b99e9bfd72%3Da45491cd-af54-4004-a990-00ee5e3871ae 47/144
2021/‫‏‬6/‫‏‬22 Immersive Reader

Figure 1.

You obtain an x-ray (see Figure 2).

Figure 2.

What do these findings reveal?

No answer selected.

olecranon fracture
…e.com%2Fcustomscrip%2F154857%3Fscored%263739a18c-0c68-43cc-a4cb-b8b99e9bfd72%3Da45491cd-af54-4004-a990-00ee5e3871ae 48/144
2021/‫‏‬6/‫‏‬22 Immersive Reader

anterior dislocation of the elbow

medial epicondyle fracture

posterior dislocation of the elbow

Incorrect

Educational Objective:

Describe the clinical presentation associated with posterior elbow dislocation.

Key Point:

The classic mechanism of injury for posterior elbow dislocation is a fall on the
outstretched hand or wrist, with the elbow either extended or hyperextended. The
patient will present with the elbow in flexion at ~45 degrees and with prominence
of the olecranon process.

Explanation:

The patient’s x-ray shows posterior elbow dislocation with a slightly displaced,
avulsed fragment of the coronoid process. Compare the Figures to notice how
relatively unimpressive the clinical presentation of a dislocated elbow looks in
comparison with the more dramatic-looking x-ray.

The elbow is the second most commonly dislocated joint (after the shoulder) in
the upper extremity. The classic mechanism of injury for posterior elbow
dislocation is a fall on the outstretched hand or wrist, with the elbow either
extended or hyperextended. The patient will present with the elbow in flexion at
about 45 degrees and with prominence of the olecranon process. Dislocations of
the elbow require considerable forces, so associated fractures of adjacent bony
structures are common.

Immediate reduction of these dislocations is imperative to relieve the frequently


associated neurovascular compromise and to prevent compartment syndrome.
Posterior elbow dislocations can be reduced with a partner by bringing the elbow
into flexion while steady countertraction is applied to the anterior surface of the
distal humerus.

Anterior elbow dislocations are rare and usually the result of a direct blow to the
olecranon with the elbow in a flexed position.

Nursemaids elbow is usually diagnosed clinically.


…e.com%2Fcustomscrip%2F154857%3Fscored%263739a18c-0c68-43cc-a4cb-b8b99e9bfd72%3Da45491cd-af54-4004-a990-00ee5e3871ae 49/144
2021/‫‏‬6/‫‏‬22 Immersive Reader

In children, the same mechanism of injury results in an extension supracondylar


fracture rather than a dislocated elbow.

Reference:

Bookman K. Humerus and elbow. In: Walls R, et al. Rosen’s Emergency


Medicine: Concepts and Clinical Practice. 9th ed., 2018: 530-548.e1.

37. Incorrect

Question Tools:

A 2-year-old girl brought in by her mother presents to you with limping after
returning from a visit with her father. You obtain an x-ray (see Figure).

Figure.

What do these radiographic findings indicate?

No answer selected.

spiral fracture, suggestive of abuse

spiral fracture, likely accidental

normal variant

…e.com%2Fcustomscrip%2F154857%3Fscored%263739a18c-0c68-43cc-a4cb-b8b99e9bfd72%3Da45491cd-af54-4004-a990-00ee5e3871ae 50/144
2021/‫‏‬6/‫‏‬22 Immersive Reader

vessel shadow

Incorrect

Educational Objective:

Recognize spiral fractures based on radiographic findings.

Key Point:

Spiral fractures of the distal tibia in a toddler are usually accidental.

Explanation:

A "toddler fracture" refers to a nondisplaced spiral fracture of the distal tibia


found in a child between 9 months and 3 years of age. The classic distal fracture
is often related to accidental trauma. By contrast, a midshaft fracture is suggestive
of abuse. Obviously, the context of the injury is also important, because a distal
fracture will not always be accidental and a proximal fracture will not always be
secondary to abuse; however, it is helpful to understand this basic evidence-based
rule.

Reference:

Halsey MF, Finzel KC, Carrion WV, Haralabatos SS, Gruber MA, Meinhard BP.
Toddler's fracture: presumptive diagnosis and treatment. J Pediatr Orthop.
2001;21(2):152-156.

Heinrich SD, Mooney JF. Fractures of the shaft of the tibia and fibular. In:
Wilkins KE, Beaty JH, eds. Rockwood and Wilkins' Fractures in Children. 4th ed.,
2006:1033.

38. Incorrect

Question Tools:

A 41-year-old man presents to you after being involved in a motor vehicle


collision. He has an obvious left knee dislocation. After reduction, which of the
following is the most appropriate way to assess his posterior tibial nerve function?

No answer selected.

dorsiflexion and sensation to the first web space of the foot

…e.com%2Fcustomscrip%2F154857%3Fscored%263739a18c-0c68-43cc-a4cb-b8b99e9bfd72%3Da45491cd-af54-4004-a990-00ee5e3871ae 51/144
2021/‫‏‬6/‫‏‬22 Immersive Reader

dorsiflexion and sensation to the dorsum of the foot

plantar flexion and sensation to the lateral foot

plantar flexion and plantar sensation of the foot

Incorrect

Educational Objective:

Identify posterior tibial nerve function.

Key Point:

Integrity of the posterior tibial nerve is assessed by checking for the presence of
absence of plantar sensation.

Explanation:

Injury to the peroneal nerve is the most common major neurologic problem
associated with knee dislocation. Less commonly, the posterior tibial nerve may
also be injured, manifesting as diminished plantar sensation and plantar flexion of
the foot. Complete tibial nerve palsy in the acute setting has been associated with
a poor prognosis for recovery.

Reference:

Palin DJ. Knee and lower leg. In: Walls R, et al, eds. Rosen's Emergency
Medicine: Concepts and Clinical Practice. 19th ed., 2018:698-722.

39. Incorrect

Question Tools:

Which of the below statements is CORRECT about olecranon fractures?

No answer selected.

Radial nerve contusion or laceration is the most commonly associated injury.

Loss of sensation over the palmar aspect of the fifth digit and hypothenar
eminence or motor weakness in the interossei muscles of the hand suggests
median nerve injury.

Small, posterior olecranon chip fractures in an otherwise intact, stable elbow do

…e.com%2Fcustomscrip%2F154857%3Fscored%263739a18c-0c68-43cc-a4cb-b8b99e9bfd72%3Da45491cd-af54-4004-a990-00ee5e3871ae 52/144
2021/‫‏‬6/‫‏‬22 Immersive Reader

not require splinting.

The amount of bony displacement of fragments should be noted on x-ray with


the elbow in 90 degrees of flexion.

Incorrect

Educational Objective:

Discuss when to surgically manage an olecranon fracture.

Key Point:

The amount of displacement of fragments should be noted on radiographs with


the elbow in 90-degree flexion.

Explanation:

All olecranon fractures are significant because the triceps attaches to the tip of the
olecranon. Even trivial seeming olecranon fractures can lead to malfunction of the
triceps mechanism, which can result in significant disability. Regarding olecranon
fractures, the amount of bony displacement of fragments should be noted on x-ray
with the elbow in 90 degrees of flexion.

A 2-mm displacement of olecranon fracture on x-ray is a sign easily missed for


triceps instability, which is an indication for surgery. An olecranon fracture that
widens when the elbow is moved from extension to flexion is also defined as a
displaced fracture and is another indication for surgery.

An olecranon fracture associated with an inability to extend the elbow against


force is another sign of possible damage to the triceps mechanism.

The ulnar nerve is the most commonly associated injured structure (injured in
10% of olecranon fractures). Loss of sensation over the palmar aspect of the fifth
digit and hypothenar eminence or motor weakness in the interossei muscles of the
hand suggests ulnar nerve injury.

Undisplaced olecranon fractures can be conservatively treated on an outpatient


basis with ice, analgesia, and immobilization in 45 to 90 degrees of flexion.
Splinting is important to protect the triceps mechanism from accidentally avulsing
during the healing process.

Olecranon fractures occur more commonly in adults than children.

Reference:
…e.com%2Fcustomscrip%2F154857%3Fscored%263739a18c-0c68-43cc-a4cb-b8b99e9bfd72%3Da45491cd-af54-4004-a990-00ee5e3871ae 53/144
2021/‫‏‬6/‫‏‬22 Immersive Reader

Bookman K. Humerus and elbow. In: Walls R, et al. Rosen’s Emergency


Medicine: Concepts and Clinical Practice. 9th ed., 2018: 530-548.e1.

40. Incorrect

Question Tools:

Which of the following statements regarding acromioclavicular (AC) joint


injuries is correct?

No answer selected.

For all AC injuries, routine trauma shoulder x-ray series and an additional
scapular Y-view should be ordered.

On x-ray, the gap between the acromion and clavicle is used to grade the degree
of AC separation.

In a grade 2 AC injury, the coracoclavicular distance will be greater on the


injured than the uninjured side.

The use of weights to differentiate grade 1, 2, and 3 injuries on x-ray is no


longer recommended.

Incorrect

Educational objective:

List the appropriate steps in the clinical management of acromioclavicular joint


injury.

Key Point:

Stress views with weights held or suspended from the wrists have been shown to
be noncontributory in the diagnosis of AC injuries and are no longer
recommended.

Explanation:

Injury to the AC joint results from a fall on or a blow to the tip of the shoulder.
The force of the injury drives the scapula downward and medially to produce the
injury. This is an injury that separates the scapula from the clavicle. The weak AC
ligaments rupture first. With increasing force, the coracoclavicular ligament

…e.com%2Fcustomscrip%2F154857%3Fscored%263739a18c-0c68-43cc-a4cb-b8b99e9bfd72%3Da45491cd-af54-4004-a990-00ee5e3871ae 54/144
2021/‫‏‬6/‫‏‬22 Immersive Reader

ruptures, and the attachments of the deltoid and trapezius muscles are torn from
the distal clavicle.

Care must be taken to order the correct x-rays because normal shoulder x-rays
overpenetrate the AC joint. X-rays of the AC joint should use one-third to two-
thirds less intensity than that required for regular shoulder x-rays. In addition, the
anteroposterior view should include both the right and left AC joint on a single,
wide film. An axillary lateral view and a 15-degree cephalic tilt view should also
be ordered. The axillary lateral view is useful for identifying associated fractures
and posterior dislocation of the clavicle. Historically, clinicians focused on the
radiographic appearance of the AC joint, but the focus has now shifted to the
coracoclavicular distance on x-ray.

Three types of AC injuries can be distinguished: sprain, subluxation, and


dislocation. AC dislocations are further subdivided into 4 subcategories, leading
to a total system of 6 grades as follows:

1. Sprain (grade 1 injury/first-degree sprain): The patient has a partial


tear of the AC ligament with tenderness over this joint, but AC
separation is not visible on x-ray.
2. Subluxation (grade 2 injury): The AC ligament is completely torn, but
the coracoclavicular ligament is still intact (the deltoid and trapezius
muscle insertions may have minor tears). This results in minor upward
or posterior displacement of the distal clavicle on physical
examination and x-ray. However, the coracoclavicular distance (not
acromioclavicular distance) remains normal on x-ray (range, 11-13
mm) and similar on the injured and uninjured sides.
3. Dislocation (grade 3 injury): There is a more pronounced upward
displacement of the acromion, caused by the upward pull of the
trapezius muscle on the scapula. Gravity displaces the hanging arm
downward. The diagnosis is made by comparing the left and right
sides. A difference of > 5 mm between the right and left
coracoclavicular distance (not AC distance) is considered to be
diagnostic.
4. Grade 4 injury: These are similar to grade 3 injuries in that they also
involve complete disruption of both ligaments and the muscles. In
addition, the clavicle is displaced posteriorly into or through the
trapezius muscle.
5. Grade 5 injury: These are similar to grade 3 injuries in that they also
demonstrate complete disruption of both ligaments and the
attachments of the deltoid and trapezius muscles. In addition, major
separation of the clavicle and the acromion can be seen, which is

…e.com%2Fcustomscrip%2F154857%3Fscored%263739a18c-0c68-43cc-a4cb-b8b99e9bfd72%3Da45491cd-af54-4004-a990-00ee5e3871ae 55/144
2021/‫‏‬6/‫‏‬22 Immersive Reader

caused by the clavicle being displaced superiorly instead of


posteriorly.
6. Grade 6 injury: These are rare and involve a complete disruption of
attachments as described with type 3 and inferior displacement of the
clavicle below the acromion or the coracoid.

Patients with strains and subluxations may be treated with a sling for 1 to 2
weeks. Newer studies have shown that, contrary to earlier opinion, most grade 3
injuries can be reduced and then conservatively managed. Grade 3 injuries should
receive early orthopedic referrals (within 72 hours) to allow for a timely
differentiation between surgical versus nonsurgical management. Grades 4 to 6
require operative repair.

Reference:

Bengtzen R, Daya M. Shoulder (Chapter 46). In: Wall R, et al, eds. Rosen’s
Emergency Medicine: Concepts and Clinical Practice. 19th ed., 2018:549-568.e2.

41. Incorrect

Question Tools:

A 33-year-old man presents to you with left knee pain after skiing. He tells you
that he felt a "pop" when he "caught an edge" and fell forward.

On examination, he has mild effusion.

Which of the following is the best way to evaluate the anterior cruciate ligament
(ACL) for injury in the acute setting?

No answer selected.

Apley test

anterior drawer test

Lachman test

pivot shift test

Incorrect

…e.com%2Fcustomscrip%2F154857%3Fscored%263739a18c-0c68-43cc-a4cb-b8b99e9bfd72%3Da45491cd-af54-4004-a990-00ee5e3871ae 56/144
2021/‫‏‬6/‫‏‬22 Immersive Reader

Educational Objective:

Examine the appropriate clinical test for evaluating an anterior cruciate ligament
injury.

Key Point:

The Lachman test is considered the best clinical test to evaluate for an injury to
the ACL.

Explanation:

The Lachman, anterior drawer, and pivot shift tests assess the stability of the
ACL; by contrast, the posterior drawer examines the integrity of the posterior
cruciate ligament. The Lachman test is the only ACL stability test that should be
used in the acute setting. One study found the Lachman test was 99% accurate
compared with 70% for the anterior drawer test in the acute setting.

The anterior drawer test is more useful for the evaluation of chronic injuries,
because joint effusion and hamstring spasm may cause a number of false-negative
results in the acute setting. The pivot shift test is very specific for an injury of the
ACL, but it is not very sensitive. In addition, it is possible to worsen ligament
tears; therefore, this test is generally not recommended in the acute setting.

References:

Palin DJ. Knee and lower leg. In: Walls R, et al, eds. Rosen's Emergency
Medicine: Concepts and Clinical Practice. 19th ed., 2018:698-722.

42. Incorrect

Question Tools:

Elbow dislocation usually results from a fall on the outstretched extremity with
hyperextension and axial compression of the joint.

Which of the following statements about the management of posterior elbow


dislocation is CORRECT?

No answer selected.

Orthopedic consultation is often not necessary to proceed with reduction.

Reduction should never be attempted in the presence of associated fractures.

…e.com%2Fcustomscrip%2F154857%3Fscored%263739a18c-0c68-43cc-a4cb-b8b99e9bfd72%3Da45491cd-af54-4004-a990-00ee5e3871ae 57/144
2021/‫‏‬6/‫‏‬22 Immersive Reader

Regional blocks are more successful in aiding reduction than conscious


sedation.

Posterior dislocations are reduced with an assistant immobilizing the humerus


just above the flexed elbow; the olecranon is pulled/pushed back into position
by slowly extending the elbow.

Incorrect

Educational Objective:

Examine the steps for managing posterior elbow dislocation.

Key Point:

Reduction should be attempted as soon as possible, especially if neurovascular


compromise is present. Orthopedic consultation is usually not necessary to
proceed.

Explanation:

The most serious complication of elbow dislocation is vascular compromise,


which may be present even if distal pulses are intact, thereby making expeditious
reduction of a dislocated elbow a high priority. Orthopedic consultation is usually
not necessary to proceed, although obtaining radiographic evaluation is crucial
before manipulation to rule out elbow fracture that falsely presents as dislocation
on examination. (This is more common in pediatric patients who tend to have a
fracture rather than dislocation at the elbow.)

About 12% to 60% of pre- and postreduction radiographs of elbow dislocations


will also show periarticular fractures. The most common associated fracture is
radial head fracture, which occurs in 10% of elbow dislocations. Operative
findings have revealed unrecognized osteochondral injuries in nearly 100% of
acute elbow dislocations. Closed reduction of elbow dislocations due to a fall onto
the outstretched hand can often be carried out despite the presence of these small,
associated fractures; most of these associated fractures do not require special care.
The same is not true of high-energy fracture-dislocations of the elbow and those
with significant amounts of soft-tissue damage; these combined
fracture/dislocations of the elbow typically need open reduction and repair in the
operating room.

Intra-articular injection of the joint or conscious sedation are the 2 most helpful
approaches to closed reduction in the emergency department; regional blocks or
general anesthesia is rarely needed or helpful.
…e.com%2Fcustomscrip%2F154857%3Fscored%263739a18c-0c68-43cc-a4cb-b8b99e9bfd72%3Da45491cd-af54-4004-a990-00ee5e3871ae 58/144
2021/‫‏‬6/‫‏‬22 Immersive Reader

Posterior dislocations require 2 people for reduction: the assistant immobilizes the
humerus and provides countertraction while the treating physician applies traction
to the partially flexed, distal forearm. During the reduction attempt, the elbow
should ideally be flexed to 30 degrees and the forearm supinated. After reduction,
the joint should be gently moved through its normal range of motion to check
stability. If it is stable, the elbow is flexed to approximately 90 degrees (or as
much as circulation allows) and immobilized in a posterior splint (not a circular
splint).

Postreduction radiographs should be obtained to look for concomitant fractures of


the coronoid process or radial head or, in children, separation of the medial
epicondylar apophysis. Many fractures only become visible after reduction.

Obviously, the neurovascular status should be rechecked after reduction and an


emergent orthopedic consultation should be initiated if continued or new
neurovascular compromise is present. The median nerve, which courses together
with the brachial artery, can become trapped between the olecranon and the distal
end of the humerus during reduction, leading to a new emergency after reduction
has been accomplished.

Gentle range-of-motion exercises may be initiated in 3 to 5 days in most patients.


Rarely, elbow joints that remain unstable may need either prolonged
immobilization or internal fixation (most commonly for instability due to
concomitant fracture).

References:

Bookman K. Humerus and elbow. In: Walls R, et al. Rosen’s Emergency


Medicine: Concepts and Clinical Practice. 9th ed., 2018: 530-548.e1.

Kuhn MA. Acute elbow dislocations. Orthop Clin North Am. 2008;39(2):155-
161, v.

Miller M, Thompson S. DeLee and Drez's Orthopaedic Sports Medicine. 4th ed.,
2014.

43. Incorrect

Question Tools:

A 20-year-old man presents to you complaining of right shoulder pain after being
driven into the boards during an ice hockey match. He is holding his arm close to
the body and there is tenderness to palpation over the lateral aspect of the

…e.com%2Fcustomscrip%2F154857%3Fscored%263739a18c-0c68-43cc-a4cb-b8b99e9bfd72%3Da45491cd-af54-4004-a990-00ee5e3871ae 59/144
2021/‫‏‬6/‫‏‬22 Immersive Reader

shoulder. The lateral tip of the clavicle is prominent. What is the most likely
diagnosis?

No answer selected.

acromioclavicular dislocation

inferior glenohumeral dislocation

scapulothoracic joint dissociation

posterior glenohumeral dislocation

Incorrect

Educational Objective:

Recognize the clinical presentation of acromioclavicular dislocation.

Key Point:

The mechanism of injury for acromioclavicular dislocation involves a fall or blow


to the point of the shoulder with the arm adducted.

Explanation:

This clinical presentation is suggestive of a grade 3 acromioclavicular joint


dislocation. Grade 3 injuries are characterized by complete disruption of the
acromioclavicular and coracoclavicular ligaments as well as attachments of the
deltoid and trapezius muscles. Radiographic features of a grade 3 injury include a
widened joint space and an increased coracoclavicular distance (normal range,
1.1-1.3 cm).

In most patients, conservative management of grade 3 injuries provides equal or


better functional results than surgical correction. For younger athletic patients, or
patients who do manual labor using arms over the head, operative repair may
occasionally be indicated for grade III injuries.

Grade 3 injuries should receive early orthopedic referrals (within 72 hours) to


allow for a timely differentiation between surgical versus nonsurgical
management.

Reference:

…e.com%2Fcustomscrip%2F154857%3Fscored%263739a18c-0c68-43cc-a4cb-b8b99e9bfd72%3Da45491cd-af54-4004-a990-00ee5e3871ae 60/144
2021/‫‏‬6/‫‏‬22 Immersive Reader

Bengtzen R, Daya M. Shoulder (Chapter 46). In: Wall R, et al, eds. Rosen’s
Emergency Medicine: Concepts and Clinical Practice. 19th ed., 2018:549-568.e2.

44. Incorrect

Question Tools:

A 27 year-old construction worker returns for re-evaluation of continued pain


after a fall off scaffolding 1 month ago. Today, he attempted to return to work, but
his boss insisted on a re-evaluation after watching him awkwardly executing
manual tasks.

What do the follow-up x-rays seen in Figures 1 and 2 of the worker’s humerus
reveal?

…e.com%2Fcustomscrip%2F154857%3Fscored%263739a18c-0c68-43cc-a4cb-b8b99e9bfd72%3Da45491cd-af54-4004-a990-00ee5e3871ae 61/144
2021/‫‏‬6/‫‏‬22 Immersive Reader

Figure 1.

…e.com%2Fcustomscrip%2F154857%3Fscored%263739a18c-0c68-43cc-a4cb-b8b99e9bfd72%3Da45491cd-af54-4004-a990-00ee5e3871ae 62/144
2021/‫‏‬6/‫‏‬22 Immersive Reader

Figure 2.

No answer selected.

shoulder dislocation and nonunion of proximal humeral fracture

radial head dislocation

occult supracondylar fracture

…e.com%2Fcustomscrip%2F154857%3Fscored%263739a18c-0c68-43cc-a4cb-b8b99e9bfd72%3Da45491cd-af54-4004-a990-00ee5e3871ae 63/144
2021/‫‏‬6/‫‏‬22 Immersive Reader

missed, pathologic growth at proximal humeral shaft

Incorrect

Educational Objective:

Identify complications associated with a humerus fracture.

Key Point:

In any humeral shaft fractures, radiographs of both the joint below and above
(elbow and shoulder) should be obtained.

Explanation:

The proximal x-ray shows a healing fracture with the classic interim callus. A
single view of the shoulder can easily look like a shoulder dislocation, even if
none is present, and is therefore inadequate to diagnose a dislocation.

However, the elbow x-ray shows a radial head dislocation (the radial head does
not align properly with the capitellum) that was missed during the initial
presentation.

Reference:

Bookman K. Humerus and elbow. In: Walls R, et al. Rosen’s Emergency


Medicine: Concepts and Clinical Practice. 9th ed., 2018: 530-548.e1.

45. Incorrect

Question Tools:

What is the recurrence rate among patients younger than 30 years of age
following a primary anterior glenohumeral dislocation?

No answer selected.

20%-39%

80%-100%

40%-59%

60%-79%

…e.com%2Fcustomscrip%2F154857%3Fscored%263739a18c-0c68-43cc-a4cb-b8b99e9bfd72%3Da45491cd-af54-4004-a990-00ee5e3871ae 64/144
2021/‫‏‬6/‫‏‬22 Immersive Reader

Incorrect

Educational objective:

Identify the recurrence rate of primary anterior glenohumeral dislocations.

Key point:

Recurrence is a common issue following primary anterior glenohumeral


dislocations in patients aged < 30 years.

Explanation:

The recurrence rate among patients younger than 30 years of age following a
primary anterior glenohumeral dislocation is between 80% and 100%.
Predisposing factors include the presence of a Hill-Sachs deformity
(posterolateral compression fracture of the humeral head), glenoid rim fracture,
and an anteroinferior capsulolabral avulsion as a result of the primary dislocation.
The latter is believed to be the primary predisposing factor, and early arthroscopic
repair appears to decrease the risk of recurrence. Recurrence rates decline with
increasing age.

Reference:

Bengtzen R, Daya M. Shoulder (Chapter 46). In: Wall R, et al, eds. Rosen’s
Emergency Medicine: Concepts and Clinical Practice. 19th ed., 2018:549-568.e2.

46. Incorrect

Question Tools:

A 37-year-old man presents to you after a severe blow to the right scapula after he
fell at a construction site. He complains of pain to the right scapular area. Series
of x-rays show normal lung fields and a normal shoulder girdle, except for a
comminuted right scapular fracture with a translucency across the acromial
process epiphysis.

Which of the following statements is correct?

No answer selected.

Scapular fractures are associated with pneumothoraces, hemothoraces, or lung


contusions, which may all present in a delayed fashion.

Blunt trauma resulting in fracture of the scapula is associated with an increased

…e.com%2Fcustomscrip%2F154857%3Fscored%263739a18c-0c68-43cc-a4cb-b8b99e9bfd72%3Da45491cd-af54-4004-a990-00ee5e3871ae 65/144
2021/‫‏‬6/‫‏‬22 Immersive Reader

incidence of blunt aortic injury.

A fracture line across the epiphysis of the acromial process is an indication for
surgery.

Fractures of the scapular neck require surgery.

Incorrect

Educational objective:

Describe the complications associated with fractures of the scapula.

Key point:

The most important aspect of scapular fractures is the high incidence (range,
75%-98%) of associated injuries to the ipsilateral lung, chest wall, and shoulder
girdle complex.

Explanation:

Scapular fractures account for 1% of all fractures and occur primarily in men as a
result of motor vehicle collisions, significant falls, and crush injuries. All
pulmonary problems associated with scapular fractures may present in a delayed
fashion.

Rib fractures are the most common injury found in association with scapular
fractures. Many patients who have rib fractures in addition to their scapular
fracture will also have pneumothorax, hemothorax, or lung contusion. Initially
normal findings on pulmonary evaluation should be cautiously interpreted in any
patient with a scapular fracture (similar to this patient), but especially in the
subset of patients who have an associated rib fracture (not present in this patient).

The majority of scapular fractures can be conservatively managed. Scapular


fractures involving the articular surface of the glenoid fossa or those that impinge
on the shoulder mechanism require surgical correction.

The os acromiale (unfused acromial process epiphysis) is present in 3% of the


general population and should not be confused with fracture.

References:

Bengtzen R, Daya M. Shoulder (Chapter 46). In: Wall R, et al, eds. Rosen’s
Emergency Medicine: Concepts and Clinical Practice. 19th ed., 2018:549-568.e2.

…e.com%2Fcustomscrip%2F154857%3Fscored%263739a18c-0c68-43cc-a4cb-b8b99e9bfd72%3Da45491cd-af54-4004-a990-00ee5e3871ae 66/144
2021/‫‏‬6/‫‏‬22 Immersive Reader

Tucek M. Associated injuries of the scapula fractures. Rozhl Chir.


2010;89(5):288-292.

47. Incorrect

Question Tools:

Which of the following is the MOST SERIOUS complication associated with


posterior elbow dislocations?

No answer selected.

Brachial artery injury

ulnar artery injury

Compartment syndrome

ulnar nerve injury

Incorrect

Educational Objective:

Recognize the most serious complication associated with posterior elbow


dislocation.

Key Point:

The most serious (but not the most common) complication associated with
posterior elbow dislocations is vascular compromise due to an associated injury to
the brachial artery.

Explanation:

The most serious (not the most common) complication associated with posterior
elbow dislocations is vascular compromise due to associated injury to the brachial
artery. The reported incidence is 8%; signs of brachial artery injury can include
presence of a bruit/thrill, loss of distal pulses, or signs consistent with ischemia
such as severe pain in the forearm and hand. Emergency angiography and
consultation should be considered if there is suspicion of vascular compromise.
Unrecognized, the vascular compromise can lead to a compartment syndrome
and, ultimately, to Volkman contracture.

…e.com%2Fcustomscrip%2F154857%3Fscored%263739a18c-0c68-43cc-a4cb-b8b99e9bfd72%3Da45491cd-af54-4004-a990-00ee5e3871ae 67/144
2021/‫‏‬6/‫‏‬22 Immersive Reader

The presence of distal pulses does not rule out brachial artery injury.

Reference:

Bookman K. Humerus and elbow. In: Walls R, et al. Rosen’s Emergency


Medicine: Concepts and Clinical Practice. 9th ed., 2018: 530-548.e1.

48. Incorrect

Question Tools:

A 16-year-old boy injured his left elbow while snowboarding. He presents with
obvious left elbow dislocation.

Which of the following is the MOST likely to be associated with this injury?

No answer selected.

Distal radial fracture

Axillary nerve injury

Musculocutaneous nerve injury

Median and/or ulnar nerve injury

Incorrect

Educational Objective:

List the complications associated with posterior elbow dislocation.

Key Point:

The median and ulnar nerves are the most commonly injured structures in elbow
dislocations (radial and musculocutaneous nerve injuries also occurs, but they are
less common).

Explanation:

Because of proximity, specific nerve, arterial, and bony injuries should be


suspected and ruled out with certain orthopedic injury patterns. The median and
ulnar nerves are the most commonly injured structures in elbow dislocations.

…e.com%2Fcustomscrip%2F154857%3Fscored%263739a18c-0c68-43cc-a4cb-b8b99e9bfd72%3Da45491cd-af54-4004-a990-00ee5e3871ae 68/144
2021/‫‏‬6/‫‏‬22 Immersive Reader

Musculocutaneous nerve injury is most commonly seen with forearm injuries


(although it may also occur with elbow injury), whereas axillary nerve injuries are
seen with shoulder dislocations (see Figure).

Figure 1.

A proximal radial head fracture can be seen with elbow dislocation, but distal
radial fractures (at the wrist) are not commonly associated with elbow
dislocations.

References:

Bookman K. Humerus and elbow. In: Walls R, et al. Rosen’s Emergency


Medicine: Concepts and Clinical Practice. 9th ed., 2018: 530-548.e1.

Goldstein RY, Pace JL, Skaggs DL. Medial epicondyle fractures, elbow
dislocations, and transphyseal separations. Pediatr Upper Extremity. 2015;1167-
1202.

Ramalingam S, Mithani SK, Jong-Ryoon B, Ruch DS. Closed median nerve


rupture from elbow trauma. Hand. 2015;10(2):346-348.

Sears BW, Spear LM. Evaluation and management of adult elbow dislocations in
the emergency department. Emerg Med. 2014.

49. Incorrect

Question Tools:

A 14-year-old girl presents with a closed humeral shaft fracture after a fall from a
horse. Which of the following statements is CORRECT regarding humeral shaft
fractures?

No answer selected.

…e.com%2Fcustomscrip%2F154857%3Fscored%263739a18c-0c68-43cc-a4cb-b8b99e9bfd72%3Da45491cd-af54-4004-a990-00ee5e3871ae 69/144
2021/‫‏‬6/‫‏‬22 Immersive Reader

Humeral shaft fractures displaced by > 2 mm should be reduced by closed


reduction to minimize continued radial nerve neuropraxia.

The humerus is a common site of underlying, preexisting pathology only


diagnosed at the time of fracture.

Radial nerve palsy is an indication for surgery.

In humeral shaft fractures, injury to the radial nerve is associated with a 40%
risk of brachial artery trauma.

Incorrect

Educational Objective:

Examine the relationship between malignancy and humerus shaft fractures.

Key Point:

The mid-humeral shaft is indeed a common site for benign tumors, cysts, primary
bone malignancies, and metastatic disease.

Explanation:

It is not uncommon to discover underlying pathology in the humerus while


viewing x-rays for shaft fracture in patients of all ages. Typically, the fracture will
not heal until the underlying pathology has been addressed.

If no underlying pathology is present, then most closed humeral shaft fractures,


even if they are displaced or comminuted, respond well to conservative
management. If the fracture is grossly displaced or comminuted, then the hanging
cast technique sometimes is used. The most common complication, radial nerve
injury, occurs in 20% of humerus fractures. This nerve injury resolves
spontaneously in most patients, although recovery may take several months.

It is important to note that, if a radial nerve palsy developed after manipulation,


then this is an indication for operative exploration and internal fixation of the
fracture. This is because postreduction palsies are highly suggestive of nerve
entrapment that does not resolve on its own. Radial nerve injuries associated with
penetrating trauma or open fractures are also likely to be permanent unless
surgically corrected, so they usually warrant operative exploration.

All patients with humeral shaft fractures should be referred to an orthopedic


surgeon for close follow-up and repeat neurovascular examinations. In particular,

…e.com%2Fcustomscrip%2F154857%3Fscored%263739a18c-0c68-43cc-a4cb-b8b99e9bfd72%3Da45491cd-af54-4004-a990-00ee5e3871ae 70/144
2021/‫‏‬6/‫‏‬22 Immersive Reader

the heavy, hanging cast can cause gradual distraction of the fracture.

Emergent consultation of an orthopedist should be initiated for all patients with


radial nerve compromise, severely displaced or comminuted fractures, open
fractures, or fractures associated with forearm fractures in the same extremity.
Even though fractures in these categories may be managed conservatively, the
decision should be made by a specialist.

Open reduction and internal fixation are commonly needed for the following
humeral shaft fractures:

Open fractures
Multiple injuries that preclude immobilization
Bilateral fractures
Poor reduction
Poor patient compliance
Failure of closed treatment
Irreducible radial nerve entrapment
Fractures through pathologic bone

Injuries to the brachial artery occur rarely and, if clinically suggested,


angiography or other vascular studies should be considered.

Median and ulnar nerve injuries are usually associated with penetrating trauma to
the upper arm, not with closed humeral shaft fractures.

Reference:

Bookman K. Humerus and elbow. In: Walls R, et al. Rosen’s Emergency


Medicine: Concepts and Clinical Practice. 9th ed., 2018: 530-548.e1.

50. Incorrect

Question Tools:

A 15-year-old girl presents to you with left knee pain and 2 weeks of episodes of
her knee "giving way." She does not have a history of trauma, and her
examination is normal except for tenderness over the medial femoral condyle.
You obtain radiography (see Figures 1 and 2).

…e.com%2Fcustomscrip%2F154857%3Fscored%263739a18c-0c68-43cc-a4cb-b8b99e9bfd72%3Da45491cd-af54-4004-a990-00ee5e3871ae 71/144
2021/‫‏‬6/‫‏‬22 Immersive Reader

Figure 1.

Figure 2.

Which of the following is the most likely diagnosis?

No answer selected.

…e.com%2Fcustomscrip%2F154857%3Fscored%263739a18c-0c68-43cc-a4cb-b8b99e9bfd72%3Da45491cd-af54-4004-a990-00ee5e3871ae 72/144
2021/‫‏‬6/‫‏‬22 Immersive Reader

stress fracture

osteosarcoma

osteochondritis dissecans

Osgood Schlatter disease

Incorrect

Educational Objective:

Identify the clinical presentation of osteochondritis dissecans.

Key Point:

Typically, individuals with osteochondritis dissecans of the knee do not have a


history of trauma and may experience swelling, pain, and giving-way episodes.
On radiography, a subcortical lucency can be seen, and an osteochondral fragment
may be seen separated from the underlying bone.

Explanation:

Osteochondritis dissecans is a rare orthopedic disorder occurring mainly in


adolescents that results in the partial or total separation of articular cartilage
segments and subchondral bone from the underlying bone. Its etiology is
unknown. Osteochondritis dissecans can occur at the knee, talus, or other joints.
Typically, individuals with osteochondritis dissecans of the knee do not have a
history of trauma and may experience swelling, pain, and giving-way episodes.
Oftentimes the only physical finding is localized tenderness of the affected
condyle.

In general, routine radiographic views are diagnostic. A subcortical lucency can


be seen, and an osteochondral fragment may be seen separated from the
underlying bone (see Figure 3).

…e.com%2Fcustomscrip%2F154857%3Fscored%263739a18c-0c68-43cc-a4cb-b8b99e9bfd72%3Da45491cd-af54-4004-a990-00ee5e3871ae 73/144
2021/‫‏‬6/‫‏‬22 Immersive Reader

Figure 3.

Patients with suspected osteochondritis dissecans should not bear weight until
they have been seen by an orthopedic specialist.

Reference:

Palin DJ. Knee and lower leg. In: Walls R, et al, eds. Rosen's Emergency
Medicine: Concepts and Clinical Practice. 19th ed., 2018:698-722.

51. Incorrect

Question Tools:

The recommended basic series of radiographs for evaluating traumatic injuries of


the shoulder consists of which of the following 3-view combinations?

No answer selected.

true anteroposterior (AP), apical oblique, transthoracic lateral

true anteroposterior (AP), axillary lateral, trans-scapular lateral

standard anteroposterior (AP), apical oblique, trans-scapular lateral

standard anteroposterior (AP), axillary lateral, transthoracic lateral

…e.com%2Fcustomscrip%2F154857%3Fscored%263739a18c-0c68-43cc-a4cb-b8b99e9bfd72%3Da45491cd-af54-4004-a990-00ee5e3871ae 74/144
2021/‫‏‬6/‫‏‬22 Immersive Reader

Incorrect

Educational objective:

Identify appropriate imaging studies for shoulder trauma.

Key Point:

The recommended 3-view trauma series for the shoulder consists of true AP,
trans-scapular lateral, and axillary lateral views.

Explanation:

The true AP view is preferred over standard AP views because it projects the
glenohumeral joint without any bony overlap. The standard AP views show the
humerus in either internal or external rotation, which exposes either the greater or
less tuberosity, but blurs the glenohumeral joint.

Proper radiologic examination of any orthopedic injury requires at least 2


different views at right angles to one another (orthogonal). For the shoulder, there
are 3 acceptable orthogonal views: the axillary lateral, trans-scapular lateral ("Y
view"), and apical oblique views. Of these 3 views, either the axillary lateral view
(more commonly used) or the trans-scapular lateral (less commonly used because
it is slightly more difficult to interpret by nonradiologists) is recommended for the
shoulder trauma series. The apical oblique view is not typically part of a routine
trauma series.

The transthoracic view is unacceptable because of image overlap.

Reference:

Bengtzen R, Daya M. Shoulder (Chapter 46). In: Wall R, et al, eds. Rosen’s
Emergency Medicine: Concepts and Clinical Practice. 19th ed., 2018:549-568.e2.

52. Incorrect

Question Tools:

The posterior compartment of the upper arm contains which of the following
structures?

No answer selected.

radial nerve

…e.com%2Fcustomscrip%2F154857%3Fscored%263739a18c-0c68-43cc-a4cb-b8b99e9bfd72%3Da45491cd-af54-4004-a990-00ee5e3871ae 75/144
2021/‫‏‬6/‫‏‬22 Immersive Reader

median nerve

ulnar nerve

brachial artery

Incorrect

Educational Objective:

Discuss the posterior compartment of upper arm.

Key Point:

The posterior compartment of the upper arm contains 2 structures: the radial nerve
and the triceps brachii muscle.

Explanation:

The anterior compartment of the upper arm contains 3 muscles: the biceps brachii,
the brachialis, and the coracobrachialis. The anterior compartment also contains
the brachial artery and the median, ulnar, and musculocutaneous nerves (see
Figure). By contrast, the posterior compartment of the upper arm contains 2
structures: the radial nerve and the triceps brachii muscle.

…e.com%2Fcustomscrip%2F154857%3Fscored%263739a18c-0c68-43cc-a4cb-b8b99e9bfd72%3Da45491cd-af54-4004-a990-00ee5e3871ae 76/144
2021/‫‏‬6/‫‏‬22 Immersive Reader

Figure.

References:

Bookman K. Humerus and elbow. In: Walls R, et al. Rosen’s Emergency


Medicine: Concepts and Clinical Practice. 9th ed., 2018: 530-548.e1.

Hacking C, Trajcevska E. Posterior compartment of the arm. Accessed March 21,


2018.

53. Incorrect

Question Tools:

A 37-year-old sedentary woman who just started walking for exercise presents
after slipping on wet leaves and landing on her outstretched hand. She presents
with elbow pain on active flexion, extension, pronation, and supination, but you
note no visible deformity. Neurovascular examination is intact.

You obtain radiography to rule out occult fracture. Which of the following signs is
always pathological and indicative of an occult radial head fracture in an

…e.com%2Fcustomscrip%2F154857%3Fscored%263739a18c-0c68-43cc-a4cb-b8b99e9bfd72%3Da45491cd-af54-4004-a990-00ee5e3871ae 77/144
2021/‫‏‬6/‫‏‬22 Immersive Reader

otherwise normal x-ray?

No answer selected.

posterior fat pad sign

abnormal anterior humeral line

periosteal elevation of the radius

anterior fat pad sign

Incorrect

Educational Objective:

Discuss interpretation of the anterior versus posterior fat pad sign on elbow x-rays
in patients with elbow trauma and a normal x-ray.

Key Point:

A visible posterior fat pad is never normal and therefore a definite sign of
hemarthrosis and likely occult fracture.

Explanation:

The posterior fat around the proximal elbow is normally hidden by the olecranon
and coronoid processes. Hemorrhage forces fat posteriorly out of the olecranon
fossa, making the normally invisible posterior fat pad visible on the lateral x-ray
view.

In the setting of trauma, more than 90% of patients with a posterior fat pad sign
have intra-articular skeletal injury. In adults, the posterior fat pad sign implies an
occult radial head fracture, whereas the exact same fat pad sign usually indicates
supracondylar injury (which may or may not be otherwise occult) in children. On
a true lateral elbow x-ray of a normal elbow, a thin anterior strip of lucency
(anterior fat pad) is visible. In the presence of intra-articular hemorrhage, bulging
of the joint capsule displaces the normally visible anterior fat pad, changing it
from a slim sliver to a "billowing sail" with convex margins (the so-called "sail
sign"). Thus, the anterior fat pad sign may also give away the presence of an
occult fracture; however, the anterior fat pad can also appear enlarged if the x-ray
is slightly rotated (not a true lateral view), rendering the anterior fat pad a less
dependable indicator of occult fracture than the posterior fat pad.

…e.com%2Fcustomscrip%2F154857%3Fscored%263739a18c-0c68-43cc-a4cb-b8b99e9bfd72%3Da45491cd-af54-4004-a990-00ee5e3871ae 78/144
2021/‫‏‬6/‫‏‬22 Immersive Reader

Abnormalities in the anterior humeral line are indeed an important radiographic


sign, but they do not point to radial head fracture. Instead, these abnormalities
may be the only evidence of minimally displaced supracondylar fracture. The
anterior humeral line is a line drawn along the anterior surface of the humerus on
the lateral x-ray view. Normally the line transects the middle third of the
capitellum.

Periosteal elevation is classically associated with osteomyelitis.

Radial head fracture is the most common elbow fracture in adults (i.e., individuals
in whom the physes at the elbow have closed). Supracondylar fractures occur
mostly in children and adolescents with immature skeletons. Displaced or
comminuted fractures of the radial head may require excision of the radial head to
restore elbow mobility.

References:

Bookman K. Humerus and elbow. In: Walls R, et al. Rosen’s Emergency


Medicine: Concepts and Clinical Practice. 9th ed., 2018: 530-548.e1.

Sandstrom CK. Imaging of elbow fractures and dislocations in adults. Published


May 13, 2015. Accessed March 21, 2018.

54. Incorrect

Question Tools:

A 24-year-old patient presents to you after a significant distraction/hyperextension


injury to his shoulder. He is now complaining about numbness and weakness in
his arm.

On examination, he cannot use his deltoid muscle and has numbness along the
lateral side of his arm. You suspect an injury to the brachial plexus.

Which of the following statements is correct about this patient’s injury?

No answer selected.

Brachial plexus injuries present in the peripheral nerve rather than as nerve-
root patterns.

He injured the brachial plexus in the C4 distribution.

He injured the brachial plexus in the C6 distribution.

…e.com%2Fcustomscrip%2F154857%3Fscored%263739a18c-0c68-43cc-a4cb-b8b99e9bfd72%3Da45491cd-af54-4004-a990-00ee5e3871ae 79/144
2021/‫‏‬6/‫‏‬22 Immersive Reader

He injured the brachial plexus in the C5 distribution.

Incorrect

Educational Objective:

Identify sensory and motor distribution of specific brachial plexus nerves.

Key point:

Brachial plexus integrity is tested by evaluating the myotomes and dermatomes of


each nerve root (the same neurovascular examination one would perform when
evaluating patients with neck injuries).

Explanation:

This patient injured the brachial plexus in the C5 distribution. C5 of the brachial
plexus affects the lateral side of the arm.

The point is not necessarily to know the exact nerve root that is injured, but
instead to recognize when an orthopedic injury is presenting in a dermatome
pattern. However, recognizing several specific patterns makes it easier to identify
injuries to the nerve roots.

C2 to C7 nerve roots have the following myotomes and dermatomes shown in the
Table.

Nerve Root SENSORY AREA MUSCLE

C2-4 Trapezius

C5 Lateral arm Deltoid

C6 Lateral forearm and thumb Biceps

C7 Tip of long finger Thumb extensors

C8 Tip of little finger and medial Finger flexors


forearm

T1 Medial arm  Hand interossei

Reference:

…e.com%2Fcustomscrip%2F154857%3Fscored%263739a18c-0c68-43cc-a4cb-b8b99e9bfd72%3Da45491cd-af54-4004-a990-00ee5e3871ae 80/144
2021/‫‏‬6/‫‏‬22 Immersive Reader

Bengtzen R, Daya M. Shoulder (Chapter 46). In: Wall R, et al, eds. Rosen’s


Emergency Medicine: Concepts and Clinical Practice. 19th ed., 2018:549-568.e2.

55. Incorrect

Question Tools:

Following a recent fracture of the left wrist, a 45-year-old truck driver presents
acutely to you with a complaint of stiffness in his left shoulder that is
accompanied by acute pain during the night. He says it prevented him from
sleeping after a particularly long day of driving. The pain localizes over the
deltoid area and is most severe at night.

There is uniform limitation of all active glenohumeral motion and a sense of


mechanical restriction to joint motion on passive testing.

What is the most likely diagnosis?

No answer selected.

rotator cuff tear

adhesive capsulitis ("frozen shoulder")

bicipital tenosynovitis

calcific tendinitis or subacromial bursitis

Incorrect

Educational objective:

Identify the clinical presentation for adhesive capsulitis.

Key point:

Adhesive capsulitis (frozen shoulder) is characterized by an inflammatory


reaction within the capsule and synovium of the glenohumeral joint that leads to
the formation of adhesions within the capsule and inferior axillary fold. The

Explanation:

…e.com%2Fcustomscrip%2F154857%3Fscored%263739a18c-0c68-43cc-a4cb-b8b99e9bfd72%3Da45491cd-af54-4004-a990-00ee5e3871ae 81/144
2021/‫‏‬6/‫‏‬22 Immersive Reader

A frozen shoulder can result from any condition associated with prolonged
dependency or immobility of the arm (eg, recent fracture, mastectomy,
impingement syndrome), and the most appropriate form of therapy is prevention
through the early use of range-of-motion exercises.

Pain between 60 and 120 degrees of abduction is a consistent feature of calcific


tendinitis and subacromial bursitis. A positive Yergason test and pain over the
bicipital groove are characteristic of bicipital tenosynovitis. A Yergason test is
positive if, with the elbow flexed, supination of the forearm against resistance
causes pain in the anterior and inner aspect of the shoulder.

Reference:

Bengtzen R, Daya M. Shoulder (Chapter 46). In: Wall R, et al, eds. Rosen’s
Emergency Medicine: Concepts and Clinical Practice. 19th ed., 2018:549-568.e2.

56. Incorrect

Question Tools:

Anterior dislocations are usually the result of a severe blow from behind to the
olecranon while the elbow is in a flexed position. Which of the following
statements is CORRECT about anterior elbow dislocations?

No answer selected.

Reduction is difficult and is usually performed in the operating room.

Anterior elbow dislocations have a higher incidence rate of vascular impairment


than the more common posterior dislocation.

Soft-tissue trauma is usually less significant with anterior elbow dislocations


than with the other elbow dislocations.

Anterior elbow dislocations usually present with a fully flexed elbow and
supinated forearm.

Incorrect

Educational objective:

Discuss the clinical correlations associated with anterior elbow dislocations.

Key Point:

…e.com%2Fcustomscrip%2F154857%3Fscored%263739a18c-0c68-43cc-a4cb-b8b99e9bfd72%3Da45491cd-af54-4004-a990-00ee5e3871ae 82/144
2021/‫‏‬6/‫‏‬22 Immersive Reader

Anterior elbow dislocations are more likely to result in vascular impairment than
posterior elbow dislocations.

Explanation:

Anterior dislocations are rare, but they are associated with severe soft-tissue
trauma (avulsion of triceps mechanism, vascular injuries) and are frequently open
(in which case they are treated as an open fracture). Anterior elbow dislocations
do indeed have a higher incidence of vascular impairment than posterior
dislocations.

The patient normally presents with a fully extended elbow and the forearm in
supination. Anterior elbow dislocations can often be reduced successfully with
conscious sedation in the emergency department. The procedure itself consists of
distal traction of the wrist and a backward pressure on the forearm while grasping
the distal humerus. A click usually indicates that reduction has been achieved.
Due to the high incidence of vascular injury, emergent orthopedic referral should
be strongly considered in all patients with anterior elbow dislocations, regardless
of initial success in reducing the injury.

References:

Bookman K. Humerus and elbow. In: Walls R, et al. Rosen’s Emergency


Medicine: Concepts and Clinical Practice. 9th ed., 2018: 530-548.e1.

Miller M, Thompson S. DeLee and Drez's Orthopaedic Sports Medicine. 4th ed.,
2014.

57. Incorrect

Question Tools:

Midshaft humeral fractures often result in which of the following neurovascular


complications?

No answer selected.

brachial artery injury

median nerve injury

radial nerve injury

ulnar nerve injury

…e.com%2Fcustomscrip%2F154857%3Fscored%263739a18c-0c68-43cc-a4cb-b8b99e9bfd72%3Da45491cd-af54-4004-a990-00ee5e3871ae 83/144
2021/‫‏‬6/‫‏‬22 Immersive Reader

Incorrect

Educational Objective:

Describe the complications associated with humerus fractures.

Key Point:

With a reported incidence as high as 20%, radial nerve injury is the most common
complication associated with midshaft fractures of the humerus.

Explanation:

Midshaft humeral fractures often result in radial nerve injury. The radial nerve
leaves the axilla and spirals posteriorly around the humerus within the radial
groove before crossing anterior to the lateral epicondyle to reenter the forearm
between the heads of the coracobrachialis and brachialis brachii muscles. In most
instances, this is a benign neurapraxia that resolves spontaneously over several
months. Radial nerve function is best assessed by evaluating the motor function of
the extensor muscles of the fingers and wrist.

The ulnar nerve courses under the medial epicondyle and enters the forearm
between the 2 heads of the flexor carpi ulnaris. The ulnar nerve may be injured
with fractures of the medial epicondyle or the olecranon process. The ulnar nerve
supplies sensation to the palmar aspect of the fifth digit and the hypothenar
eminence and motor function to the intrinsic muscles of the hands.

Reference:

Bookman K. Humerus and elbow. In: Walls R, et al. Rosen’s Emergency


Medicine: Concepts and Clinical Practice. 9th ed., 2018: 530-548.e1.

58. Incorrect

Question Tools:

Orthopedic consultation in the emergency department is imperative for which of


the following injuries?

No answer selected.

fracture of the proximal humeral epiphysis

minimally displaced midclavicular fracture

…e.com%2Fcustomscrip%2F154857%3Fscored%263739a18c-0c68-43cc-a4cb-b8b99e9bfd72%3Da45491cd-af54-4004-a990-00ee5e3871ae 84/144
2021/‫‏‬6/‫‏‬22 Immersive Reader

nondisplaced 2-part fracture of the proximal humerus

type 1 fractures of the lateral clavicle

Incorrect

Educational objective:

Examine shoulder injuries requiring immediate orthopedic consultation.

Key Point:

Fractures of the proximal humerus should be carefully evaluated for involvement


of the hypertrophic zone of the epiphyseal growth plate.

Explanation:

Fractures of the humerus that involve the growth plate can be observed in young
males aged 11 to 17 years, usually after a fall onto the outstretched hand. These
fractures may be serious because of the potential for growth disturbance, even
under the most ideal conditions.

Nondisplaced fractures of the proximal humerus that do not involve the growth
plate will heal within 4 to 6 weeks, even if several fragments are present, because,
in this area of the body, the fragments are held together by a capsule, periosteum,
and surrounding muscles.

Most fractures of the clavicle will heal uneventfully and can be followed-up by a
primary care physician. Type 2 lateral clavicle fractures and severely displaced
midclavicular fractures should be referred for orthopedic follow-up.

Reference:

Bengtzen R, Daya M. Shoulder (Chapter 46). In: Wall R, et al, eds. Rosen’s
Emergency Medicine: Concepts and Clinical Practice. 19th ed., 2018:549-568.e2.

59. Incorrect

Question Tools:

Anterior glenohumeral dislocations are often associated with which of the


following neurovascular complications?

No answer selected.

…e.com%2Fcustomscrip%2F154857%3Fscored%263739a18c-0c68-43cc-a4cb-b8b99e9bfd72%3Da45491cd-af54-4004-a990-00ee5e3871ae 85/144
2021/‫‏‬6/‫‏‬22 Immersive Reader

axillary artery injury

axillary nerve injury

radial nerve injury

brachial plexus injury

Incorrect

Educational Objective:

Identify the complications of glenohumeral dislocations.

Key Point:

Axillary nerve injury is a complication associated with anterior shoulder


dislocations.

Explanation:

Anterior glenohumeral dislocations are often associated with injury to the axillary
nerve. The incidence rate of axillary nerve injury after anterior shoulder
dislocations ranges from 5% to 54%, and the injury is more common in people
older than 50 years of age. Axillary nerve function can be assessed by testing
sensation over the lateral aspect of the shoulder and motor function of the deltoid
and teres minor muscles. Most axillary nerve injuries are neurapraxic, and the
prognosis for recovery of function is good.

Thrombosis of the axillary artery may be caused by luxatio erecta (also known as
inferior glenohumeral dislocation). Brachial plexus injuries are usually the
consequence of scapulothoracic dissociation. Radial nerve injuries are common
complications of midshaft humeral fractures. The suprascapular nerve innervates
the supraspinatus and infraspinatus muscles and can be injured by fractures of the
scapula that extend into the suprascapular notch.

Reference:

Bengtzen R, Daya M. Shoulder (Chapter 46). In: Wall R, et al, eds. Rosen’s
Emergency Medicine: Concepts and Clinical Practice. 19th ed., 2018:549-568.e2.

60. Incorrect

Question Tools:

…e.com%2Fcustomscrip%2F154857%3Fscored%263739a18c-0c68-43cc-a4cb-b8b99e9bfd72%3Da45491cd-af54-4004-a990-00ee5e3871ae 86/144
2021/‫‏‬6/‫‏‬22 Immersive Reader

What is the most common muscle injured in a rotator cuff tear?

No answer selected.

supraspinatus

teres minor

infraspinatus

subscapularis

Incorrect

Educational Objective:

Distinguish common muscles injured in rotator cuff tears.

Key point:

The supraspinatus is the most common muscle injured in a rotator cuff tear.

Explanation:

The most common muscle injured in a rotator cuff tear is the supraspinatus.
Rotator cuff tears are chronic in 90% of cases. Most tears occur near the
attachment of the supraspinatus. Studies of deceased donors have demonstrated a
hypovascular area within the supraspinatus tendon, which might explain the
propensity of rotator cuff tears located in this area (referred to as the "critical
zone").

Initial symptoms include pain that is worse at night. Worsening pain is followed
by increasing weakness of the arm, especially with flexion and abduction.
Physical findings depend on the size and completeness of the tear. Pain is usually
present over the site of rupture (greater tuberosity), and a defect (humeral head)
may be palpable. The drop-arm test is positive with large tears. Rotator cuff tears
can also occur as a result of anterior dislocations in older patients and are often
initially misdiagnosed as axillary nerve injuries.

The rotator cuff includes the tendinous insertions of the following 4 muscles (see
Figures 1 and 2):

1. Teres minor
2. Supraspinatus
3. Infraspinatus
…e.com%2Fcustomscrip%2F154857%3Fscored%263739a18c-0c68-43cc-a4cb-b8b99e9bfd72%3Da45491cd-af54-4004-a990-00ee5e3871ae 87/144
2021/‫‏‬6/‫‏‬22 Immersive Reader

4. Subscapularis

Figure 1.

In Figure 1, the subscapularis "coats" the "inside" surface of the scapula,


"sandwiched" between the scapula and underlying ribs. It is the only muscle of
the rotator cuff that attaches anteriorly to the lesser tuberosity of the humerus. The
fact that it attaches anteriorly to the arm is counterintuitive, because the muscle
itself lies on the posterior side of the body in the back.

Figure 2.

Figure 2 shows the arrangement of all 4 rotator cuff muscles.

…e.com%2Fcustomscrip%2F154857%3Fscored%263739a18c-0c68-43cc-a4cb-b8b99e9bfd72%3Da45491cd-af54-4004-a990-00ee5e3871ae 88/144
2021/‫‏‬6/‫‏‬22 Immersive Reader

The subscapularis is the only muscle that attaches to the superior portion of the
cuff; the other 3 muscles all attach posteriorly (infraspinatus and teres minor) or
anteriorly (subscapularis).

The teres major attaches to the anterior humerus below the rotator cuff, so it is not
a part of the rotator cuff, although the teres minor is (see Figure 3).

Figure 3.

The greater tuberosity is easy to identify in images of the isolated humerus rotated
such that both tuberosities protrude. However, in vivo, the greater tuberosity is not
palpated as a tuberosity; rather, it is simply the palpable, broad, "outside" part of
the humeral head underneath the deltoid muscle. Tenderness at the "outside" top
part of that ball signifies rotator cuff problems (site of supraspinatus insertion).

The drop-arm test is performed by passively abducting the arm to 90 degrees and
asking the patient to hold the arm in this position. The test is positive (implying a
larger rotator cuff tear) if light pressure on the distal forearm or wrist causes the
patient to suddenly drop his or her arm.

References:

Chansky HA, Iannotti JP The vascularity of the rotator cuff. Clin Sports Med.
1991;10(4):807-822.

Simons S, et al. Presentation and diagnosis of rotator cuff tears. Accessed April


21, 2018.

61. Incorrect

Question Tools:

…e.com%2Fcustomscrip%2F154857%3Fscored%263739a18c-0c68-43cc-a4cb-b8b99e9bfd72%3Da45491cd-af54-4004-a990-00ee5e3871ae 89/144
2021/‫‏‬6/‫‏‬22 Immersive Reader

A 24-year-old man with a history of epilepsy presents to you complaining of right


shoulder pain. The shoulder has a flat, squared-off appearance, and his arm is
fixed in adduction and internal rotation. Abduction is limited and external rotation
is completely blocked.

What is the most likely diagnosis?

No answer selected.

subacromial posterior dislocation

subclavicular anterior dislocation

subglenoid posterior dislocation

subcoracoid anterior dislocation

Incorrect

Educational objective:

Characterize the clinical presentation of posterior shoulder dislocation.

Key point:

Posterior should dislocations can occur during convulsive seizures.

Explanation:

This is a classic presentation of a posterior shoulder dislocation. During


convulsive seizures (epileptic or following electrical shock), the stronger internal
rotator muscles overpower the weaker external rotators to produce the injury.
Posterior dislocations are further classified into 3 types (subacromial, subglenoid,
and subspinous) based on the position of the humeral head. The subacromial
variety (present in this patient) accounts for 98% of all posterior dislocations.

Anterior glenohumeral dislocations can also be classified into 4 types


(subcoracoid, subglenoid, subclavicular, and intrathoracic) based on the position
of the humeral head. The most common type is the subcoracoid variety.

Inferior glenohumeral dislocations are rare, and this patient presents classically
with the arm locked straight up (overhead) in 110 to 160 degrees of abduction.
The elbow is flexed and the forearm typically rests on top of the head. Because
inferior dislocations are so rare, they are easily mistaken as an "odd" (because of
the arm resting on the head) anterior dislocation. Classic maneuvers for reducing
…e.com%2Fcustomscrip%2F154857%3Fscored%263739a18c-0c68-43cc-a4cb-b8b99e9bfd72%3Da45491cd-af54-4004-a990-00ee5e3871ae 90/144
2021/‫‏‬6/‫‏‬22 Immersive Reader

an anterior dislocation will fail and put the patient at risk for iatrogenic injury.
After sedation, inferior dislocations are reduced by creating cephalad traction on
the arm while an assistant uses a towel to provide downward counter-traction of
the shoulder. Eighty percent of patients with luxatio erecta (ie, inferior dislocation
of the glenohumeral joint) will have a rotator cuff injury; neurologic injuries
(axillary nerve) are also common.

Reference:

Bengtzen R, Daya M. Shoulder (Chapter 46). In: Wall R, et al, eds. Rosen’s
Emergency Medicine: Concepts and Clinical Practice. 19th ed., 2018:549-568.e2.

62. Incorrect

Question Tools:

The primary functions of the rotator cuff are to hold the humeral head in place and
to actively initiate a full range of arm motions at the shoulder.

Which of the following statements is correct about the presentation, examination,


or management of acute rotator cuff tears?

No answer selected.

Point tenderness is usually present over the acromioclavicular joint.

The subscapularis is essential for the first 30 degrees of shoulder abduction.

A discrepancy between active and passive range of motion is highly suggestive


of a rotator cuff tear.

Rotator cuff tears are not typically visible on ultrasonography.

Incorrect

Educational Objective:

Examine the clinical presentation associated with rotator cuff tears.

Key point:

A discrepancy between active and passive range of motion is indeed highly


suggestive of a rotator cuff tear.

Explanation:
…e.com%2Fcustomscrip%2F154857%3Fscored%263739a18c-0c68-43cc-a4cb-b8b99e9bfd72%3Da45491cd-af54-4004-a990-00ee5e3871ae 91/144
2021/‫‏‬6/‫‏‬22 Immersive Reader

Rotator cuff tears typically involve the dominant arm and occur in men older than
40 years of age who either tried to stop a fall onto the outstretched hand or who
were lifting a heavy object overhead. When examining the rotator cuff, it is
helpful to know that the supraspinatus is essential for the first 30 degrees of
shoulder abduction. The infraspinatus and teres minor act as external rotators,
whereas the subscapularis is an internal rotator. A discrepancy between active and
passive range of motion is indeed highly suggestive of a rotator cuff tear (similar
to a quadriceps tear and knee motion). A patient with a large tear cannot even
initiate shoulder abduction, whereas patients with small tears might be able to
achieve some abduction; However, in such cases, the health care professional
should watch that the patient is not rotating the scapula rather than the
glenohumeral joint to achieve abduction.

Point tenderness over the site of rupture (usually the greater tuberosity) is another
helpful examination finding that is suggestive of a rotator cuff tear. On x-ray,
superior displacement of the humeral head is the hallmark of a complete tear
(seen on the external rotation view of shoulder). A distance of less than 6 mm is
highly suggestive of a complete tear. The normal distance from the superior
aspect of the humerus to the undersurface of the acromion ranges from 7 to 14
mm.

A study found that ultrasonography was capable of detecting 87% of partial and
acute rotator cuff tears, making ultrasonography a reasonable initial diagnostic
modality. If it is needed, follow-up arthroscopy or magnetic resonance imaging
can confirm or refine the diagnosis using ultrasonography.

In patients with acute rotator cuff tears, early surgical repair (before 3 weeks) is
preferred, especially for a young or active person. The poor blood supply to the
rotator cuff (especially to the supraspinatus muscle) might be to blame for the
suboptimal healing with nonoperative management of the injury.

Reference:

Bengtzen R, Daya M. Shoulder (Chapter 46). In: Wall R, et al, eds. Rosen’s
Emergency Medicine: Concepts and Clinical Practice. 19th ed., 2018:549-568.e2.

63. Incorrect

Question Tools:

The plasma bicarbonate value increases by __ in acute respiratory acidosis and by


__ in chronic respiratory acidosis.

…e.com%2Fcustomscrip%2F154857%3Fscored%263739a18c-0c68-43cc-a4cb-b8b99e9bfd72%3Da45491cd-af54-4004-a990-00ee5e3871ae 92/144
2021/‫‏‬6/‫‏‬22 Immersive Reader

No answer selected.

4 mEq/L per 10 mm Hg PCO2 rise; 5 mEq/L per 10 mm Hg PCO2 rise

2 mEq/L per 10 mm Hg PCO2 rise; 5-6 mEq/L per 10 mm Hg PCO2 rise

3 mEq/L per 10 mm Hg PCO2 rise; 9 mEq/L per 10 mm Hg PCO2 rise

1 mEq/L per 10 mm Hg PCO2 rise; 3 mEq/L per 10 mm Hg PCO2 rise

Incorrect

Incorrect!

The duration of respiratory acidosis can be estimated by the degree of


compensation.

Acute respiratory acidosis will result in an increase of bicarbonate of 1 mEq/L per


10 mm Hg PCO2. Chronic respiratory acidosis will result in an increase of
bicarbonate of 3 mEq/L per 10 mm Hg PCO2. Acute respiratory alkalosis will
result in a decrease of bicarbonate of 2 mEq/L per 10 mm Hg PCO2. Chronic
respiratory alkalosis will result in a decrease of bicarbonate of 4 mEq/L per 10
mm Hg PCO2.

References:

Charney AN, Hoffman RS. Fluid, electrolyte, and acid-base disorders. In: Nelson
LS, et al, eds. Goldfrank's Toxicologic Emergencies. 9th ed., 2011.

Strayer RJ. Acid-base disorders. In: Marx JA, et al, eds. Rosen's Emergency
Medicine: Concepts and Clinical Practice. 8th ed., 2014.

64. Incorrect

Question Tools:

A 16-year-old girl presents to you with left leg pain after a skiing injury 24 hours
ago. You obtain radiography, the results of which reveal a tibia-fibula fracture.
After splinting, she continues to complain of severe pain.

Which of the following is the most appropriate next step in this patient’s
evaluation?

No answer selected.

…e.com%2Fcustomscrip%2F154857%3Fscored%263739a18c-0c68-43cc-a4cb-b8b99e9bfd72%3Da45491cd-af54-4004-a990-00ee5e3871ae 93/144
2021/‫‏‬6/‫‏‬22 Immersive Reader

measure compartment pressures

consider drug-seeking behavior

popliteal arteriography

magnetic resonance imaging (MRI) of the lumbar spine

Incorrect

Educational Objective:

Describe the appropriate workup of involving tibia-fibula fractures.

Key Point:

Compartment syndrome is a complication of tibia-fibula fractures.

Explanation:

Tibia-fibula fractures are typically straightforward to diagnose on plain films;


however, there are several important complications involved in their management.
If the patient continues to experience severe pain after splinting, then limb
ischemia, compartment syndrome, and nerve root compression are complications
to consider. In this scenario, compartment syndrome is most likely secondary to
the fact that the patient was injured 24 to 48 hours ago.

Patients with significant tibial shaft fractures should be initially hospitalized so


that they can be observed for signs of compartment syndrome and provided with
adequate pain management.

Reference:

Palin DJ. Knee and lower leg. In: Walls R, et al, eds. Rosen's Emergency
Medicine: Concepts and Clinical Practice. 19th ed., 2018:698-722.

65. Incorrect

Question Tools:

Which of the following is the formula to calculate the anion gap?

No answer selected.

…e.com%2Fcustomscrip%2F154857%3Fscored%263739a18c-0c68-43cc-a4cb-b8b99e9bfd72%3Da45491cd-af54-4004-a990-00ee5e3871ae 94/144
2021/‫‏‬6/‫‏‬22 Immersive Reader

Na + Ca – (Cl + bicarbonate)

K – (Na + bicarb)

(bicarb + Na) – K

Na – (Cl + bicarbonate)

Incorrect

Incorrect!

To calculate the anion gap, subtract bicarbonate and chloride from sodium (all
measured in mEq/L). The normal value is a range and is reported in various
studies to be between 3 and 16 mEq/L. In daily clinical practice, a range of 3 to 11
mEq/L is often used, although these exact numbers can vary. Some equations
include serum [K]; however, K varies by as little as 1 to 2 above and below
normal, meaning that its value will rarely alter the anion gap significantly.

This equation can also be thought of as representing the difference between


measured and unmeasured cations and anions.

To maintain electroneutrality, measured cations and unmeasured cations must


equal measured anions plus unmeasured anions. Rearranging the formula yields:

Na + K + unmeasured cations = Cl + HCO3 + unmeasured anions

Na + K – HCO3 – Cl = unmeasured anions – unmeasured cations

References:

Charney AN, Hoffman RS. Fluid, electrolyte, and acid-base disorders. In: Nelson
LS, et al, eds. Goldfrank's Toxicologic Emergencies. 9th ed., 2011.

Strayer RJ. Acid-base disorders. In: Marx JA, et al, eds. Rosen's Emergency
Medicine: Concepts and Clinical Practice. 8th ed., 2014.

66. Incorrect

Question Tools:

A patient has a bicarbonate concentration greater than 26 mEq/L (24 mEq/L in


infants and small children). This is called __, which could be a primary problem
or compensation for __.

…e.com%2Fcustomscrip%2F154857%3Fscored%263739a18c-0c68-43cc-a4cb-b8b99e9bfd72%3Da45491cd-af54-4004-a990-00ee5e3871ae 95/144
2021/‫‏‬6/‫‏‬22 Immersive Reader

No answer selected.

respiratory alkalosis; metabolic acidosis

metabolic alkalosis; respiratory acidosis

metabolic acidosis; respiratory alkalosis

respiratory acidosis; metabolic alkalosis

Incorrect

Incorrect!

Changes in both PCO2 and bicarbonate values can be either primary or


compensatory. An elevated bicarbonate concentration (alkalosis) will result in an
elevated pH value (alkalemia).

Reference:

Strayer RJ. Acid-base disorders. In: Marx JA, et al, eds. Rosen's Emergency
Medicine: Concepts and Clinical Practice. 8th ed., 2014.

67. Incorrect

Question Tools:

A 15-year-old boy is brought in by his parents for evaluation of a gunshot wound


to the right arm. Per their report, his older brother was shooting squirrels with a
22-gauge handgun when the patient accidentally stepped into the line of fire and
experienced a through and through injury. His parents immediately dressed his
wounds and brought him in for evaluation.

Upon arrival, the patient is awake, alert, and in no acute distress. Vital signs are
normal. Examination of the right arm reveals entrance and exit wounds to the
lateral upper arm, several centimeters proximal to the elbow. There is no active
bleeding, but a small, nonpulsatile hematoma is observed.

Brachial, radial, and ulnar pulses are equal by palpation and also by handheld
Doppler ultrasonography. Arterial pressure index is 1.0. Radiographic findings
confirm no humerus fracture or retained bullet, and ultrasonography with duplex
of the extremity demonstrates an intimal flap of the brachial artery, spanning less
than 5 mm in length.

…e.com%2Fcustomscrip%2F154857%3Fscored%263739a18c-0c68-43cc-a4cb-b8b99e9bfd72%3Da45491cd-af54-4004-a990-00ee5e3871ae 96/144
2021/‫‏‬6/‫‏‬22 Immersive Reader

Which of the following is the most appropriate management of this patient’s


arterial injury?

No answer selected.

inpatient observation with serial examinations

operative exploration

observation as an outpatient

angiography and endovascular stenting

Incorrect

Educational Objective:

Outline criteria for the expectant management of minor vascular injuries.

Key Point:

Minor nonocclusive vascular injuries that have no evidence of bleeding and have
intact distal circulation may be safely monitored on an outpatient basis.

Explanation:

Minor nonocclusive vascular injuries can be safely managed expectantly. Criteria


for outpatient observation include low-velocity missile wounds (eg, from a
handgun vs an assault rifle), intact distal circulation, absence of active
hemorrhage, and minimal arterial wall disruption on imaging. Specifically, intimal
flaps extending less than 5 mm and pseudoaneurysms less than 5 mm in diameter
resolve on their own in 85% of cases. The usual approach is to repeat physical
examinations and potentially imaging with ultrasonography or computed
tomography angiography over the course of 3 months.

Reference:

Raja AS. Peripheral vascular injury. In: Walls R, et al. Rosen’s Emergency
Medicine: Concepts and Clinical Practice. 9th ed., 2018:435-444.

68. Incorrect

Question Tools:

…e.com%2Fcustomscrip%2F154857%3Fscored%263739a18c-0c68-43cc-a4cb-b8b99e9bfd72%3Da45491cd-af54-4004-a990-00ee5e3871ae 97/144
2021/‫‏‬6/‫‏‬22 Immersive Reader

A 63-year-old woman with known long-standing alcohol abuse has attempted to


quit drinking. She appears underweight and malnourished, and she is also sweaty
and has a strong tremor. Her blood pressure is 150/90 mm Hg, and her pulse is
110 bpm. Following the examination she develops convulsions. How should her
convulsions and withdrawal be managed?

No answer selected.

Control the seizure with diazepam and offer hospitalization for medically
assisted withdrawal.

Consider a quick-acting phenytoin to control the seizure and offer


hospitalization for medically assisted withdrawal.

Consider a quick-acting benzodiazepine to prevent further seizures.

Consider a quick-acting benzodiazepine to prevent further seizures and offer


hospitalization for medically assisted withdrawal.

Offer hospitalization for medically assisted withdrawal.

Incorrect

Hospital admission for medically assisted withdrawal should be offered to people


in acute withdrawal and/or those at high risk of developing alcohol withdrawal
seizures or delirium tremens. Importantly, healthcare professionals caring for
people in acute withdrawal should be experienced and skilled in the assessment of
withdrawal symptoms. If withdrawal seizures develop, then quick-acting
benzodiazepines such as lorazepam should be offered to reduce the likelihood of
further seizures. Phenytoin should not be offered to patients with withdrawal
symptoms. Studies show that diazepam has minimally shorter time to onset than
lorazepam (average of 85 seconds versus 91 seconds to seizure termination – see
Gathwala reference), however, lorazepam is preferred over diazepam (or
midazolam) for its significantly longer duration of action (20 minutes versus
several hours). The differences are due to the fact that diazepam is more
lipophilic, thus redistributing itself out of the brain into fat tissues. Lorazepam is
less lipophilic and remains more concentrated within the brain.

References:

National Clinical Guideline Centre for Acute and Chronic Conditions. Alcohol-
Use Disorders. Diagnosis and Clinical Management of Alcohol-Related Physical
Complications. London: National Institute for Health and Clinical Excellence;
2010.
…e.com%2Fcustomscrip%2F154857%3Fscored%263739a18c-0c68-43cc-a4cb-b8b99e9bfd72%3Da45491cd-af54-4004-a990-00ee5e3871ae 98/144
2021/‫‏‬6/‫‏‬22 Immersive Reader

Gathwala G. Intravenous diazepam, midazolam and lorazepam in acute seizure


control. Indian J Pediatr. 2012 Mar;79(3):327-32.

Trinka E. Benzodiazepines used Primarily for Emergency Treatment (Diazepam,


Lorazepam and Midazolam). Chapter 34 in The Treatment of Epilepsy, Third
Edition. 2009. pp.431 - 446

69. Incorrect

Question Tools:

In acute acidosis, the oxyhemoglobin dissociation curve is shifted to the __,


causing oxygen release in the tissues to __.

No answer selected.

left; increase

right; increase

left; remain constant

right; decrease

Incorrect

Incorrect!

Because tissue metabolism results in acid production (carbonic or lactic), this is


an adaptive mechanism to minimize acidemia. The absence of oxygen results in
anaerobic metabolic and acid production, by increasing oxygen delivery,
oxidative metabolism is stimulated with the production of CO2 (excreted through
the lungs), and reducing acid intermediaries that cannot be excreted through the
lungs.

Reference:

Strayer RJ. Acid-base disorders. In: Marx JA, et al, eds. Rosen's Emergency
Medicine: Concepts and Clinical Practice. 8th ed., 2014.

70. Incorrect

Question Tools:

…e.com%2Fcustomscrip%2F154857%3Fscored%263739a18c-0c68-43cc-a4cb-b8b99e9bfd72%3Da45491cd-af54-4004-a990-00ee5e3871ae 99/144
2021/‫‏‬6/‫‏‬22 Immersive Reader

A 42-year-old man is brought in by emergency medical services for evaluation of


blast injuries after an explosion at a factory. The patient is awake and crying out
in pain upon his arrival.

Vital signs are: heart rate 120 beats/minute, blood pressure 90/60 mm Hg,
respiratory rate 20 breaths/minute, and oxygen saturation 95% on room air.
Primary survey reveals an intact airway and equal breath sounds, but you observe
a large wound of the distal left lower extremity with pulsatile bleeding. Direct
pressure fails to control hemorrhage.

Which of the following is the most appropriate NEXT step to control this patient’s
bleeding?

No answer selected.

angiography

application of a tourniquet

blind clamping at the site of hemorrhage

amputation

Incorrect

Educational Objective:

Analyze hemorrhage control methods for peripheral vascular injury.

Key Point:

Tourniquets improve survival rates among patients with severe limb trauma when
direct pressure is unable to control bleeding.

Explanation:

Direct digital pressure is the first step in an attempted control of hemorrhage in


severe limb trauma. When this maneuver fails, the American College of Surgeons
and the Eastern Association for the Surgery of Trauma recommend applying a
tourniquet. Tourniquets are safe and effective for up to 6 hours (the time limit of
warm ischemia for limb salvage) and have been associated with improved
survival rates in patients with arterial bleeding. A tourniquet should remain in
place until definitive surgical management in the operating room.

…e.com%2Fcustomscrip%2F154857%3Fscored%263739a18c-0c68-43cc-a4cb-b8b99e9bfd72%3Da45491cd-af54-4004-a990-00ee5e3871ae 100/144
2021/‫‏‬6/‫‏‬22 Immersive Reader

While clamping of a clearly visualized vessel may control hemorrhage, blind


clamping is never appropriate, because it may damage surrounding structures and
potentially worsen bleeding. Angiography with endovascular stenting is a crucial
first maneuver for proximal vascular injuries not amenable to direct pressure or
control through tourniquet (eg, subclavian artery), but it is inappropriate for a
distal extremity wound with active arterial hemorrhage that can be more rapidly
controlled by a tourniquet. Amputation to control bleeding is a last resort in the
operating room for a clearly devitalized limb.

References:

American College of Surgeons Committee on Trauma. Advanced Life Support


(ATLS) Student Course Manual. 9th ed., 2012.

Fox N, Rajani RR, Bokhari F, et al; Eastern Association for the Surgery of
Trauma. Evaluation and management of penetrating lower extremity arterial
trauma: an Eastern Association for the Surgery of Trauma practice management
guideline. J Trauma Acute Care Surg. 2012;73(5 suppl 4):S315-S320.

Raja AS. Peripheral vascular injury. In: Walls R, et al. Rosen’s Emergency
Medicine: Concepts and Clinical Practice. 9th ed., 2018:435-444.

71. Incorrect

Question Tools:

A 12-year-old boy presents with his parents for evaluation of a left upper
extremity injury inflicted with a BB gun. Per their report, the patient was
accidentally shot in the arm by his younger brother while they were playing
together.

On examination, the patient is awake, alert, and in no acute distress. His vital
signs are normal. Examination of the left upper extremity reveals a single entry
wound on the ventral surface, just proximal to the antecubital fossa. It is not
actively bleeding, and there is no apparent exit wound.

Which of the following methods will be the MOST reliable in detecting vascular
injury in this patient?

No answer selected.

capillary refill

pulse oximetry

…e.com%2Fcustomscrip%2F154857%3Fscored%263739a18c-0c68-43cc-a4cb-b8b99e9bfd72%3Da45491cd-af54-4004-a990-00ee5e3871ae 101/144
2021/‫‏‬6/‫‏‬22 Immersive Reader

palpation of pulses

arterial pressure index

Incorrect

Educational Objective:

Examine appropriate methods for evaluating vascular injury.

Key Point:

Normal pulses, capillary refill, and pulse oximetry cannot exclude vascular injury.
Patients who sustain trauma via a mechanism that places them at risk for arterial
injury should undergo evaluation with an arterial pressure index or handheld
Doppler ultrasonography.

Explanation:

Although they are commonly included in the physical examination of trauma


patients, pulse examinations, capillary refill, and pulse oximetry are unreliable
predictors of arterial injury. With regard to the pulse examination, false-positive
results may occur due to shock, pre-existing vascular disease, operator technique,
or arterial spasm. Meanwhile, distal pulses can persist despite proximal arterial
injury, and false-negative results may arise with transmission of a pulse through a
clot, past an intimal flap, or through collateral circulation.

Capillary refill depends on age, sex, and temperature, and an arbitrary 2-second
cutoff results in a significant false-positive rate among older patients. Delayed
capillary refill by itself is an unreliable predictor of arterial injury. Likewise, pulse
oximetry is an insensitive measure for identifying limb ischemia after trauma, and
it is neither discriminatory nor useful for this purpose.

Handheld Doppler ultrasonography and, in particular, arterial pressure index are


more reliable methods for detecting arterial injury. Arterial injury is suggested by
an absent Doppler signal or by a change in the usual triphasic quality of the
Doppler pulse to a biphasic or monophasic waveform. An arterial pressure index
is calculated by inflating a standard blood pressure cuff proximal to the injury and
recording the handheld Doppler systolic pressure distal to the injury. This
measurement is repeated on the uninjured limb. A ratio less than 0.9 between
these 2 measurements is more than 95% sensitive and specific for vascular injury,
with high positive and negative predictive values.

…e.com%2Fcustomscrip%2F154857%3Fscored%263739a18c-0c68-43cc-a4cb-b8b99e9bfd72%3Da45491cd-af54-4004-a990-00ee5e3871ae 102/144
2021/‫‏‬6/‫‏‬22 Immersive Reader

Arterial injuries do not always immediately bleed in a clinically detectable way.


Some injuries self-seal in certain positions and only become obvious when the
vessel is stretched or pressure is increased.

Reference:

Raja AS. Peripheral vascular injury. In: Walls R, et al. Rosen’s Emergency
Medicine: Concepts and Clinical Practice. 9th ed., 2018:435-444.

72. Incorrect

Question Tools:

A 20-year-old man presents to you after being involved in a motor vehicle


collision. During the event, he was an unrestrained driver. He is unresponsive due
to head injury, but he is otherwise surprisingly stable. Routine chest x-ray is
obtained as part of the trauma package (see Figure).

Figure.

Which of the following statements about this patient’s x-ray is correct?


…e.com%2Fcustomscrip%2F154857%3Fscored%263739a18c-0c68-43cc-a4cb-b8b99e9bfd72%3Da45491cd-af54-4004-a990-00ee5e3871ae 103/144
2021/‫‏‬6/‫‏‬22 Immersive Reader

No answer selected.

The patient’s x-ray shows a left scapular injury.

The patient’s x-ray shows an injury that typically requires surgery.

The patient’s x-ray shows a left-sided, central injury.

The patient’s x-ray shows a right-sided, central injury.

Incorrect

Educational objective:

Employ radiography to identify sternoclavicular dislocation.

Key point:

On x-ray, it can be difficult to distinguish between anterior and posterior


dislocations. The clavicles in this x-ray are asymmetrical, making it possible to
distinguish which side is abnormal.

Explanation:

The patient's x-ray shows a left sternoclavicular dislocation. Anterior dislocations


are more common than posterior sternoclavicular dislocations at a ratio of 9:1.
This injury is relatively benign, whereas posterior dislocations are potentially life-
threatening.

If conscious, the patient with an anterior dislocation of the medial clavicular joint
will present with the injured extremity flexed at the elbow and supported across
the trunk by the opposite arm. Palpation will further support whether the
dislocation is anterior or posterior, and computed tomography can verify the
clinical impression. This patient's repeat clinical examination showed that this
was an anterior rather than posterior sternoclavicular dislocation.

Anterior dislocations are usually the result of indirect anterolateral forces to the
shoulder that initiated a violent backward roll of the shoulder. This applies
pressure to the shoulder girdle and causes the clavicle to pop out of joint at the
sternal junction.

Anterior sternoclavicular dislocations are usually no cause for alarm. They can
often be reduced in the emergency department, although there is no urgency to
reduce them immediately. Despite the application of a clavicular splint, many

…e.com%2Fcustomscrip%2F154857%3Fscored%263739a18c-0c68-43cc-a4cb-b8b99e9bfd72%3Da45491cd-af54-4004-a990-00ee5e3871ae 104/144
2021/‫‏‬6/‫‏‬22 Immersive Reader

anterior sternoclavicular dislocations remain unstable and eventually dislocate


again.

In patients younger than 25 years of age, sternoclavicular dislocations actually are


a Salter type 1 injury, because the medial epiphysis of the clavicle has not yet
fused.

Reference:

Bengtzen R, Daya M. Shoulder (Chapter 46). In: Wall R, et al, eds. Rosen’s


Emergency Medicine: Concepts and Clinical Practice. 19th ed., 2018:549-568.e2.

73. Incorrect

Question Tools:

A 20-year-old man presents to you after a severe crush injury from a military tank
accident. The patient was standing in the open hatch when the tank hit a ditch and
rolled over.

Chest x-ray is obtained (see Figure).

Figure.

Which of the following statements is correct about this patient’s injuries?

…e.com%2Fcustomscrip%2F154857%3Fscored%263739a18c-0c68-43cc-a4cb-b8b99e9bfd72%3Da45491cd-af54-4004-a990-00ee5e3871ae 105/144
2021/‫‏‬6/‫‏‬22 Immersive Reader

No answer selected.

The patient has a right sternoclavicular dislocation, but it is not possible to


discern from these findings whether the dislocation is anterior or posterior.

The patient has a right anterior sternoclavicular dislocation.

The patient has a left anterior sternoclavicular dislocation.

The patient has a left posterior sternoclavicular dislocation.

Incorrect

Educational Objective:

Distinguish the differences between left and right sternoclavicular dislocations.

Key Point:

Posterior sternoclavicular dislocations are life threatening and are less common
than anterior sternoclavicular dislocations.

Explanation:

This patient has sustained multiple injuries, one of which is a right-sided posterior
sternoclavicular dislocation. Challenge yourself to find all injuries on the initial
chest x-ray before reading on.

Findings on the initial chest x-ray reveal massive subcutaneous emphysema, a


cricothyroidotomy tube in place, right sternoclavicular separation, multiple
fractures, and a disarticulation of the lower thoracic spine.

In general, posterior dislocations of the sternoclavicular joint are rare (1:9


compared with anterior sternoclavicular dislocations), but they can be life
threatening. They may mechanically compromise the airway or signify the
presence of underlying, life-threatening tracheal/bronchial injuries.

A dislocation that is present can be discerned radiographically, but whether the


dislocation is anterior or posterior can usually not be stated with certainty from
plain films alone.

Reference:

Bengtzen R, Daya M. Shoulder (Chapter 46). In: Wall R, et al, eds. Rosen’s
Emergency Medicine: Concepts and Clinical Practice. 19th ed., 2018:549-568.e2.

…e.com%2Fcustomscrip%2F154857%3Fscored%263739a18c-0c68-43cc-a4cb-b8b99e9bfd72%3Da45491cd-af54-4004-a990-00ee5e3871ae 106/144
2021/‫‏‬6/‫‏‬22 Immersive Reader

74. Incorrect

Question Tools:

A 32-year-old woman with no past medical history is brought in by emergency


medical services after sustaining a penetrating injury to the right thigh from a
broken bottle during a bar fight.

Upon arrival, she is awake, alert, and yelling in pain. Vital signs are: heart rate
100 beats/minute, blood pressure 125/85 mm Hg, respiratory rate 14
breaths/minute, and oxygen saturation 97% on room air.

Physical examination reveals a deep laceration to the right anterior thigh with a
stable hematoma and no active hemorrhage. Dorsalis pedis pulses are palpable
bilaterally, but the ankle brachial index (ABI) ipsilateral to the injury is 0.8.

Which of the following is the best next step in management of this patient's
condition?

No answer selected.

diagnostic catheter-based angiography

computed tomography angiography (CTA)

inpatient observation with serial examinations

operative exploration

Incorrect

Educational Objective:

Characterize the diagnostic workup of suspected vascular trauma.

Key Point:

CTA is now the mainstay diagnostic test in the workup of possible arterial injury.

Explanation:

Patients who present with soft signs of vascular injury and without obvious
uncontrolled hemorrhage or limb ischemia should undergo diagnostic imaging to
evaluate for arterial trauma. Toward this end, CTA is highly sensitive and specific,

…e.com%2Fcustomscrip%2F154857%3Fscored%263739a18c-0c68-43cc-a4cb-b8b99e9bfd72%3Da45491cd-af54-4004-a990-00ee5e3871ae 107/144
2021/‫‏‬6/‫‏‬22 Immersive Reader

is readily available at most trauma centers, and has largely supplanted catheter-
based angiography as the diagnostic study of choice.

This patient’s abnormal ABI and stable hematoma constitute soft signs of vascular
injury, so operative exploration would be inappropriate in this scenario.
Observation with serial examinations could miss a potentially dangerous injury to
the femoral vessels, so diagnostic CTA is the most appropriate next step.

References:

American College of Surgeons Committee on Trauma. Advanced Life Support


(ATLS) Student Course Manual. 9th ed., 2012.

Fox N, Rajani RR, Bokhari F, et al; Eastern Association for the Surgery of
Trauma. Evaluation and management of penetrating lower extremity arterial
trauma: an Eastern Association for the Surgery of Trauma practice management
guideline. J Trauma Acute Care Surg. 2012;73(5 suppl 4):S315-S320.

Raja AS. Peripheral vascular injury. In: Walls R, et al. Rosen’s Emergency
Medicine: Concepts and Clinical Practice. 9th ed., 2018:435-444.

75. Incorrect

Question Tools:

Which of the following is the most common type of glycogen-storage disease


(GSD) associated with hypoglycemia?

No answer selected.

type 4 GSD

type 1 GSD (von Gierke disease)

type 2 GSD

type 3 GSD

Incorrect

Educational Objective:

Identify glycogen storage disorders or other enzyme deficiency disorders as


causes of recurrent hypoglycemia.

…e.com%2Fcustomscrip%2F154857%3Fscored%263739a18c-0c68-43cc-a4cb-b8b99e9bfd72%3Da45491cd-af54-4004-a990-00ee5e3871ae 108/144
2021/‫‏‬6/‫‏‬22 Immersive Reader

Key point:

The physician should suspect a rare disorder in severe cases of hypoglycemia.

Explanation:

The most common type of GSD associated with hypoglycemia is type 1 GSD.
The disease is associated with G6PD deficiency, in which patients are unable to
release free glucose from G6PD. This  results in hepatomegaly due to glycogen
storage. Individuals with type 1 GSD frequently present with hypoglycemia in
combination with lactic acidosis, hyperuricemia, and hyperlipidemia.

Hypoglycemia is rarely found in other types of GSD.

Reference:

Goldman L, Schafer AI. Goldman's Cecil Medicine. 24th ed., 2012.

76. Incorrect

Question Tools:

A 50-year-old woman with a history of recent gastric bypass surgery for morbid
obesity presents to you with symptoms of hypoglycemia. She tells you that the
symptoms are particularly worse 1 hour after she eats. You diagnose her patient
with dumping syndrome.

What are the nonpharmacologic measures that can be taken to reduce the
symptoms of hypoglycemia in this patient?

No answer selected.

Advise her to consume pectin and guar to increase viscosity and prevent rapid
emptying and absorption.

Explain to her that she must eat larger meals.

She must start consuming milk and milk products.

Tell her that she must increase her fluid intake.

Advise her to increase her intake of simple sugars.

Incorrect

…e.com%2Fcustomscrip%2F154857%3Fscored%263739a18c-0c68-43cc-a4cb-b8b99e9bfd72%3Da45491cd-af54-4004-a990-00ee5e3871ae 109/144
2021/‫‏‬6/‫‏‬22 Immersive Reader

Educational objective:

Describe the management of dumping syndrome.

Key point:

The physician should recognize the appropriate management of hypoglycemia


using nonpharmacologic measures in patients with dumping syndrome.

Explanation:

This patient should be advised to consume pectin and guar to increase viscosity
and prevent rapid emptying and absorption. This is because pectin and guar have
been used to increase the viscosity of intraluminal contents and relieve the
symptoms of rapid emptying and absorption.

Advising the patient to eat larger meals is not appropriate in a patient who has
undergone gastric bypass surgery. Increasing her fluid intake will lead to rapid
gastric emptying, which is a cause of dumping syndrome. Increasing her intake of
simple sugars and milk and milk products will lead to an osmotic shift, thus
worsening the dumping syndrome.

The symptoms are partially due to an increase in vasoactive substances related to


rapid gastric emptying. Nonpharmacologic therapy consist of the following:

Divide caloric intake into 6 small meals


Limit fluid intake with meals
Decrease carbohydrate intake and avoid simple sugars
Increase dietary fiber
Avoid milk and milk products

Octreotide is the most commonly used agent to inhibit the release of insulin and
other vasoactive substances released by gut, and it also works by decreasing
gastric emptying.

References:

Ferri FF. Ferri's Clinical Advisor. 2017.

Goldman L, Schafer AI. Goldman's Cecil Medicine. 24th ed., 2012.

77. Incorrect

Question Tools:

…e.com%2Fcustomscrip%2F154857%3Fscored%263739a18c-0c68-43cc-a4cb-b8b99e9bfd72%3Da45491cd-af54-4004-a990-00ee5e3871ae 110/144
2021/‫‏‬6/‫‏‬22 Immersive Reader

Which of the following is a characteristic feature of Hirata disease?

No answer selected.

decreased insulin levels

high incidence in western countries

initial hyperglycemia followed by hypoglycemia

severe pathologic changes in the pancreas

Incorrect

Educational objective:

Recognize rare causes of hypoglycemia.

Key point:

There are key features that will aid in the diagnosis of insulin autoimmune
syndrome.

Explanation:

Individuals with Hirata disease have are markedly elevated insulin levels (> 100
mIU/mL). After a meal or glucose load, those with the disease may demonstrate
initial hyperglycemia followed by hypoglycemia a few hours later due to the
binding kinetics of endogenous insulin by antibodies. This results in
hyperinsulinemic hypoglycemia with high titers of antibodies to endogenous
insulin in the absence of pathologic abnormalities in the pancreas. It rarely occurs
in persons living in western countries.

No pathologic changes in the pancreas occur in the setting of Hirata disease; by


contract, individuals with the disease will have increased levels of insulin. Hirata
disease is an uncommon cause of hypoglycemia in western countries.

Reference:

Goldman L, Schafer AI. Goldman's Cecil Medicine. 24th ed., 2012.

78. Incorrect

Question Tools:

…e.com%2Fcustomscrip%2F154857%3Fscored%263739a18c-0c68-43cc-a4cb-b8b99e9bfd72%3Da45491cd-af54-4004-a990-00ee5e3871ae 111/144
2021/‫‏‬6/‫‏‬22 Immersive Reader

A 35-year-old nurse with insulin-dependent diabetes is a frequent visitor to the


emergency department. She also sees specialists for different ailments, but she has
not received any definitive diagnosis. Today, she is presenting to you with
sudden-onset severe hypoglycemia.

With her previous history of multiple symptoms with no definitive diagnosis,


Munchausen syndrome has been considered by her previous clinician.

What are the most helpful laboratory findings for diagnosing factitious
hypoglycemia in this scenario?

No answer selected.

increased insulin and increased C-peptide levels

increased insulin and decreased or absent C-peptide level

normal insulin and increased C-peptide levels

normal insulin and normal C-peptide levels

Incorrect

Educational objective:

Define the appropriate methods for diagnosing factitious hypoglycemia.

Key Point:

Factitious hypoglycemia should be considered in different clinical scenarios.

Explanation:

Sudden-onset hypoglycemia with no relation to meals should raise the suspicion


of factitious hypoglycemia in this patient; moreover, she is health care
professional with multiple episodes of symptoms with no definitive diagnosis.
Insulin levels above 100 mIU/mL are rarely seen in patients with insulinoma, so
they should raise the suspicion index for factitious hypoglycemia. Low or
undetectable C-peptide levels in the setting of elevated plasma insulin levels are
suggestive of exogenous insulin use.

Because it is exogenous insulin, C-peptide levels will be very low or absent; thus,
in this case, the most important laboratory findings are the increased insulin level
and the decreased or absent C-peptide level.

…e.com%2Fcustomscrip%2F154857%3Fscored%263739a18c-0c68-43cc-a4cb-b8b99e9bfd72%3Da45491cd-af54-4004-a990-00ee5e3871ae 112/144
2021/‫‏‬6/‫‏‬22 Immersive Reader

Reference:

Goldman L, Schafer AI. Goldman's Cecil Medicine. 24th ed., 2012.

79. Incorrect

Question Tools:

A patient has a bicarbonate concentration of less than 20 mEq/L (or 18 mEq/L in


infants and small children), representing __. This could be a primary problem or
compensation for __.

No answer selected.

respiratory alkalosis; metabolic acidosis

metabolic acidosis; respiratory alkalosis

respiratory acidosis; metabolic alkalosis

metabolic alkalosis; respiratory acidosis

Incorrect

Incorrect!

The blood pH value determines whether the decreased bicarbonate value is


primary or compensatory. With a normal PCO2 value, a decreased bicarbonate
value will result in a decreased pH value (acidemia), as is apparent from the
Henderson-Hasselbalch equation:

pH = pK + log (bicarbonate ÷ carbonic acid) or pH = pK + log (bicarbonate ÷


[0.03 × PCO2])

If the pH value is normal or elevated, then the lowered bicarbonate value (acidotic
process) is an attempt to compensate for primary respiratory alkalosis.

References:

Luther JM, Breyer-Lewis J. Acid-base disorders. In: The Cleveland Clinic


Intensive Review of Internal Medicine. 5th ed., 2009: 598.

Strayer RJ. Acid-base disorders. In: Marx JA, et al, eds. Rosen's Emergency
Medicine: Concepts and Clinical Practice. 8th ed., 2014.

…e.com%2Fcustomscrip%2F154857%3Fscored%263739a18c-0c68-43cc-a4cb-b8b99e9bfd72%3Da45491cd-af54-4004-a990-00ee5e3871ae 113/144
2021/‫‏‬6/‫‏‬22 Immersive Reader

80. Incorrect

Question Tools:

Which of the following is a prominent feature of non–islet-cell tumor


hypoglycemia (also known as insulin-like growth factor 2 [IGF-2]-oma)?

No answer selected.

IGF-2-oma is a neoplasm similar to insulinoma.

IGF-2-oma has a neuroepithelial origin.

IGF-2-oma causes increased plasma levels of insulin.

The cause of hypoglycemia in patients with IGF-2-oma is overproduction of


IGF-2 by the tumor.

Incorrect

Educational objective:

Diagnose IGF-2-oma.

Key point:

The physician should recognize hypoglycemic episodes in the setting of IGF-2-


oma.

Explanation:

IGF-2-oma is a rare paraneoplastic syndrome of hypoglycemia associated with


benign or malignant neoplasms other than insulinoma. The associated tumors are
usually of mesenchymal and hepatic origin. The underlying mechanism of
hypoglycemia is the overproduction of IGF-2 which has potent insulin-like
activity and therefore mimics fasting hypoglycemia produced by insulin-
producing islet-cell tumors. IGF-2-oma should be suspected in any patients with
hypoglycemia of unknown cause. In general, elevated IGF-2 levels can be
observed with suppressed levels of plasma insulin and growth hormone as well as
a low serum glucose.

References:

…e.com%2Fcustomscrip%2F154857%3Fscored%263739a18c-0c68-43cc-a4cb-b8b99e9bfd72%3Da45491cd-af54-4004-a990-00ee5e3871ae 114/144
2021/‫‏‬6/‫‏‬22 Immersive Reader

Bodnar TW, Acevedo MJ, Pietropaolo M. Management of non-islet-cell tumor


hypoglycemia: a clinical review. J Clin Endocrinol Metab. 2014;99(3):713-722.

Goldman L, Schafer AI. Goldman's Cecil Medicine. 24th ed., 2012.

81. Incorrect

Question Tools:

A patient has metabolic alkalosis with an increased bicarbonate level. The


chloride concentration will tend to do which of the following?

No answer selected.

decrease

increase

be determined by chronicity of metabolic alkalosis

remain stable

Incorrect

Incorrect!

The sum of the bicarbonate and chloride values must remain approximately
constant to balance most of the positive ions (primarily sodium). Measured
cations plus unmeasured cations must equal measured anions plus unmeasured
anions to maintain electroneutrality.

Reference:

Strayer RJ. Acid-base disorders. In: Marx JA, et al, eds. Rosen's Emergency
Medicine: Concepts and Clinical Practice. 8th ed., 2014.

82. Incorrect

Question Tools:

Anion-gap acidosis is likely to be present when (1) a metabolic acidosis is present


plus (2) the anion gap exceeds what value?

No answer selected.

…e.com%2Fcustomscrip%2F154857%3Fscored%263739a18c-0c68-43cc-a4cb-b8b99e9bfd72%3Da45491cd-af54-4004-a990-00ee5e3871ae 115/144
2021/‫‏‬6/‫‏‬22 Immersive Reader

3-5 mEq/L

1-3 mEq/L

5-10 mEq/L

12 mEq/L

Incorrect

The anion gap is the difference between the most abundant serum cation(s)
(sodium Na+ +/- potassium K+) and the most abundant anions (chloride Cl– and
bicarbonate HCO3–). The gap between cations and anions is artificially created by
the fact that not all anions and cations in the blood are incorporated into the anion
gap equation.

The incorporated cation(s) typically outnumber the incorporated anions (e.g. more


of the other present anions than cations are ignored); therefore even a healthy
person without a metabolic acidosis will have an anion gap of approximately 3 to
12 mEq/L. Up to 30% of healthy people may even have anion gaps as high as 20
-29 mEq/L. However, for clinical purposes, an anion gap over 12 mEq/L should
draw attention because it usually points to the presence of a metabolic acidosis.

The normal gap has such a wide range because it is affected both by laboratory
method and the person’s albumin level. Albumin makes up the bulk of the missing
anions (with phosphorus making up most of the rest). A high albumin level
increases the person’s normal anion gap (a higher number of non-counted anions
are present). The converse, a low albumin level can lead to such a low baseline
anion gap that an elevated gap is accidentally misinterpreted as normal (eg in very
sick ICU patients).

The clinical utility of the anion gap is to narrow the differential in the patient
with a metabolic acidosis. 

A normal anion gap acidosis (sometimes called nonanion gap acidosis) results
from the absorption or generation of an acid that dissociates into hydrogen and
chlorine, thus leading to acidemia (too many H+ ions), but at the same time
providing the extra chloride needed to avoid upsetting the anion gap equation
(usually seen in acidosis related to GI or renal loss of bicarbonate).

By contrast, the anion gap becomes elevated when the deficit in bicarbonate
anions (which creates the acidosis) is not compensated by chloride (thus not

…e.com%2Fcustomscrip%2F154857%3Fscored%263739a18c-0c68-43cc-a4cb-b8b99e9bfd72%3Da45491cd-af54-4004-a990-00ee5e3871ae 116/144
2021/‫‏‬6/‫‏‬22 Immersive Reader

correcting the increased the anion gap), but instead is balanced by an unmeasured
anion (eg, lactate, ketone anions, sulfate, phosphate).

Pearl: any metabolic acidosis may or may not be compensated by a respiratory


alkalosis, thus leading to a low, normal, or very rarely even slightly high pH in
conjunction with any metabolic acidosis (the common assumption the pH will be
low with an anion gap acidosis is false).

References:

Strayer RJ. Acide-Base Disorders. (Chapter 116) In: Walls R, et al. Rosen’s
Emergency Medicine: Concepts and Clinical Practice. 9th ed., 2018: 1509-
1515.e2

Sparshott A. Anion Gap. Life in the Fast Lane. updated November 9, 2018.


Accessed January 5, 2019.

83. Incorrect

Question Tools:

A conscious patient is brought in by emergency medical services after


experiencing electric shock. Amongst several complaints, the patient has severe
shoulder pain. On examination, the patient is semi-reclined and holding his left
arm tightly across the front of his body.

You obtain an x-ray (see Figure 1).

…e.com%2Fcustomscrip%2F154857%3Fscored%263739a18c-0c68-43cc-a4cb-b8b99e9bfd72%3Da45491cd-af54-4004-a990-00ee5e3871ae 117/144
2021/‫‏‬6/‫‏‬22 Immersive Reader

Figure 1.

Which of the following statements is correct about this patient's injury?

No answer selected.

This is the most commonly missed fracture in the upper extremity.

This injury requires surgery.

Neurologic injury is especially common with this injury.

The patient’s ability to supinate his left wrist should be checked.

Incorrect

Educational Objective:

Recognize radiographic findings indicative of posterior shoulder dislocation.

Key point:

…e.com%2Fcustomscrip%2F154857%3Fscored%263739a18c-0c68-43cc-a4cb-b8b99e9bfd72%3Da45491cd-af54-4004-a990-00ee5e3871ae 118/144
2021/‫‏‬6/‫‏‬22 Immersive Reader

Posterior shoulder dislocations are the most frequently missed dislocation in the
body (~ 50% missed on first presentation).

Explanation:

This patient has a posterior shoulder dislocation. Both the history of electric shock
and the arm held tightly across the body are classic (a postseizure patient or a
patient who braced him/herself during an automobile collision are also at high risk
for having a posterior shoulder dislocation). Posterior shoulder dislocations are
the most frequently missed dislocation in the body (~ 50% missed on first
presentation). Even experienced emergency physicians can miss these, especially
if the dislocation is accompanied by a shoulder fracture that seems to explain the
shoulder pain and decreased range of motion.

Approximately 50% of posterior shoulder dislocations are accompanied by


fracture. As a rule, a posterior shoulder dislocation is more painful than anterior
dislocation, but both neurologic or vascular injuries are less common than in
anterior dislocations. On examination, external rotation will be completely
blocked and abduction will be severely limited. A subtle but extremely reliable
sign is the fact that the patient with a posterior shoulder dislocation will never be
able to supinate his or her forearm (consider documenting the presence or absence
of forearm supination in the chart of every patient presenting with a shoulder
injury).

The anteroposterior x-ray of a posteriorly dislocated shoulder can look completely


normal and can be nondiagnostic. A lateral x-ray is required to confirm the
diagnosis. In this patient, the x-ray is indeed abnormal, but only subtly normal. In
the anteroposterior x-ray of the normal shoulder, the humeral head will produce
an elliptically shaped overlap shadow with the glenoid. The posteriorly dislocated
shoulder will produce an elliptical shadow that looks distorted and irregular (true
of this patient’s x-ray). However, the posteriorly dislocated shoulder may also
produce a "dead-normal" looking x-ray, where there is no humeral-glenoid
overlap shadow. In these cases, the humeral head seems to be perfectly positioned
opposite to the glenoid fossa without touching the fossa (without any elliptical
shadow overlap). Ironically, these x-rays look "more" normal than the x-ray of a
normal shoulder, where some degree of humeral-glenoid overlap will be present.

Posterior shoulder dislocations are more difficult to reduce than anterior ones.
Attempts can be made to reduce posterior dislocations under sedation in the
emergency department, but often general anesthesia will be required.

Figure 2 shows the widened gap, which looks deceptively normal, of a posterior
shoulder dislocation.

…e.com%2Fcustomscrip%2F154857%3Fscored%263739a18c-0c68-43cc-a4cb-b8b99e9bfd72%3Da45491cd-af54-4004-a990-00ee5e3871ae 119/144
2021/‫‏‬6/‫‏‬22 Immersive Reader

Figure 2.

Figure 3.

References:

Bengtzen R, Daya M. Shoulder (Chapter 46). In: Wall R, et al, eds. Rosen’s


Emergency Medicine: Concepts and Clinical Practice. 19th ed., 2018:549-568.e2.

Brady WJ. Challenging and elusive orthopedic injuries: diagnostic and treatment


strategies - part I: upper extremity fractures and dislocations. Published April 26,
1999. Accessed April 21, 2018.

84. Incorrect

…e.com%2Fcustomscrip%2F154857%3Fscored%263739a18c-0c68-43cc-a4cb-b8b99e9bfd72%3Da45491cd-af54-4004-a990-00ee5e3871ae 120/144
2021/‫‏‬6/‫‏‬22 Immersive Reader

Question Tools:

A 35-year-old woman presents to you with right lower leg pain after a fall while
hiking. You obtain radiography and evaluate her ankle, the findings of which
reveal a spiral fracture of the distal tibia.

Which of the following is the most appropriate next step in her evaluation?

No answer selected.

x-ray of the proximal fibula

x-ray of the cervical spine

arteriography of the popliteal artery

magnetic resonance imaging (MRI) of the ankle

Incorrect

Educational Objective:

Describe the appropriate workup of Maisonneuve fracture.

Key Point:

In all cases of a suspected Maisonneuve fracture, radiography of the


proximal fibula is indicated.

Explanation:

This patient may have a Maisonneuve fracture, so radiography of the proximal


fibula should be performed. A Maisonneuve fracture is a disruption of the medial
ankle that consists of a medial malleolar fracture or tearing of the deltoid
ligament, a complete tear of the syndesmotic ligament where the tibia and fibula
join, and a proximal fibula fracture. As a result of such an injury, the fibula "floats
free" in relation to the tibia, which can result in unstable ankle mortise, thereby
requiring surgical fixation.

Reference:

Palin DJ. Knee and lower leg. In: Walls R, et al, eds. Rosen's Emergency
Medicine: Concepts and Clinical Practice. 19th ed., 2018:698-722.

85. Incorrect

…e.com%2Fcustomscrip%2F154857%3Fscored%263739a18c-0c68-43cc-a4cb-b8b99e9bfd72%3Da45491cd-af54-4004-a990-00ee5e3871ae 121/144
2021/‫‏‬6/‫‏‬22 Immersive Reader

Question Tools:

An 18-year-old patient presents to the emergency department complaining of


shoulder pain after a fight.

On examination, the patient’s left arm is found to be hanging by his side. He is


incapable of completely adducting or internally rotating the arm (see Figure).

Figure.

Which of the following statements is correct?

No answer selected.

The anterior-posterior radiograph is usually nondiagnostic in this injury.

The weight-hanging method for addressing this injury is no longer


recommended.

Repeat x-rays are no longer recommended after successfully addressing this


injury as long as the post-treatment examination findings are normal.

…e.com%2Fcustomscrip%2F154857%3Fscored%263739a18c-0c68-43cc-a4cb-b8b99e9bfd72%3Da45491cd-af54-4004-a990-00ee5e3871ae 122/144
2021/‫‏‬6/‫‏‬22 Immersive Reader

The scapular method entails a medial rotation of the scapular tip, possibly
resulting in such a subtle correction of the injury that the end point is missed by
both the operator and the patient.

Incorrect

Educational objective:

Identify the clinical presentation of anterior shoulder dislocation.

Key point:

Multiple reduction methods for anterior shoulder dislocations exist; all have
advantages and disadvantages.

Explanation:

This patient is presenting with the classic signs of an anterior shoulder dislocation
(empty fossa in the Figure; limited internal rotation and adduction on
examination). In contrast to posterior dislocations, the anteroposterior radiography
in anterior dislocations is normally diagnostic. The younger the patient is at the
first occurrence of an anterior shoulder dislocation, the more likely he/she will
experience recurrence.

Multiple reduction methods exist that all have their advantages and disadvantages.
Increasingly, the single-operator methods are preferred over the 2-operator
methods simply because they do not require the simultaneous presence of 2 staff
members. The weight-on-arm method is still acceptable; the only methods no
longer advocated are Hippocrates ancient foot-in-armpit method and the Kocher
method. All methods require patience and gentleness rather than sudden
maneuvers.

The Stimson method of laying the patient prone and attaching weights (5-10
pounds) is time consuming (20-30 minutes) and entails the risk of the sedated
patient sliding off the bed, but it is approximately 90% successful and very simple
to execute with minimal staff required.

The popular traction-countertraction method is popular generally because of


wide-spread operator familiarity with it (the standard 2-person method). It is more
painful than several of the other methods and therefore requires more sedation.

The scapular manipulation method has gained in popularity, may work without
sedation, and is also about 90% successful. This method can be carried out in
various positions (seated, supine, prone) and entails a medial rotation of the
…e.com%2Fcustomscrip%2F154857%3Fscored%263739a18c-0c68-43cc-a4cb-b8b99e9bfd72%3Da45491cd-af54-4004-a990-00ee5e3871ae 123/144
2021/‫‏‬6/‫‏‬22 Immersive Reader

inferior scapular tip while/after the arm has been relaxed by traction (supine or
seated version) or hanging (prone version). The reduction may indeed be so subtle
that both the patient and the operator initially miss its completion.

Several other methods (eg, Milch method, forearm supination method) have also
been shown to be up to 90% successful without sedation and might be worthwhile
learning.

Postreduction x-rays are essential because, in some patients, the classic Hill-Sachs
deformity of the humeral head only becomes visible on postreduction film.

References:

Bengtzen R, Daya M. Shoulder (Chapter 46). In: Wall R, et al, eds. Rosen’s
Emergency Medicine: Concepts and Clinical Practice. 19th ed., 2018:549-568.e2.

Naples RM, Ufberg JW. Management of common dislocations. In: Roberts JR, et
al, eds. Roberts: Clinical Procedures in Emergency Medicine. 7th ed., 2019:980-
1026.e3.

86. Incorrect

Question Tools:

A patient has a PCO2 value greater than 45 mm Hg. This represents __ and could
be a primary problem (if the pH is low) or compensation for __ (if the pH is high).

No answer selected.

metabolic acidosis; respiratory alkalosis

respiratory acidosis; metabolic alkalosis

metabolic alkalosis; respiratory acidosis

respiratory alkalosis; metabolic acidosis

Incorrect

Incorrect!

An elevated PCO2 value implies alveolar hypoventilation, which could be


primary (eg, respiratory or neurologic disorder), resulting in acidemia (lowered
blood pH), or compensation for primary metabolic alkalemia (elevated blood pH).

…e.com%2Fcustomscrip%2F154857%3Fscored%263739a18c-0c68-43cc-a4cb-b8b99e9bfd72%3Da45491cd-af54-4004-a990-00ee5e3871ae 124/144
2021/‫‏‬6/‫‏‬22 Immersive Reader

It is worth pointing out that acidosis and alkalosis refer to the processes (the
insult, such as an acidotic process like hypoventilation) that result in changes in
blood pH (the end result, such as acidemia or alkalemia), respectively. With
acidemia, there is an actual decrease in blood pH (the result), whereas acidosis
refers to the processes (the insults) that attempt to decrease blood pH (which may
or may not be successful).

Reference:

Strayer RJ. Acid-base disorders. In: Marx JA, et al, eds. Rosen's Emergency
Medicine: Concepts and Clinical Practice. 8th ed., 2014.

87. Incorrect

Question Tools:

A patient has a PCO2 value less than 35 mm Hg, which represents __. This could
be a primary problem (if the pH is high: alkalemia) or compensation for __ (if the
pH is low: acidemia).

No answer selected.

respiratory acidosis; metabolic alkalosis

respiratory alkalosis; metabolic acidosis

metabolic alkalosis; respiratory acidosis

metabolic acidosis; respiratory alkalosis

Incorrect

Incorrect!

Any patient who is hyperventilating (eg, anxiety, hypoxia, acidemia) will have a
low PCO2 value, which is defined as respiratory alkalosis (the physiologic insult).
Counterintuitive presence of a low pH value (the physiologic result of acidemia
despite an alkalotic insult) in the context of a low PCO2 value indicates metabolic
acidosis with compensatory respiratory alkalosis (2 competing processes).

Reference:

Strayer RJ. Acid-base disorders. In: Marx JA, et al, eds. Rosen's Emergency
Medicine: Concepts and Clinical Practice. 8th ed., 2014.
…e.com%2Fcustomscrip%2F154857%3Fscored%263739a18c-0c68-43cc-a4cb-b8b99e9bfd72%3Da45491cd-af54-4004-a990-00ee5e3871ae 125/144
2021/‫‏‬6/‫‏‬22 Immersive Reader

88. Incorrect

Question Tools:

An 84-year-old man with coronary heart disease and hypertension presents to you
1 week after cardiac catheterization with worsening swelling and ecchymosis at
his femoral vascular access site. He reports that he recently underwent
catheterization for unstable angina, and his recovery was uneventful. Within 2
days of being home, however, he noted worsening ecchymosis at the right femoral
puncture site. He denies loss of sensation or distal foot pain.

On examination, he appears well and has normal vital signs. The femoral
catheterization site is notable for marked ecchymosis and hematoma with a
palpable thrill. A systolic bruit is noted on auscultation of the site. Pedal pulses
are intact and equal bilaterally.

Which of the following is the most appropriate next step in the diagnostic workup
of this patient?

No answer selected.

operative exploration

computed tomography angiography

diagnostic catheter-based angiography

duplex ultrasonography

Incorrect

Educational Objective:

Describe the clinical presentation and diagnostic workup of postcatheterization


pseudoaneurysm.

Key Point:

Hematoma with a bruit and thrill after vascular access should raise suspicion for
postprocedure pseudoaneurysm. Duplex ultrasonography is the diagnostic test of
choice.

Explanation:

…e.com%2Fcustomscrip%2F154857%3Fscored%263739a18c-0c68-43cc-a4cb-b8b99e9bfd72%3Da45491cd-af54-4004-a990-00ee5e3871ae 126/144
2021/‫‏‬6/‫‏‬22 Immersive Reader

Pseudoaneurysm is a well-described complication of arterial catheterization. The


most common risk factor is inadequate manual compression at the time of the
procedure, although the use of large-bore sheaths, postprocedural anticoagulation,
age older than 65 years, and obesity are also contributory. Patients usually
develop pseudoaneurysms within 3 to 7 days of catheterization, although late
presentations with symptoms of embolization or neuropathy are also common.
Within the early postprocedure period, clinical features include a large hematoma
with a thrill and bruit. Diagnosis is best achieved through duplex ultrasonography,
which offers the benefit of concurrent treatment through compression or thrombin
injection.

Pearl: This situation is not to be confused with the situation of a traumatic arterial
injury, in which the extent of the injury itself is unknown (different from
procedure-related puncture bleeding) and needs to be explored in more detail,
especially when associated with signs of distal ischemia. In the case of arterial
trauma, CT angiography or surgical exploration will often be the better choice
than duplex ultrasonography.

Reference:

Raja AS. Peripheral vascular injury. In: Walls R, et al. Rosen’s Emergency


Medicine: Concepts and Clinical Practice. 9th ed., 2018:435-444.

89. Incorrect

Question Tools:

A 19-year-old man injured in a motor vehicle collision presents to you with left
knee pain. He has tenderness across the lateral aspect of the knee on examination
without effusion. You obtain radiography, which shows a lateral tibial plateau
fracture.

Which of the following is true regarding tibial plateau fractures?

No answer selected.

Tibial plateau fractures are rarely intra-articular.

Most tibial plateau fractures involve the medial plateau.

Tibial plateau fractures produce a high percentage of vascular complications.

Magnetic resonance imaging (MRI) should be performed prior to disposition.

…e.com%2Fcustomscrip%2F154857%3Fscored%263739a18c-0c68-43cc-a4cb-b8b99e9bfd72%3Da45491cd-af54-4004-a990-00ee5e3871ae 127/144
2021/‫‏‬6/‫‏‬22 Immersive Reader

Incorrect

Educational Objective:

List the complications associated with tibial plateau fractures.

Key Point:

Fractures of the tibial plateau produce many vascular complications

Explanation:

Fractures of the tibial plateau produce many vascular complications. The popliteal
artery, which is immobile in this region, branches to the posterior and anterior
tibial arteries located at the upper portion of the interosseous membrane.
Fragments from bicondylar or comminuted fractures involving the subcondylar
area may cause injury to the popliteal artery, possibly resulting in distal
circulatory compromise due to vascular impairment. Displaced fractures of the
lateral condyle may also cause injury to the anterior tibial artery and produce
peroneal nerve paralysis. In most cases, stretching of the peroneal nerve is the
cause of injury.

Because a valgus stress with an abduction force on the leg usually is the initial
mechanism of injury, up to 70% of condylar fractures involve the lateral plateau.
Adduction forces on the distal leg may account for up to 23% of medial plateau
fractures; however, both plateaus can be involved in up to nearly one-third of
cases.

Although MRI is helpful in evaluating these fractures, it can be performed as an


outpatient procedure.

Because the tibial plateau accounts for most of the tibial joint surface at the knee,
these fractures are frequently intra-articular.

Reference:

Palin DJ. Knee and lower leg. In: Walls R, et al, eds. Rosen's Emergency
Medicine: Concepts and Clinical Practice. 19th ed., 2018:698-722.

90. Incorrect

Question Tools:

A 45-year-old man presents to you with confusion. He also strongly smells of


alcohol. You suspect that he may be hypoglycemic, so you want to correct his
…e.com%2Fcustomscrip%2F154857%3Fscored%263739a18c-0c68-43cc-a4cb-b8b99e9bfd72%3Da45491cd-af54-4004-a990-00ee5e3871ae 128/144
2021/‫‏‬6/‫‏‬22 Immersive Reader

hypoglycemia.

Which of the following statements is true regarding glucose administration in this


case?

No answer selected.

thiamine must be given before glucose to prevent Korsakoff syndrome

thiamine may avoid Korsakoff syndrome, but its administration should not
delay correction of hypoglycemia

glucose should be given without concern for thiamine

glucose should be repleted orally

Incorrect

Educational objective:

Recognize hypoglycemia induced by alcohol intoxication.

Key Point:

It is important for physicians to understand carbohydrate metabolism in patients


with alcohol use disorder.

Explanation:

This patient is acutely intoxicated with alcohol. However, it is important for


clinicians to have an index of suspicion for severe hypoglycemia in such cases,
because alcohol impairs gluconeogenesis in the liver and can lead to profound
hypoglycemia and sometimes death. Traditionally intravenous thiamine was
recommended  prior to correcting the hypoglycemia in order to prevent
irreversible Wernicke-Korsakoff syndrome, however rigorous studies evaluating
glucose and thiamine administration are lacking. Case reports suggest that if
thiamine is not immediately available, glucose should still be administered to
prevent the morbidity and mortality from hypoglycemia These same reports
suggest that prolonged glucose supplementation without the addition of thiamine
can, indeed, be a risk factor for Wernicke-Korsakoff syndrome, and so while it
should not delay correction of hypoglycemia, thiamine should be administered
once it is available.

…e.com%2Fcustomscrip%2F154857%3Fscored%263739a18c-0c68-43cc-a4cb-b8b99e9bfd72%3Da45491cd-af54-4004-a990-00ee5e3871ae 129/144
2021/‫‏‬6/‫‏‬22 Immersive Reader

In this case, administration of oral glucose will not be effective as intravenous


repletion, because absorption may be slow; in addition, other deficiencies do not
have to be corrected as emergently as thiamine.

References:

Latt N, Dore G. “Thiamine in the treatment of Wernicke encephalopathy in


patients with alcohol use disorders.” Intern Med J 2014, 44(9):911-915.

Merlin MA, Carluccio A, Raswant N, Dossantos F, Ohman-Strickland P, Lehrfeld


DP. “Comparison of prehospital glucose with or without IV thiamine.” West J
Emerg Med 2012, 13(5):406-409.

Schabelman E, Kuo D. “Glucose before thiamine for Wernicke encephalopathy: a


literature review.” J Emerg Med 2012, 42(4):488-494.

91. Incorrect

Question Tools:

A decrease in blood pH of 0.1 will tend to be associated with what change in


plasma potassium?

No answer selected.

0.5 mEq/L increase

0.2 mEq/L decrease

0.5 mEq/L decrease

0.2 mEq/L increase

Incorrect

A decrease in blood pH of 0.1 will tend to be associated with an increase of 0.5


mEq/L in plasma potassium. Acidemia increases plasma potassium levels,
whereas alkalemia will reduce potassium levels.

Reference:

Strayer RJ. Acid-base disorders. In: Marx JA, et al, eds. Rosen's Emergency
Medicine: Concepts and Clinical Practice. 8th ed., 2014.

…e.com%2Fcustomscrip%2F154857%3Fscored%263739a18c-0c68-43cc-a4cb-b8b99e9bfd72%3Da45491cd-af54-4004-a990-00ee5e3871ae 130/144
2021/‫‏‬6/‫‏‬22 Immersive Reader

92. Incorrect

Question Tools:

A 33-year-old homeless man who abuses alcohol is evaluated in the emergency


department. His laboratory studies reveal mild acidemia (pH of 7.28), a sodium
level of 133, a potassium value of 3.3, a chloride value of 100, and a bicarbonate
value of 22. His albumin value is 3.0. The normal anion gap for your laboratory
analyzer is 12 to 16.

The patient's anion gap is _________________. A normal anion gap for this
patient is ______________.

No answer selected.

15; the same for every patient regardless of other parameters

22; 10-13

8; 13-16

11; 9-13

Incorrect

Determining the anion gap should take into account the effects of
hypoalbuminemia. The anion gap decreases by approximately 3 for every 1 g/dL
decrement in serum albumin. Assuming a normal anion gap of 12 to 16 (given the
particular laboratory mentioned in the question) with a normal albumin value (4
g/dL), an albumin value of 3 in the present case represents a drop by 1 g/dL; thus,
the anion gap should decrease by 3 (now 9-13). The anion gap is calculated using
the formula:

Na – (Cl + HCO3).

Reference:

Charney AN, Hoffman RS. Fluid, electrolyte, and acid-base disorders. In: Nelson
LS, et al, eds. Goldfrank's Toxicologic Emergencies. 9th ed., 2011.

93. Incorrect

Question Tools:

…e.com%2Fcustomscrip%2F154857%3Fscored%263739a18c-0c68-43cc-a4cb-b8b99e9bfd72%3Da45491cd-af54-4004-a990-00ee5e3871ae 131/144
2021/‫‏‬6/‫‏‬22 Immersive Reader

In respiratory alkalosis, if the PCO2 value goes below approximately __, a vicious
cycle may result, with hypocarbia causing cerebral vasoconstriction, which in turn
causes ischemic brain stem metabolic acidosis, stimulating ventilation, and
causing worsening respiratory alkalosis.

No answer selected.

30 mm Hg

25 mm Hg

20 mm Hg

10 mm Hg

Incorrect

This is a classic example of a pathophysiologic vicious cycle. The fact that


cerebral vasoconstriction occurs as a reflex to hypocarbia is sometimes
therapeutically used in intubated patients with signs of brain herniation and in
whom cerebral edema may be temporarily decreased through mechanical
hyperventilation. 

Although it was once recommended, hyperventilation is no longer part of current


management guidelines in all patients with cerebral edema because studies have
shown that hyperventilation does reduce cerebral edema but at the price of
increasing cerebral ischemia. For this reason, hyperventilation (PCO2 of 30-35
mm Hg) is now only recommended to reduce intracranial pressure for a short
period in severely ill patients (eg, signs of brain herniation), and it can only be
used as a temporizing measure while other methods of intracranial pressure
control are initiated. Hyperventilation loses effectiveness after 16 hours.

References:

Constantine M, Jogoda J. Brain trauma foundation revises guidelines. Accessed


October 17, 2015.

Rangel-Castilla L. Closed Head Injury Treatment & Management. Accessed


October 17, 2015.

Strayer RJ. Acid-base disorders. In: Marx JA, et al, eds. Rosen's Emergency
Medicine: Concepts and Clinical Practice. 8th ed., 2014.

94. Incorrect

…e.com%2Fcustomscrip%2F154857%3Fscored%263739a18c-0c68-43cc-a4cb-b8b99e9bfd72%3Da45491cd-af54-4004-a990-00ee5e3871ae 132/144
2021/‫‏‬6/‫‏‬22 Immersive Reader

Question Tools:

A 35-year-old man complains of episodes of confusion, sweating, palpitations,


and lightheadedness approximately 2 to 3 hours after eating for the past month.
His medical history is notable for gastric bypass surgery 6 months ago,
hypertension, and hyperlipidemia. He takes hydrochlorothiazide and rosuvastatin.
He does not smoke cigarettes, drink alcohol, or use illicit drugs. He sells antique
cars for a living.

A review of systems is notable for an 80-pound weight loss. His body mass index
is 32 kg/m2. Physical examination findings are otherwise normal.

During one of these episodes, his blood glucose level is 35 mg/dL, his insulin
level is 86 mIU/L (normal < 30), and his C-peptide is 7.3 ng/mL (normal < 0.6).

What is the most likely cause of his symptoms?

No answer selected.

dumping syndrome

exogenous insulin use

nesidioblastosis

vitamin B12 deficiency

adrenal insufficiency

Incorrect

Educational Objective:

Discuss the differential diagnosis of hypoglycemia.

Key Point:

The physician should recognize the signs of nesidioblastosis.

Explanation:

The most likely cause in this patient is nesidioblastosis. Hypoglycemia can be


either insulin-mediated or non–insulin-mediated. In non–insulin-mediated
hypoglycemia, such as adrenal insufficiency or liver failure, the insulin level in
the blood during hypoglycemia is appropriately low. In insulin-mediated

…e.com%2Fcustomscrip%2F154857%3Fscored%263739a18c-0c68-43cc-a4cb-b8b99e9bfd72%3Da45491cd-af54-4004-a990-00ee5e3871ae 133/144
2021/‫‏‬6/‫‏‬22 Immersive Reader

hypoglycemia, the insulin level is inappropriately not suppressed. If the C-peptide


level is low, the insulin results from an exogenous injection. If the C-peptide level
is elevated with the insulin, then the insulin comes from islet cells; thus, the
differential diagnosis should include insulinoma, sulfonylurea ingestion, and islet-
cell hyperplasia (nesidioblastosis), which is associated with weight loss after
gastric bypass surgery and can in turn cause dumping syndrome (but the dumping
syndrome does not cause the hypoglycemia).

Reference:

Goldman L, Schafer AI. Goldman's Cecil Medicine. 24th ed., 2012.

95. Incorrect

Question Tools:

A 72-year-old woman with a history of diverticulosis and prior carotid


endarterectomy presents from her skilled nursing facility where she is a resident.
She has bright red blood in her rectum and hypotension.

Upon arrival, she is awake but confused. Her vital signs are: heart rate 120
beats/minute, blood pressure 75/45 mm Hg, respiratory rate 18 breaths/minute,
and oxygen saturation 96% on room air.

Peripheral vascular access is noted to be difficult, so the decision is made to place


a 9-Fr introducer. As one inferior jugular vein is thrombosed, and the other is
beneath the site of her prior endarterectomy, catheter placement is attempted in
the right subclavian. The tract is dilated, and, upon threading the sheath, the
output through the catheter is noted to be bright red and pulsatile.

Which of the following statements about this scenario is correct?

No answer selected.

Use of ultrasonography decreases risk of iatrogenic pneumothorax but does not


decrease the risk of this particular complication.

Endovascular methods cannot treat this complication.

The introducer should be removed immediately.

Bedside manometry prior to the procedure could have avoided this


complication.

…e.com%2Fcustomscrip%2F154857%3Fscored%263739a18c-0c68-43cc-a4cb-b8b99e9bfd72%3Da45491cd-af54-4004-a990-00ee5e3871ae 134/144
2021/‫‏‬6/‫‏‬22 Immersive Reader

Incorrect

Educational Objective:

Identify the risk factors and management of inadvertent arterial injury during
central venous catheterization.

Key Point:

Bedside manometry and use of ultrasonography can prevent inadvertent arterial


cannulation during central line placement. If arterial injury occurs, then the
catheter should be kept in place and an endovascular specialist consulted for
management.

Explanation:

Return of bright red, pulsatile blood is concerning for subclavian artery


cannulation, a complication more likely to occur in emergent settings. Use of
manometry and ultrasonography can diminish the risk of this complication. If
inadvertent arterial cannulation occurs, then removal of the catheter is associated
with a high risk of hemorrhage, even in compressible sites that allow for
application of pressure (eg, femoral vessels). The best management in the event of
inadvertent arterial cannulation is to leave the catheter in place and to consult
vascular surgery or a similar specialist in endovascular treatment.

Manometry consists of placing a hub with a side port on the initial needle and
measuring the pressure within the punctured vessel. In this way, entry into a high-
pressure artery can be detected prior to dilation of the artery

References:

Bowdle A. Vascular complications of central venous catheter placement:


evidence-based methods for prevention and treatment. J Cardiothoracic Vascular
Anesth. 2014;28(2):358-368.

Brass P, Hellmich M, Kolodziej L, Schick G, Smith AF. Ultrasound guidance


versus anatomical landmarks for subclavian or femoral vein catheterization.
Cochrane Database Syst Rev. 2015:CD011447.

Dixon OG, Smith GE, Carradice D, Chetter IC. A systematic review of


management of inadvertent arterial injury during central venous catheterization. J
Vasc Access. 2017;18(2):97-102.

Raja AS. Peripheral vascular injury. In: Walls R, et al. Rosen’s Emergency
Medicine: Concepts and Clinical Practice. 9th ed., 2018:435-444.

…e.com%2Fcustomscrip%2F154857%3Fscored%263739a18c-0c68-43cc-a4cb-b8b99e9bfd72%3Da45491cd-af54-4004-a990-00ee5e3871ae 135/144
2021/‫‏‬6/‫‏‬22 Immersive Reader

96. Incorrect

Question Tools:

A 28-year-old man is brought in by emergency medical services (EMS) after


sustaining a stab wound to the left thigh. Per the EMS report, he was attacked by
his partner with a kitchen knife. There was a significant amount of hemorrhage
noted at the scene.

Upon arrival, the patient is awake and alert but combative. Vital signs are: heart
rate 100 beats/minute, blood pressure 135/85 mm Hg, respiratory rate 16
breaths/minute, and oxygen saturation 98% on room air. Examination of the left
thigh reveals no active bleeding, but it does demonstrate an expanding hematoma.
His pedal pulses are diminished by handheld Doppler ultrasonography, and he
appears to have a femoral nerve deficit.

Which of the following features of this patient’s history and physical examination
are "hard" signs of a significant vascular injury?

No answer selected.

diminished pulses

femoral nerve deficit

expanding hematoma

significant hemorrhage at the scene

Incorrect

Educational Objective:

List "hard" signs of a significant vascular injury.

Key Point:

Pulsatile hemorrhage, expanding hematoma, absent distal pulses, palpable thrill,


and audible bruit are "hard" signs of arterial injury, and their presence in a trauma
patient is often best evaluated either with computed tomography angiography or
operative exploration (ultrasound may be adequate in certain situations).

Explanation:

…e.com%2Fcustomscrip%2F154857%3Fscored%263739a18c-0c68-43cc-a4cb-b8b99e9bfd72%3Da45491cd-af54-4004-a990-00ee5e3871ae 136/144
2021/‫‏‬6/‫‏‬22 Immersive Reader

"Hard" signs of vascular injury are predictive of arterial damage in 90% of cases.
Such findings include pulsatile hemorrhage, expanding hematoma, absent distal
pulses, palpable thrill, and an audible bruit. When these are present in a trauma
patient (especially in the presence of distal ischemia), such findings should be
promptly investigated with computed tomography angiography or, if the patient is
unstable or the duration of warm ischemia has been significant, with immediate
surgical intervention.

"Soft" signs of vascular injury are also significant, but less predictive findings: Up
to 35% of patients demonstrate arterial injury on imaging. Included among soft
findings are a history of significant hemorrhage at the scene, nonexpanding
hematoma, diminished pulses or ankle brachial index of the injured extremity,
extremity peripheral nerve deficit, and bony injury or proximate penetrating
wound. Patients with soft signs of vascular injury should undergo diagnostic
imaging, but only a small proportion of such patients will require emergent repair
of arterial injuries.

Pearl: This situation is not to be confused with the situation of a post-procedure


arterial hematoma eg of the femoral artery after a radiological procedure in the
catheter lab. The presentation may be similar (hematoma, thrill, etc), but the
context and severity are different and the size of the arterial injury is known.
Evaluation of a bleeding post-puncture site is best achieved through duplex
ultrasonography, which offers the benefit of concurrent treatment through
compression of, or thrombin injection into the already known arterial injury site.

Reference:

Raja AS. Peripheral vascular injury. In: Walls R, et al. Rosen’s Emergency


Medicine: Concepts and Clinical Practice. 9th ed., 2018:435-444.

97. Incorrect

Question Tools:

The most common cause of endogenous hyperinsulinemic hypoglycemia is which


of the following?

No answer selected.

noninsulinoma pancreatogenous hypoglycemia syndrome

congenital hypoglycemia

insulin autoimmune syndrome


…e.com%2Fcustomscrip%2F154857%3Fscored%263739a18c-0c68-43cc-a4cb-b8b99e9bfd72%3Da45491cd-af54-4004-a990-00ee5e3871ae 137/144
2021/‫‏‬6/‫‏‬22 Immersive Reader

insulinoma

Incorrect

Educational Objective:

Discuss the differential diagnosis of hypoglycemia.

Key Point:

Physicians should be able to recognize the most common causes of hypoglycemia.

Explanation:

The 2 most common conditions associated with excess production of certain


hormones include hyperinsulinemic hypoglycemia and non–islet-cell tumor
hypoglycemia (also known as insulin-like growth factor 2 [IGF-2]-oma).

Insulinomas have the highest incidence in the fifth and sixth decades of life. They
are insulin-secreting tumors of pancreatic origin. They are benign, solitary, and
less than 2 cm in diameter. The symptoms are more evident in patients who are
fasting or after physical exercise.

Insulin autoimmune syndrome is associated with high titers of antibodies to


endogenous insulin; however, this condition rarely occurs in western countries.
Islet-cell tumor is also rarely seen, as is congenital hypoglycemia. 

Noninsulinoma pancreatogenous hypoglycemia syndrome is associated with islet


hypertrophy and nesidioblastosis. Fasting hypoglycemia is rare and the condition
is characterized by postprandial hypoglycemia.

The diagnosis of insulinoma is based on findings of abnormal serum levels of


insulin and C-peptide at the time of fasting hypoglycemia.

It is important to remember that these insulinomas can occur in isolation or as part


of multiple endocrine neoplasia type 1, so it is important to rule out this
association.

References:

Goldman L, Schafer AI. Goldman's Cecil Medicine. 24th ed., 2012.

Service FJ, Vella A. Insulinoma. Updated December 2016. Accessed March 6th,
2018.

…e.com%2Fcustomscrip%2F154857%3Fscored%263739a18c-0c68-43cc-a4cb-b8b99e9bfd72%3Da45491cd-af54-4004-a990-00ee5e3871ae 138/144
2021/‫‏‬6/‫‏‬22 Immersive Reader

98. Incorrect

Question Tools:

A 56-year-old man with a history of substance abuse is brought in by emergency


medical services (EMS) following a motor vehicle collision. Per the EMS report,
the patient was the restrained driver in a head-on collision at 35 miles/hour. He
extricated himself from the vehicle, but he is unable to walk because he cannot
bear weight on his right leg.

Upon arrival, the patient is alert but intoxicated. Primary survey reveals no
immediately life-threatening injuries. His vital signs are: heart rate 90
beats/minute, blood pressure 140/90 mm Hg, respiratory rate 12 breaths/minute,
and oxygen saturation 96% on room air. Secondary survey is notable for
tenderness and ecchymotic swelling of the right knee.

Manipulation of the joint reveals ligamentous disruption with significant


anteroposterior instability.

Which potential injury is the MOST important to evaluate in this patient?

No answer selected.

acetabular fracture

tibial plateau fracture

posterior tibial artery injury

popliteal artery injury

Incorrect

Educational Objective:

Summarize risk factors for popliteal artery injury after blunt trauma.

Key Point:

Posterior knee dislocation is associated with a high risk of popliteal artery injury.
Patients with suspected spontaneously reduced posterior dislocation should
undergo a careful neurovascular examination.

Explanation:

…e.com%2Fcustomscrip%2F154857%3Fscored%263739a18c-0c68-43cc-a4cb-b8b99e9bfd72%3Da45491cd-af54-4004-a990-00ee5e3871ae 139/144
2021/‫‏‬6/‫‏‬22 Immersive Reader

This patient’s history of a high-energy trauma mechanism and the finding of


ligamentous disruption of the knee are concerning for spontaneously reduced knee
dislocation. Diagnosis of vascular injury can be missed in such scenarios and later
misconstrued as compartment syndrome. A thorough neurovascular evaluation,
including ankle brachial indices, is crucial.

The most common vascular injuries following posterior knee dislocation are to
the popliteal artery, which can be limb threatening if missed. While tibial plateau
fractures and acetabular fractures may coincide with knee dislocation, the most
crucial injury to evaluate is potential popliteal artery damage. Because the
posterior tibial artery courses distal to the patella, an isolated injury to this vessel
is less likely.

Reference:

Raja AS. Peripheral vascular injury. In: Walls R, et al. Rosen’s Emergency
Medicine: Concepts and Clinical Practice. 9th ed., 2018:435-444.

99. Incorrect

Question Tools:

Which of the following statements regarding patellar fractures is correct?

No answer selected.

A displaced patellar fracture is defined as a fracture with > 3 mm anterior or


posterior step-off between any 2 patellar fragments.

All patella fractures (except minor rim avulsions) are intra-articular.

All transverse patellar fractures result in a complete loss of knee extension.

Direct trauma is the most common mechanism for patella fractures.

Incorrect

Educational Objective:

Identify patellar fractures.

Key Point:

…e.com%2Fcustomscrip%2F154857%3Fscored%263739a18c-0c68-43cc-a4cb-b8b99e9bfd72%3Da45491cd-af54-4004-a990-00ee5e3871ae 140/144
2021/‫‏‬6/‫‏‬22 Immersive Reader

All fractures of the patella, with the exception of small avulsion fractures of the
rim, are considered intra-articular.

Explanation:

With the exception of minor rim avulsions, all patella fractures are intra-articular;
thus, a break in the skin over a patella fracture may indicate an open joint. Open
patella injuries are surgical emergencies that require orthopedic consultation.
Patellar fractures not classified as intra-articular include small vertical avulsions
of the patellar edge at the site of tendon insertion, as well as marginal fractures.

Fracture displacement involves the abnormal position of the distal fracture


fragment as it relates to the proximal bone, and it may result in rotation,
angulation, loss of alignment, change of bone length, or a combination of all these
features. Displacement of a patellar fracture is defined as any separation of
fragments greater than 3 mm in any direction. Fragment separation may be purely
longitudinal with no step-off between the fragments.

Patients with a displaced patellar fracture will typically lose all knee extension.
By contrast, even with transverse fractures, patients with a nondisplaced fracture
and an intact retinaculum will have some degree of preserved knee extension.
Displacement implies a complete tear in the retinaculum and indicates that
surgical repair is necessary.

Indirect trauma during forceful contraction of the quadriceps against a flexed knee
causes 50% to 80% of patellar fractures, usually transverse. Direct trauma caused
by the knee striking the dashboard during a motor vehicle collision is another
common mechanism.

After orthopedic consultation, patients with closed fractures may be discharged in


a knee immobilizer (knee in full extension) with outpatient orthopedic follow-up.
Patients with displaced fractures may require surgical repair, but this can also be
managed on an outpatient basis.

A bipartite or multipartite patella can mimic a fracture. Smooth, sclerotic margins


distinguish these genetic variations from true fractures, although fractures may
also coexist.

References:

Hals GD, Cruea S, Moses D. Evaluation of the acutely injured knee in the ED:
diagnosis and treatment: part I. Published April 29, 2007. Emerg Med
Rep. Accessed January 18, 2018.

…e.com%2Fcustomscrip%2F154857%3Fscored%263739a18c-0c68-43cc-a4cb-b8b99e9bfd72%3Da45491cd-af54-4004-a990-00ee5e3871ae 141/144
2021/‫‏‬6/‫‏‬22 Immersive Reader

Lamoureux C. Patella fracture imaging. Published November 13, 2015. Accessed


January 18, 2018.

Palin DJ. Knee and lower leg. In: Walls R, et al, eds. Rosen's Emergency
Medicine: Concepts and Clinical Practice. 19th ed., 2018:698-722.

Radiology Masterclass website. Introduction to trauma x-ray. Accessed January


18, 2018.

100. Incorrect

Question Tools:

A 52-year-old man with type 2 diabetes mellitus presents to you with left knee
pain after he tripped on a curb. On examination, he cannot extend his knee. There
is tenderness over the patella and the patella is inferiorly displaced.

This presentation is most consistent with which of the following?

No answer selected.

patella dislocation

anterior cruciate ligament (ACL) tear

quadriceps tendon rupture

patellar tendon rupture

Incorrect

Educational Objective:

Identify the clinical presentation of quadriceps tendon rupture.

Key Point:

With a quadriceps rupture, a low-riding patella (patella baja) and inferior


retraction may be seen.

Explanation:

A rupture of the quadriceps tendon is more common in patients with systemic


disease, such as those with gout, chronic renal failure, diabetes,
hyperparathyroidism, and obesity, compared with the general population. Those
…e.com%2Fcustomscrip%2F154857%3Fscored%263739a18c-0c68-43cc-a4cb-b8b99e9bfd72%3Da45491cd-af54-4004-a990-00ee5e3871ae 142/144
2021/‫‏‬6/‫‏‬22 Immersive Reader

with degenerative arthritic changes in the knee may also be susceptible. However,
patellar tendon injuries in this patient population are less common.

These types of ruptures will present with inferior displacement of the patella
(patella baja), proximal ecchymosis, and swelling. By contrast, a rupture of the
patellar tendon is indicated by swelling, proximal patellar displacement (patella
alta), and inferior pole tenderness.

Reference:

Palin DJ. Knee and lower leg. In: Walls R, et al, eds. Rosen's Emergency
Medicine: Concepts and Clinical Practice. 19th ed., 2018:698-722.

101. Incorrect

Question Tools:

A 19-year-old man with type 1 diabetes mellitus complains of frequent vivid


dreams, nightmares, and headaches when he wakes in the morning. He takes 20
units of 70/30 insulin at night and uses a carbohydrate-counting sliding scale of
insulin lispro with meals. He has no known complications of diabetes.

Which of the following is the most likely to help with his symptoms?

No answer selected.

Increase his dose of 70/30 insulin, because it appears that he may be


experiencing symptoms of severe hyperglycemia.

Eat a nightly bedtime snack.

Change his regimen from 70/30 insulin to glargine basal insulin and insulin
lispro mealtime insulin and then check his nocturnal glucose level.

Add metformin to his regimen to prevent the liver from excessively secreting
glucose.

Incorrect

Educational objective:

Identify the signs and symptoms of Somogyi phenomenon.

Key point:

…e.com%2Fcustomscrip%2F154857%3Fscored%263739a18c-0c68-43cc-a4cb-b8b99e9bfd72%3Da45491cd-af54-4004-a990-00ee5e3871ae 143/144
2021/‫‏‬6/‫‏‬22 Immersive Reader

Physicians should understand the underlying pathology of Somogyi phenomenon


to better treat patients with the condition.

Explanation:

The physician should adjust the patient's medication from 70/30 insulin to
glargine basal insulin and insulin lispro mealtime insulin and then check his
nocturnal glucose level.

Nocturnal hypoglycemia may present with vivid dreams, nightmares, daytime


somnolence, or morning headaches; in some cases, it may be asymptomatic.
Following nocturnal hypoglycemia, the morning blood sugar level may be high
due to either carbohydrate intake to treat the hypoglycemia or counterregulatory
hormones--collectively known as the Somogyi effect.

Although eating a snack with complex carbohydrates or proteins makes intuitive


sense for preventing nocturnal hypoglycemia, studies of this effect have been
inconsistent. Simple carbohydrates at bedtime will not provide lasting protection
against hypoglycemia. Long-acting basal insulins, such as glargine, may provide
protection from nocturnal hypoglycemia in some cases.

…e.com%2Fcustomscrip%2F154857%3Fscored%263739a18c-0c68-43cc-a4cb-b8b99e9bfd72%3Da45491cd-af54-4004-a990-00ee5e3871ae 144/144

You might also like